You are on page 1of 160

Fluid Mechanics

in
Chemical Engineering

Eun-Suok Oh
2022

School of Chemical Engineering, University of Ulsan


2

This classnote has been written based on the textbooks for fluid mechanics, Introduction to Chem-
ical Engineering Fluid Mechanics, W. M. Deen, 2016 and Fundamentals of Momentum, Heat, and
Mass Transfer, J. R. Welty, C. E. Wicks, R. E. Wilson and G. L. Rorrer, 5th ed., 2009 to teach
students in University of Ulsan. Many of paragraphs, figures, and tables of those are inserted without
editing. Some of figures and examples are also from the other textbooks, Transport Phenomena, R. B.
Bird, W. E. Stewart, and E. N. Lightfoot, 2nd ed., 2002 and Unit Operations of Chemical Engineer-
ing, W. L. McCabe, J. C. Smith, and P. Harriott, 7th ed., 2010. Most of examples in this note are
extracted from these textbooks. It is written only for undergraduate students who are taking
my Fluid Mechanics course. Commercial use of this classnote is strictly prohibited in
any forms. Any types of transfer to others are not allowed due to the copyright of the
books.
3

G03312
Fluid Mechanics

Instructor: Eun-Suok Oh (오은석)


Office: 401 Chem. Eng. Building,
Phone: 259-2783
Email: esoh1@ulsan.ac.kr

Books: Textbook
· W. M. Deen, Introduction to Chemical Engineering Fluid Mechanics, Cambridge Univer-
sity Press (2016): reference ChD . and SectD .
Additional References

· J. R. Welty, C. E. Wicks, & R. E. Wilson, Fundamentals of Momentum, Heat, and Mass


Transfer, 5th ed., John Wiley & Sons, Inc. (2007)
· W. L. McCabe, J. C. Smith, & P. Harriott, Unit Operations of Chemical Engineering,
7th ed., McGraw-Hill (2007)
· R. B. Bird, W. E. Stewart, & E. N. Lightfoot, Transport Phenomena, 2nd ed., John Wiley
& Sons, Inc. (2007)
· J. C. Slattery, Advanced Transport Phenomena, Cambridge University Press (1999)
· J. C. Slattery, L. M. C. Sagis, & Eun-Suok Oh, Interfacial Transport Phenomena, 2nd ed.,
Springer (2007)
· R. Aris, Vectors, Tensors, and the Basic Equations of Fluid Mechanics, Dover Publications,
Inc. (1990)

Grading
· Class Attendance (10%): Attendance at class is compulsory. In the case that students won’t
make it for some reasons, let me know in advance.
· Quiz (30%): On-line Quizzes will be given in the beginning of the first class of every week.
· Midterm Examination (30%): The midterm exam is scheduled at the midpoint of the course
and will cover material up to that point.
· Final Examination (30%): The final exam is scheduled at the last day of classes and will
cover the material after the midterm exam.

Office Hours

No regular time is assigned for office hours. I would be happy to meet any of you anytime
I am free.

Comments

The aim of this class is for undergraduate students to understand basic knowledge on the
engineering mechanics for fluid. This class begins by the vector and tensor analysis and
introduces basic conservation laws such as mass, momentum, and energy conservation. Note
that I WON’T follow the order in the main textbooks during the classes. Rather, I will
make the order more systematically in the viewpoint of sophomores. In addition, some of
stuff used in this class will be from the above references as well as internet data sources.
4

유체역학을 수강하면
1 흐르는 유체에서 질량이 어떻게 보존되는 이해할 수 있다.
2 뉴튼의 2법칙은 단순히 힘은 질량 곱하기 가속도 뿐만 아니라, 이로 부터 물체에 주어진 힘이
운동량을 어떻게 변화시키는지 알 수 있다.
3 힘은 물체 안에 작용하는 body force (중력 등)와 표면에 작용하는 surface force (압력, 마찰력
등)로 구성 됨을 알 수 있다.
4 점도의 물리적 의미와 변할 수 있는 값임을, 또한 점도가 물체의 운동량을 전달하는 주 원인
임을 알 수 있다.
5 역학적에너지가 항상 보존되는 것만은 아니라는 것을 알 수 있다.
6 모든 점에서 압력에너지+운동에너지+퍼텐셜에너지 합은 항상 일정 (베르누이식)을 알 수
있다.
7 산업현장에서 사용하는 펌프의 사양을 이해할 수 있다.
8 유체가 파이프나 판 사이에서 압력 또는 중력에 의하여 흐를 때 속도가 포물선 형태가 됨을 알
수 있다.
9 유체가 파이프 내 또는 유동층 반응기에서 흘러갈 때 마찰에 의하여 떨어지는 단위길이당
압력강하를 알 수 있다.
10 공기중에 기포 또는 방울은 일정한 속도로 떨어짐을 알 수 있다.
Contents

1 Preliminary for Fluid Mechanics 9


1.1 What is Fluid Mechanics for Chemical Engineers? (Deen’s book) . . . . . . . . . . . . 9
1.2 What is Transport Phenomena? . . . . . . . . . . . . . . . . . . . . . . . . . . . . . . . 9
1.3 Scalars and Vectors . . . . . . . . . . . . . . . . . . . . . . . . . . . . . . . . . . . . . . 10
1.3.1 Definition of Scalars and Vectors . . . . . . . . . . . . . . . . . . . . . . . . . . 10
1.3.2 Vector Basis and Components . . . . . . . . . . . . . . . . . . . . . . . . . . . . 11
1.3.3 Dot (Scalar or Inner) Product of Two Vectors . . . . . . . . . . . . . . . . . . . 12
1.3.4 Cross (Vector) Product of Two Vectors . . . . . . . . . . . . . . . . . . . . . . 12
1.3.5 Definition of Tensors . . . . . . . . . . . . . . . . . . . . . . . . . . . . . . . . . 13
1.3.6 Properties of Tensors . . . . . . . . . . . . . . . . . . . . . . . . . . . . . . . . . 14
1.4 Vector Differential Operator . . . . . . . . . . . . . . . . . . . . . . . . . . . . . . . . . 14
1.4.1 Gradient of a Scalar . . . . . . . . . . . . . . . . . . . . . . . . . . . . . . . . . 16
1.4.2 Gradient of a Vector . . . . . . . . . . . . . . . . . . . . . . . . . . . . . . . . . 16
1.4.3 Divergence of a Vector . . . . . . . . . . . . . . . . . . . . . . . . . . . . . . . . 17
1.4.4 Divergence of a Tensor . . . . . . . . . . . . . . . . . . . . . . . . . . . . . . . . 17
1.4.5 Curl of a Vector . . . . . . . . . . . . . . . . . . . . . . . . . . . . . . . . . . . 18
1.4.6 Laplacian of a Scalar . . . . . . . . . . . . . . . . . . . . . . . . . . . . . . . . . 18
1.5 Vector Differential Operators in Other Coordinates (App. A,B) . . . . . . . . . . . . . 18
1.5.1 Rectangular Cartesian Coordinates . . . . . . . . . . . . . . . . . . . . . . . . . 18
1.5.2 Cylindrical Coordinates . . . . . . . . . . . . . . . . . . . . . . . . . . . . . . . 20
1.5.3 Spherical Coordinates . . . . . . . . . . . . . . . . . . . . . . . . . . . . . . . . 22

2 Basic Concepts of Fluid Mechanics 23


2.1 Fluids and Mechanics . . . . . . . . . . . . . . . . . . . . . . . . . . . . . . . . . . . . 23
2.1.1 Fluids and Solids . . . . . . . . . . . . . . . . . . . . . . . . . . . . . . . . . . . 23
2.1.2 Mechanics . . . . . . . . . . . . . . . . . . . . . . . . . . . . . . . . . . . . . . . 23
2.2 Motion of a Fluid . . . . . . . . . . . . . . . . . . . . . . . . . . . . . . . . . . . . . . . 24
2.3 Time Derivatives: Partial, Total, Material (Substantial) . . . . . . . . . . . . . . . . . 25
2.3.1 Relationship between total and partial time derivatives . . . . . . . . . . . . . 27
2.3.2 Velocity and Acceleration . . . . . . . . . . . . . . . . . . . . . . . . . . . . . . 28
2.3.3 Steady and Unsteady Flows . . . . . . . . . . . . . . . . . . . . . . . . . . . . . 28
2.4 Fundamental Physics Laws . . . . . . . . . . . . . . . . . . . . . . . . . . . . . . . . . 28
2.4.1 Conservation of Mass (질량보존의 법칙) . . . . . . . . . . . . . . . . . . . . . . 28
2.4.2 Newton’s Laws of Motion . . . . . . . . . . . . . . . . . . . . . . . . . . . . . . 29
2.4.3 Thermodynamic Laws . . . . . . . . . . . . . . . . . . . . . . . . . . . . . . . . 31

3 Conservation of Mass 33
3.1 Mass Balance for a Steady-State Flow . . . . . . . . . . . . . . . . . . . . . . . . . . . 35
3.2 Equation of Continuity . . . . . . . . . . . . . . . . . . . . . . . . . . . . . . . . . . . . 36
3.2.1 The Differential Mass Balance for an Incompressible Flow . . . . . . . . . . . . 39

5
6 CONTENTS

4 Conservation of Linear Momentum and Its Derivatives 43


4.1 Force Acting on a Fluid . . . . . . . . . . . . . . . . . . . . . . . . . . . . . . . . . . . 43
4.2 Interfacial Tension Force (Surface tension, Interfacial energy) . . . . . . . . . . . . . . 46
4.3 Linear Momentum and Angular Momentum . . . . . . . . . . . . . . . . . . . . . . . . 50
4.4 Linear Momentum Balance for a Steady Flow . . . . . . . . . . . . . . . . . . . . . . . 51
4.4.1 Linear Momentum Balance over a Moving Control Volume . . . . . . . . . . . 54
4.5 Stress : Surface Force per Unit Area at a Point . . . . . . . . . . . . . . . . . . . . . . 56
4.5.1 Stress Vector and Stress Tensor . . . . . . . . . . . . . . . . . . . . . . . . . . . 56
4.5.2 Viscosity . . . . . . . . . . . . . . . . . . . . . . . . . . . . . . . . . . . . . . . 58
4.5.3 Types of Fluids (Newtonian and non-Newtonian) . . . . . . . . . . . . . . . . . 60
4.6 Equation of Motion . . . . . . . . . . . . . . . . . . . . . . . . . . . . . . . . . . . . . . 63
4.6.1 The Euler Equation for an Inviscid Flow (∇ · τ ≈ 0) . . . . . . . . . . . . . . . 66
4.6.2 The Navier-Stokes Equation for an Incompressible Flow of a Newtonian Fluid . 66
4.7 Mechanical Energy Balance (NOT Conserved): v · 2nd Law . . . . . . . . . . . . . . . 68
4.8 Equation of Mechanical Energy . . . . . . . . . . . . . . . . . . . . . . . . . . . . . . . 69
4.9 Angular Momentum Balance: r × 2nd Law . . . . . . . . . . . . . . . . . . . . . . . . 70

5 Momentum Transfer and Various Flows 79


5.1 Momentum Transfer by Velocity and Viscosity . . . . . . . . . . . . . . . . . . . . . . 79
5.2 Laminar Flows . . . . . . . . . . . . . . . . . . . . . . . . . . . . . . . . . . . . . . . . 80
5.2.1 Reynolds Number . . . . . . . . . . . . . . . . . . . . . . . . . . . . . . . . . . 81
5.2.2 No Slip Condition . . . . . . . . . . . . . . . . . . . . . . . . . . . . . . . . . . 81
5.2.3 Boundary Layer . . . . . . . . . . . . . . . . . . . . . . . . . . . . . . . . . . . 81
5.2.4 Fully Developed Flow . . . . . . . . . . . . . . . . . . . . . . . . . . . . . . . . 82
5.3 Fluid Flow Due to Pressure Difference . . . . . . . . . . . . . . . . . . . . . . . . . . . 83
5.4 Fluid Flow Due to Gravity . . . . . . . . . . . . . . . . . . . . . . . . . . . . . . . . . 86
5.5 Fluid Flow Due to Moving Plates (Couette Flow) . . . . . . . . . . . . . . . . . . . . . 89
5.6 Superposition Principle of Fluid Flows . . . . . . . . . . . . . . . . . . . . . . . . . . . 90
5.7 Fluid Flow by a Rotating Shaft . . . . . . . . . . . . . . . . . . . . . . . . . . . . . . . 92
5.8 Time-dependent Flow . . . . . . . . . . . . . . . . . . . . . . . . . . . . . . . . . . . . 94
5.9 A Creeping Flow (Re ≪ 1) . . . . . . . . . . . . . . . . . . . . . . . . . . . . . . . . . 96
5.10 Two Dimensional Flows . . . . . . . . . . . . . . . . . . . . . . . . . . . . . . . . . . . 97
5.10.1 Flow in a Tapered Channel (Lubrication Approximation for small-to-moderate
Re) . . . . . . . . . . . . . . . . . . . . . . . . . . . . . . . . . . . . . . . . . . 97
5.10.2 Irrotational Flow . . . . . . . . . . . . . . . . . . . . . . . . . . . . . . . . . . . 100
5.10.3 Creeping Flow . . . . . . . . . . . . . . . . . . . . . . . . . . . . . . . . . . . . 103

6 Application of Momentum Transfer 111


6.1 Correction of the Bernoulli Equation . . . . . . . . . . . . . . . . . . . . . . . . . . . . 111
6.1.1 Bernoulli Equation : Kinetic Energy Correction . . . . . . . . . . . . . . . . . . 112
6.1.2 Bernoulli Equation : Correction for Friction . . . . . . . . . . . . . . . . . . . . 112
6.2 Energy Loss by Skin Friction for Flow in Pipes . . . . . . . . . . . . . . . . . . . . . . 114
6.2.1 Skin Friction . . . . . . . . . . . . . . . . . . . . . . . . . . . . . . . . . . . . . 114
6.2.2 Relation between Pressure Drop and Friction Factor . . . . . . . . . . . . . . . 117
6.2.3 Noncircular Cross-Sections . . . . . . . . . . . . . . . . . . . . . . . . . . . . . 118
6.2.4 Skin Frictional Energy Loss in Terms of Friction Factor . . . . . . . . . . . . . 119
6.3 Energy Loss by Other Frictions . . . . . . . . . . . . . . . . . . . . . . . . . . . . . . . 119
6.3.1 Energy Loss by Form Friction from Changes in Velocity or Direction . . . . . . 119
6.3.2 Energy Loss by Fitting and Valves . . . . . . . . . . . . . . . . . . . . . . . . . 120
6.3.3 Total Frictional Energy Loss in the Bernoulli Equation . . . . . . . . . . . . . . 121
6.4 Bernoulli Equation with a Pump and Friction . . . . . . . . . . . . . . . . . . . . . . . 121
6.5 Buoyancy and Drag Forces . . . . . . . . . . . . . . . . . . . . . . . . . . . . . . . . . 123
6.5.1 Buoyancy Force . . . . . . . . . . . . . . . . . . . . . . . . . . . . . . . . . . . . 125
6.5.2 Drag Force . . . . . . . . . . . . . . . . . . . . . . . . . . . . . . . . . . . . . . 125
6.5.3 Drag in a Flow Past Solid Surfaces . . . . . . . . . . . . . . . . . . . . . . . . . 126
CONTENTS 7

6.5.4 Drag Coefficient (항력계수) . . . . . . . . . . . . . . . . . . . . . . . . . . . . . 127


6.5.5 Terminal Velocity . . . . . . . . . . . . . . . . . . . . . . . . . . . . . . . . . . 131
6.6 Flow in a Packed Bed . . . . . . . . . . . . . . . . . . . . . . . . . . . . . . . . . . . . 134
6.7 Flow in a Fluidized Bed . . . . . . . . . . . . . . . . . . . . . . . . . . . . . . . . . . . 136

A Summary of Differential Vector Operations 145


A.1 Rectangular Cartesian Coordinates . . . . . . . . . . . . . . . . . . . . . . . . . . . . . 145
A.2 Cylindrical Coordinates . . . . . . . . . . . . . . . . . . . . . . . . . . . . . . . . . . . 146
A.3 Spherical Coordinates . . . . . . . . . . . . . . . . . . . . . . . . . . . . . . . . . . . . 146
A.4 Vector Calculation . . . . . . . . . . . . . . . . . . . . . . . . . . . . . . . . . . . . . . 147

B Incompressible and Irrotational Flow 149

C Equation of Continuity and Motion for an Incompressible Flow 151

D Stream Function and Streamline for Incompressible Fluids 155

E Dimensional Analysis 157


E.1 Dimensions . . . . . . . . . . . . . . . . . . . . . . . . . . . . . . . . . . . . . . . . . . 157
E.2 Dimensional Analysis . . . . . . . . . . . . . . . . . . . . . . . . . . . . . . . . . . . . . 157
E.3 The Buckingham Method . . . . . . . . . . . . . . . . . . . . . . . . . . . . . . . . . . 158
E.3.1 Rank of a Matrix . . . . . . . . . . . . . . . . . . . . . . . . . . . . . . . . . . . 159
8 CONTENTS
Chapter 1

Preliminary for Fluid Mechanics

1.1 What is Fluid Mechanics for Chemical Engineers? (Deen’s


book)
Fluid mechanics is a branch of physics concerned with the mechanics of fluids (liquids, gases, and plas-
mas) and the forces on them. Fluid mechanics has a wide range of applications, including mechanical
engineering, civil engineering, chemical engineering, biomedical engineering, geophysics, astrophysics,
and biology. Fluid mechanics can be divided into fluid statics, the study of fluids at rest; and fluid
dynamics, the study of the effect of forces on fluid motion. (Wikipedia)
One thing that distinguishes fluid mechanics in chemical engineering from that in, say, aeronautical
or civil engineering, is the central importance of viscosity. Viscous stresses are at the heart of predicting
flow rates in pipes, which has always been the main application of fluid mechanics in process design.
Moreover, chemical engineering encompasses many technologies that involve bubbles, drops, particles,
porous media, or liquid films, where small length scales amplify the effects of viscosity. Another feature
of chemical engineering fluid mechanics is an emphasis on microscopic analysis to calculate velocity
fields.
Detailed velocity fields are needed to predict concentration and temperature distributions, which
in turn are essential for the analysis and design of reactors and separation devices. Of lesser concern
than in some other disciplines are the fluid dynamics of rotating machinery, flow in open channels,
and flow at near-sonic velocities (where gas compressibility is important). Thus, chemical engineering
fluid mechanics is characterized by a heightened interest in the microscopic analysis of incompressible
viscous flows. Biomedical and mechanical engineers share some of the same concerns.

1.2 What is Transport Phenomena?


The field of Transport Phenomena (이동현상) covers
- momentum transport (Fluid Dynamics) by viscous flow
- energy transport by heat conduction, convention, and radiation
- mass transport by diffusion
occurring in continua (mostly fluid).
and uses basic conservation laws of
- mass (conservation of mass)
- momentum (Newton’s second law = conservation of momentum)
- energy (thermodynamic first law = conservation of energy)
Thus the following properties are used

9
10 CHAPTER 1. PRELIMINARY FOR FLUID MECHANICS

- (scalar) mass, temperature, etc

- (vector) velocity, acceleration, force, etc

- (tensor) stress, etc.

This field was well-established by Dr. Bird’s group in University of Wisconsin and developed more
by Dr. Slattery’s group at Texas A&M University.

Stewart
Bird

Lightfoot

John C. Slattery (Texas A&M Univ)

1.3 Scalars and Vectors


1.3.1 Definition of Scalars and Vectors
Scalar

- a quantity having only magnitude

e.g. temperature, pressure, volume, time, density

Vector

- a quantity having both magnitude and one direction


1.3. SCALARS AND VECTORS 11

- generally denoted by boldface in engineering books, e.g., a


- the magnitude of a vector a is denoted |a|
e.g. velocity, force, acceleration, momentum
Tensor
- a quantity having both magnitude and two or more than two directions
- generally denoted by boldface in engineering books, e.g., T
e.g. stress, strain,

2
(1, 2, 2)

a a
2a
2 y
1
-a

1.3.2 Vector Basis and Components


Vector Basis and Components

(1, 2, 3)

u
ez 3
ey
y
ex
1
2

Figure 1.1: u = ex + 2ey + 3ez

In the 3-dimensional vector space, any vectors can be expressed by the combination of the three
independent vectors ex , ey , ez , which are called Basis Vectors.
u = ux ex + uy ey + uz ez (1.1)
The scalars ux , uy , uz are called the components of ex , ey , ez . The magnitude of u is given by
q
|u| = ux 2 + uy 2 + uz 2 (1.2)
12 CHAPTER 1. PRELIMINARY FOR FLUID MECHANICS

Generally, we choose the basis vectors whose magnitude is 1 (unit vector) and they are perpendicular
each other (orthonormal vector).

|ex | = |ey | = |ez | = 1, ex ⊥ey , ey ⊥ez , ex ⊥ez (1.3)

In addition, the basis vectors in 3-D space could be expressed as (er , eθ , ez ) or (er , eθ , eϕ )

z z ez z
ez er


ex ey uθ
uz er u
uz u ur
u eθ
y ur y y
ux uθ uφ
uy

x x x

u = uxex + uyey + uzez u = urer + uθeθ + uzez u = urer + uθeθ + uφeφ


(a) (b) (c)

Figure 1.2: Basis vectors in (a) Rectangular coordinate system, (b) Cylindrical coordinate system,
and (c) Spherical coordinate system

1.3.3 Dot (Scalar or Inner) Product of Two Vectors


The dot product of two vectors is a scalar defined by

u · v ≡ |u| |v| cos θ (1.4)

where |u| is the magnitude of u and θ is the angle between u and v.


Orthonormal basis vectors:
ex · ex = ey · ey = ez · ez = 1
ex · ey = ey · ez = ex · ez = 0
 (1.5)
δij = +1 if i = j
∴ ei · ej = δij ,
δij = 0 ̸ j
if i =

where δij is called the Kronecker delta.


For u = ux ex + uy ey + uz ez and v = vx ex + vy ey + vz ez , we have

u · v = (ux ex + uy ey + uz ez ) · (vx ex + vy ey + vz ez )
(1.6)
= (ux vx ) + (uy vy ) + (uz vz )

1.3.4 Cross (Vector) Product of Two Vectors


The cross product of two vectors is a vector defined by

u × v ≡ (|u| |v| sin θ) n (1.7)

where n is a unit vector perpendicular to both u and v and pointing in the direction that a right-
handed screw will move if turned from u toward v:

|n| = 1, n⊥u, n⊥v (1.8)


1.3. SCALARS AND VECTORS 13

Figure 1.3: Right-hand rule.

Right-hand rule: u: index finger, v: middle finger, u × v → thumb finger


Orthonormal basis vectors:
ex × ex = ey × ey = ez × ez = 0
ex × ey = ez , ey × ez = ex , ez × ex = ey (1.9)
ey × ex = −ez , ez × ey = −ex , ex × ez = −ey
For u = ux ex + uy ey + uz ez and v = vx ex + vy ey + vz ez , we have

u × v = (ux ex + uy ey + uz ez ) × (vx ex + vy ey + vz ez )
= (uy vz − uz vy )ex + (uz vx − ux vz )ey + (ux vy − uy vx )ez
(1.10)

ex ey ez

= ux uy uz
vx vy vz

1.3.5 Definition of Tensors


Sometimes, it is convenient to have more than one direction. For example, the force at a point exerted
by an external force ∆F depends on both the direction of the external force and the direction of the
surface ∆A, as shown in Fig. 1.4. In order to express the force, it is required to have two directions
in physical properties.

F F

A A

Figure 1.4: A force at a point exerted by an external force depends on both the direction of the
external force and the direction of the surface.

- a natural extension of the definition of vectors (magnitude, directions)


- the number of directions determine the dimension of a tensor n.


 n=0: scalar
X 3 X 3 X3  n=1:

vector
A= ··· Aij · · · t ei ej · · · et , .. (1.11)
i=1 j=1 t=1 | {z }
| {z } 
 .
n n 
n=m: m-th order tensor
| {z } 
n
14 CHAPTER 1. PRELIMINARY FOR FLUID MECHANICS

pressure
viscous force
viscous
force pressure force + viscous force
unit area = stress
(tensor property)

- Generally, the 2nd order tensor is just called a tensor (magnitude, 2 directions). A vector has
magnitude and one direction.

A =Axx ex ex + Axy ex ey + Axz ex ez + Ayx ey ex + Ayy ey ey + Ayz ey ez


(1.12)
+ Azx ez ex + Azy ez ey + Azz ez ez

or in a matrix form,
   
Axx Axy Axz 0 1 0
A = Ayx Ayy Ayz  , ex. A = ex ey = 0 0 0 (1.13)
Azx Azy Azz 0 0 0

and its directions are shown in Figure 1.5.

1.3.6 Properties of Tensors


Identity Tensor

I·u=u (1.14)
In a matrix form and an index notation
 
1 0 0
I = 0 1 0 = ex ex + ey ey + ez ez (1.15)
0 0 1

Symmetry
A tensor A is symmetric if
 
1 2 3
A = AT ⇔ Aij = Aji , ⃝ 4 5 (1.16)
⃝ ⃝ 6

6 independent components

1.4 Vector Differential Operator


The vector differential operator ∇, known as del or nabla, is defined in rectangular coordinates as

∂ ∂ ∂
∇= ex + ey + ez (1.17)
∂x ∂y ∂z

It cannot stand alone and must operate on a scalar, vector, or tensor function, ∇a, ∇ · a. Note that
some of books use different symbols for

x = x1 , y = x2 , z = x3 , ex = e1 = i, ey = e2 = j, ez = e3 = z (1.18)
1.4. VECTOR DIFFERENTIAL OPERATOR 15

z z z

y y y
e xe x e xe y e xe z

x x x

z z z

y y y
e ye x e ye y e ye z

x x x

z z z

y y y
e ze x e ze y e ze z

x x x

Figure 1.5: The unit dyads ei ej . Note that ei ej ̸= ej ei .


16 CHAPTER 1. PRELIMINARY FOR FLUID MECHANICS

1.4.1 Gradient of a Scalar

The gradient of s at a point is a vector pointing in the direction of the maximum rate of change. The
maximum rate of change of s at that point is given by the magnitude of the gradient vector ∇s.

∂s ∂s ∂s
∇s = grad s ≡ ex + ey + ez (1.19)
∂x ∂y ∂z

Ex. 1. If s = x2 + 2xy + y + 2z 2

Answer:

∇s = (2x + 2y)ex + (2x + 1)ey + (4z)ez

1.4.2 Gradient of a Vector

The gradient of a vector can be expressed as

∂v ∂v ∂v
∇v ≡ ex + ey + ez
∂x ∂y ∂z
∂ ∂v ∂v
= (vx ex + vy ey + vz ez )ex + ey + ez
∂x ∂y ∂z
 
∂ ∂ ∂ ∂v ∂v
= (vx ex ) + (vy ey ) + (vz ez ) ex + ey + ez
∂x ∂x ∂x ∂y ∂z
 
∂vx ∂vy ∂vz ∂v ∂v
= ex + ey + ez ex + ey + ez
∂x ∂x ∂x ∂y ∂z (1.20)
∂vx ∂vy ∂vz ∂v ∂v
= ex ex + ey ex + ez ex + ey + ez
∂x ∂x ∂x ∂y ∂z
∂vx ∂vx ∂vx
 
 ∂x ∂y ∂z 
 ∂v
 y ∂vy ∂vy 

=
 ∂x ∂y ∂z 

 ∂vz ∂vz ∂vz 
∂x ∂y ∂z

Ex. 2. If v = xex + 2xyzey + (y + z 2 )ez

∂vx ∂vx ∂vx ∂vy ∂vy ∂vy


∇v = ex ex + ex ey + ex ez + ey ex + ey ey + ey ez
∂x ∂y ∂z ∂x ∂y ∂z
∂vz ∂vz ∂vz
+ ez ex + ez ey + ez ez
∂x ∂y ∂z
= ex ex + 2yzey ex + 2xzey ey + 2xyey ez + ez ey + 2zez ez
 
1 0 0
= 2yz 2xz 2xy 
0 1 2z
1.4. VECTOR DIFFERENTIAL OPERATOR 17

1.4.3 Divergence of a Vector


The scalar or dot product of a vector and the operator ∇ is called the divergence of the vector field

∂v ∂v ∂v
∇ · v = div v ≡ · ex + · ey + · ez
∂x ∂y ∂z
∂ ∂v ∂v
= (vx ex + vy ey + vz ez ) · ex + · ey + · ez
∂x ∂y ∂z
 
∂ ∂ ∂ ∂v ∂v
= (vx ex ) + (vy ey ) + (vz ez ) · ex + · ey + · ez
∂x ∂x ∂x ∂y ∂z
 
∂vx ∂vy ∂vz ∂v ∂v
= ex + ey + ez · ex + · ey + · ez (1.21)
∂x ∂x ∂x ∂y ∂z
∂vx ∂vy ∂vz ∂v ∂v
= ex · ex + ey · ex + ez · ex + · ey + · ez
∂x ∂x ∂x ∂y ∂z
∂vx ∂v ∂v
= + · ey + · ez
∂x ∂y ∂z
∂vx ∂vy ∂vz
= + +
∂x ∂y ∂z

Ex. 3. If v = xex + 2xyzey + (y + z 2 )ez

∂vx ∂vy ∂vz


∇·v = + + = 1 + 2xz + 2z
∂x ∂y ∂z

1.4.4 Divergence of a Tensor


The dot product of a tensor and the operator ∇ is called the divergence of the tensor field

∂T ∂T ∂T
∇ · T = div T ≡ · ex + · ey + · ez
∂x ∂y ∂z
∂ ∂T ∂T
= (Txx ex ex + Txy ex ey + · · · + Tzz ez ez ) · ex + · ey + · ez
∂x ∂y ∂z
∂Txx ∂Txy ∂Tzz ∂T ∂T
= ex ex · ex + ex ey · ex + · · · + ez ez · ex + · ey + · ez
∂x ∂x ∂x ∂y ∂z
∂Txx ∂Tyx ∂Tzx ∂T ∂T
= ex + ey + ez + · ey + · ez
∂x ∂x ∂x ∂y ∂z
     
∂Txx ∂Txy ∂Txz ∂Tyx ∂Tyy ∂Tyz ∂Tzx ∂Tzy ∂Tzz
= + + ex + + + ey + + + ez
∂x ∂y ∂z ∂x ∂y ∂z ∂x ∂y ∂z
(1.22)

 
x 2xyz 1
Ex. 4. If T = 2xyz y y
1 y z

     
∂Txx ∂Txy ∂Txz ∂Tyx ∂Tyy ∂Tyz ∂Tzx ∂Tzy ∂Tzz
∇·T= + + ex + + + ey + + + ez
∂x ∂y ∂z ∂x ∂y ∂z ∂x ∂y ∂z
= (1 + 2xz) ex + (2yz + 1) ey + 2ez
18 CHAPTER 1. PRELIMINARY FOR FLUID MECHANICS

1.4.5 Curl of a Vector


The cross product of a vector and the operator ∇ is called the curl of the vector field

ex ey ez

∇ × v = curl v ≡ ∂/∂x ∂/∂y ∂/∂z
vx vy vz (1.23)
     
∂vz ∂vy ∂vx ∂vz ∂vy ∂vx
= − ex + − ey + − ez
∂y ∂z ∂z ∂x ∂x ∂y

Unlikely to the gradient and divergence, the curl of v is calculated by


 
∂v ∂v ∂v
∇× v=− × ex + × ey + × ez (1.24)
∂x ∂y ∂z

It should be noticed that the minus sign is required when the curl of a vector is calculated by the
above method.

Ex. 5. If v = xex + 2xyzey + (y + z 2 )ez

Answer:

∇ × v = (1 − 2xy) ex + (2yz) ez

1.4.6 Laplacian of a Scalar


The divergence of a gradient of a scalar is called the Laplacian of a scalar field
 
2 ∂ ∂s ∂s ∂s
∆ s ≡ ∇ · ∇s = ∇ s = ex + ey + ez · ex
∂x ∂x ∂y ∂z
   
∂ ∂s ∂s ∂s ∂ ∂s ∂s ∂s
+ ex + ey + ez · ey + ex + ey + ez · ez (1.25)
∂y ∂x ∂y ∂z ∂z ∂x ∂y ∂z
∂2s ∂2s ∂2s
= + +
∂x2 ∂y 2 ∂z 2
where ∆ is called the Laplacian operator.

∂2 ∂2 ∂2
∴ ∆ ≡ ∇2 = 2 + 2 + (1.26)
∂x ∂y ∂z 2

Ex. 6. If s = x2 + 2xy + y + 2z 2

Answer:
∂2s ∂2s ∂2s
∆s = + + =6
∂x2 ∂y 2 ∂z 2

1.5 Vector Differential Operators in Other Coordinates (App.


A,B)
1.5.1 Rectangular Cartesian Coordinates
The location of a point in three dimensional space may be specified by an ordered set of numbers
(x, y, z) in the rage of

− ∞ ≤ x ≤ +∞, −∞ ≤ y ≤ +∞, −∞ ≤ z ≤ +∞, (1.27)


1.5. VECTOR DIFFERENTIAL OPERATORS IN OTHER COORDINATES (APP. A,B) 19

z z z
ez er
ez eθ eφ
ey θ
z er
r eθ
ex y y
y r
θ φ

x x x

(a) (b) (c)

1/2
x = r cos θ r = (x 2 + y 2 ) r 2 = x2 + y 2
{ y = r sin θ
z=z
{ θ = arctan (y/x)
z=z
or
{ tan θ = y/x
z=z

ex = cos θ e r - sin θ e θ er = cos θ e x + sin θ e y


or
ey = sin θ e r + cos θ e θ eθ = -sin θ e x + cos θ e y

1/2
x = r sin θ cos φ r = (x 2 + y 2 +z 2 )
{ y = r sin θ sin φ
z = r cos θ
{ 1/2
θ = arctan ((x2 + y2 ) /z)
φ = arctan (y/x)

ex = (sin θ cos φ) er + (cos θ cos φ) eθ + (-sin φ) e φ


ey = (sin θ sin φ) er + (cos θ sin φ) eθ + (cos φ) e φ
ez = (cos θ) er + (-sin θ) eθ

Figure 1.6: (a) Rectangular coordinate system, (b) Cylindrical coordinate system, and (c) Spherical
coordinate system
20 CHAPTER 1. PRELIMINARY FOR FLUID MECHANICS

and we have
∂ ∂ ∂
∇= ex + ey + ez
∂x ∂y ∂z
(1.28)
∂2 ∂2 ∂2
∆ = ∇2 = 2 + 2 +
∂x ∂y ∂z 2
For s = s(x, y, z) and v = vx ex + vy ey + vz ez , we have

∂s ∂s ∂s
∇s = ex + ey + ez
∂x ∂y ∂z
∂v ∂v ∂v ∂vx ∂vy ∂vz
∇·v = · ex + · ey + · ez = + +
∂x ∂y ∂z ∂x ∂y ∂z
2 2 2
∂ v ∂ v ∂ v
∇2 v = + + 2
∂x2 ∂y 2 ∂z
 2 2
∂ 2 vx
 2
∂ 2 vy ∂ 2 vy
 2
∂ 2 vz ∂ 2 vz
  
∂ vx ∂ vx ∂ vy ∂ vz
= + + ex + + + ey + + + ez
∂x2 ∂y 2 ∂z 2 ∂x2 ∂y 2 ∂z 2 ∂x2 ∂y 2 ∂z 2
(1.29)

1.5.2 Cylindrical Coordinates


The location of a point in three dimensional space may be specified by an ordered set of numbers
(r, θ, z) in the rage of

0 ≤ r ≤ +∞, 0 ≤ θ ≤ 2π, −∞ ≤ z ≤ +∞, (1.30)

and we can express ∇ and ∆ in terms of the cylindrical coordinate system, (r, θ, z) and (er , eθ , ez ).

(x, y, z) → (r, θ, z) and (ex , ey , ez ) → (er , eθ , ez )

The relations between the Rectangular Cartesian and Cylindrical Coordinates are
 p  2 2
2
 r = x +y 
2
 r =x + y2

 x = r cos θ  
y
y
y = r sin θ or θ = arctan ⇔ tan θ = (1.31)
x x
z=z
  
z=z z =z
 

Let’s express the first terms in equation (1.29) in terms of (r, θ, z) and (er , eθ , ez ). For here s =
s(r, θ, z) where r(x, y, z), θ(x, y, z), z(x, y, z)
 
∂s sin θ ∂s
sx = sr rx + sθ θx + sz zx = cos θ + − (1.32)
∂r r ∂θ

from the chain rule1 :



∂s ∂s ∂r ∂s ∂θ ∂s ∂z
= + + (1.33)
∂x y,z ∂r θ,z ∂x y,z ∂θ r,z ∂x y,z ∂z r,θ ∂x y,z

and
∂r ∂θ sin θ ∂z
= cos θ, =− , =0 (1.34)
∂x y,z ∂x y,z
r ∂x y,z
In addition,
ex = cos θ er − sin θ eθ + (0) ez (1.35)
1 For example, s(u, v, w) = 2u + v + 3w where u = x + y, v = 2y + x, w = z

∂s substitution, s = 2(x + y) = (2y + x) + 3z = 3x + 4y + 3z, sx = 3
sx = =
∂x y,z chain rule, sx = su ux + sv vx + sw wx = 3
1.5. VECTOR DIFFERENTIAL OPERATORS IN OTHER COORDINATES (APP. A,B) 21

y
ey

θ er
θ
ex
p (x, y, z) = p (r, θ, z)
θ
x

Figure 1.7: Cylindrical coordinate system

Thus
sin2 θ ∂s
   
∂s ∂s sin θ cos θ ∂s ∂s
ex = cos2 θ − er + − sin θ cos θ + eθ (1.36)
∂x ∂r r ∂θ ∂r r ∂θ
Finally in the cylindrical coordinates we have
∂s 1 ∂s ∂s
∇s = er + eθ + ez (1.37)
∂r r ∂θ ∂z
Therefore we can say in this cylindrical coordinate system,
∂ 1 ∂ ∂
∇= er + eθ + ez (1.38)
∂r r ∂θ ∂z
Using this, we can express ∇ · v in terms of the cylindrical coordinates as follows:
∂v 1 ∂v ∂v
∇·v = · er + · eθ + · ez (1.39)
∂r r ∂θ ∂z
where v = v(r, θ, z) Since v = vr er + vθ eθ + vz ez , we have
∂v ∂ ∂vr ∂vθ ∂vz
= (vr er + vθ eθ + vz ez ) = er + eθ + ez
∂r ∂r ∂r ∂r ∂r
∂v ∂ ∂vr ∂er ∂vθ ∂eθ ∂vz
= (vr er + vθ eθ + vz ez ) = e r + vr + eθ + vθ + ez
∂θ ∂θ ∂θ ∂θ ∂θ ∂θ ∂θ (1.40)
∂vr ∂vθ ∂vz
= er + vr eθ + eθ − vθ er + ez
∂θ ∂θ ∂θ
∂v ∂ ∂vr ∂vθ ∂vz
= (vr er + vθ eθ + vz ez ) = er + eθ + ez
∂z ∂z ∂z ∂z ∂z
and finally
1 ∂ 1 ∂vθ ∂vz
∇·v = (rvr ) + + (1.41)
r ∂r r ∂θ ∂z
Similarly

∂2 ∂2 ∂2
     
2 ∂ ∂ ∂ ∂ ∂ ∂
∆=∇ = 2 + 2 + 2 = + +
∂x ∂y ∂z ∂x ∂x ∂y ∂y ∂z ∂z
       
∂ ∂ ∂r ∂ ∂ ∂θ ∂ ∂ ∂z
= + +
∂r ∂x ∂x ∂θ ∂x ∂x ∂z ∂x ∂x
       
∂ ∂ ∂r ∂ ∂ ∂θ ∂ ∂ ∂z
+ + + (1.42)
∂r ∂y ∂y ∂θ ∂y ∂y ∂z ∂y ∂y
       
∂ ∂ ∂r ∂ ∂ ∂θ ∂ ∂ ∂z
+ + +
∂r ∂z ∂z ∂θ ∂z ∂z ∂z ∂z ∂z
1 ∂2 ∂2
 
1 ∂ ∂
= r + 2 2+ 2
r ∂r ∂r r ∂θ ∂z
22 CHAPTER 1. PRELIMINARY FOR FLUID MECHANICS

and thus
1 ∂ 2 vr ∂ 2 vr
  
2 ∂ 1 ∂ 2 ∂vθ
∇ v= (rvr ) + 2 − 2 + er
∂r r ∂r r ∂θ2 r ∂θ ∂z 2
1 ∂ 2 vθ ∂ 2 vθ
   
∂ 1 ∂ 2 ∂vr
+ (rvθ ) + 2 + 2 + eθ (1.43)
∂r r ∂r r ∂θ2 r ∂θ ∂z 2
1 ∂ 2 vz ∂ 2 vz
   
1 ∂ ∂vz
+ r + 2 + ez
r ∂r ∂r r ∂θ2 ∂z 2

1.5.3 Spherical Coordinates


The location of a point in three dimensional space may be specified by an ordered set of numbers
(r, θ, ϕ) in the rage of

0 ≤ r ≤ +∞, 0 ≤ θ ≤ π, 0 ≤ ϕ ≤ 2π (1.44)

and we can express ∇ and ∆ in terms of the spherical coordinate system, (r, θ, ϕ) and (er , eθ , eϕ ).

(x, y, z) → (r, θ, ϕ) and (ex , ey , ez ) → (er , eθ , eϕ )

The relations between the Rectangular Cartesian and Spherical Coordinates are
 p
 
 r = x2 + y 2 + z 2 !
 x = r sin θ cos ϕ
 p
x2 + y 2


y = r sin θ sin ϕ or θ = arctan (1.45)
z
z = r cos θ
 
 y

ϕ = arctan


x
and thus
ex = (sin θ cos ϕ) er + (cos θ cos ϕ) eθ + (− sin ϕ) eϕ
ey = (sin θ sin ϕ) er + (cos θ sin ϕ) eθ + (cos ϕ) eϕ (1.46)
ez = (cos θ) er + (− sin θ) eθ + (0) eϕ

∂ 1 ∂ 1 ∂
∇= er + eθ + eϕ
∂r r ∂θ r sin θ ∂ϕ
(1.47)
∂2
   
2 1 ∂ 2 ∂ 1 ∂ ∂ 1
∆=∇ = 2 r + 2 sin θ + 2 2
r ∂r ∂r r sin θ ∂θ ∂θ r sin θ ∂ϕ2
In this coordinate systems,
∂s 1 ∂s 1 ∂s
∇s = er + eθ + eϕ
∂r r ∂θ r sin θ ∂ϕ
1 ∂ 1 ∂ 1 ∂vϕ
r 2 vr +

∇·v = 2 (vθ sin θ) +
r ∂r r sin θ ∂θ r sin θ ∂ϕ
2
∂ 2 vr
   
2 1 ∂ 2
 1 ∂ ∂v r 1
∇ v = 2 2 r vr + 2 sin θ + 2 2 er
r ∂r r sin θ ∂θ ∂θ r sin θ ∂ϕ2
∂ 2 vθ
     
1 ∂ 2 ∂vθ 1 ∂ 1 ∂ 1 2 ∂vr 2 cot θ ∂vϕ
+ 2 r + 2 (vθ sin θ) + 2 2 + 2 − 2 eθ
r ∂r ∂r r ∂θ sin θ ∂θ r sin θ ∂ϕ2 r ∂θ r sin θ ∂ϕ
∂ 2 vϕ
     
1 ∂ ∂vϕ 1 ∂ 1 ∂ 1 2 ∂vr 2 cot θ ∂vθ
+ 2 r2 + 2 (vϕ sin θ) + 2 2 + + eϕ
r ∂r ∂r r ∂θ sin θ ∂θ r sin θ ∂ϕ2 r2 sin θ ∂ϕ r2 sin θ ∂ϕ
(1.48)
Chapter 2

Basic Concepts of Fluid Mechanics

2.1 Fluids and Mechanics


2.1.1 Fluids and Solids
What is a Fluid?

- moves and flows

- gas and liquid (opposite to a Solid)

- changes its shape (mechanically, deforms continuously when subjected to shear forces)

- more mechanically, its shear stress is proportional to its rate of shear strain. On the contrary,
the shear stress of a solid is proportional to the shear strain of the solid.

2.1.2 Mechanics
What is Mechanics?

Equilibrium

Force Matter
w/wo
Deformation Motion

- the study of the equilibrium (statics) or motion (dynamics) of matter by forces

- statics + dynamics

- expressed in terms of scalar, vector, and tensor such as force (vector property), deformation
(tensor property)

- by the size of matter, macro-, micro-, nano-, molecular-, quantum-mechanics

Molecular &
Macro Micro Nano
Quantum
-mechanics -mechanics -mechanics
-mechanics
10 0 10 -3 10 -6 10 -9 10 -12

23
24 CHAPTER 2. BASIC CONCEPTS OF FLUID MECHANICS

Control surface
Material
Body (or system)
particles

Moti
on (t
)
Control volume
Boundary

Solid Fluid

Figure 2.1: Material particles, body, control volume, control surface, and motion

2.2 Motion of a Fluid


Material particle (point): a small volume of material as a basic element of material
Body (or System): a large collection of material particles
Boundary: a two-dimensional dividing surface surrounding a body
Motion: movement of a body
• Uniform translation: The velocity is the same everywhere. No deformation occurs. (Any
two points in square remains the same distance after the motion)
• Rigid-body rotation: No deformation occurs even though the linear velocity varies with
position. It is related to vorticity.
• Pure dilatation: Shape-preserving motion with deformation.
• Simple shear1 : Volume (density) preserving motion with deformation.

at t=t0
y y y y
at t=t1

x x x x
Uniform translation Rigid-body rotation Dilatation - Simple shear -
without deformation without deformation shape-preserving volume-preserving

Figure 2.2: Two-dimensional motion of fluids; uniform translation, rigid-body motion, pure dilatation,
simple shear motions.

Strain: In solid mechanics a deformation is generally expressed as a strain, which is a ratio of


lengths. For a sample of initial length L that is elongated by an amount ∆L, the strain is ∆L/L.
1 Shear is defined as relative motion between adjacent layers of a moving liquid. Some of the simplest examples

include spreading butter on bread or applying sunscreen. In each case one level of the liquid (butter on the knife) is
moving relative to the adjacent layer (butter on the bread). Shear rate is defined as the measure of the extent or rate
of relative motion between adjacent layers of a moving liquid = Velocity / Distance. Using the same example, we can
raise the shear rate by either raising the velocity of the knife or by placing the knife closer to the bread.
2.3. TIME DERIVATIVES: PARTIAL, TOTAL, MATERIAL (SUBSTANTIAL) 25

Rate of strain: (∆L/L)/∆t as ∆t → 0. Because it is the rate of deformation that determines


viscous stresses, in fluid mechanics we are concerned with a rate of strain, which is defined as
L−1 dL/dt and has the dimension of inverse time.
Control volume: is a Fixed region in space through which fluid flows and is mostly used in
fluid dynamics.
Control surface: is a Fixed two-dimensional dividing surface surrounding the control volume.

2.3 Time Derivatives: Partial, Total, Material (Substantial)


In this section, three different time derivatives encountered in transport phenomena (Fig. 2.3) will be
pointed out. Before we discuss, the definition of derivatives is explained.
• Partial differentiation (편미분) and partial derivative (편도함수)
For a two-variables function f (x, y), consider the change in f when x is infinitesimally changed
at a fixed y. It is called partial differentiation, ∂f . Mathematically,
f (x + ∆x, y) − f (x, y) ∂f
∂f = lim f (x + ∆x, y) − f (x, y) = lim ∆x = dx (2.1)
∆x→0 ∆x→0 ∆x ∂x
Here
∂f f (x + ∆x, y) − f (x, y)
= lim (2.2)
∂x ∆x→0 ∆x
is called a partial derivative
• Total differentiation (전미분) and total derivative (전도함수)
What about the total change in f (x, y) when both x and y are simultaneously changed. It is
called total differentiation, df . Mathematically,
f (x + ∆x, y + ∆y) − f (x, y + ∆y)
df = lim f (x + ∆x, y + ∆y) − f (x, y) = lim ∆x
∆x→0
∆y→0
∆x→0
∆y→0
∆x
(2.3)
f (x, y + ∆y) − f (x, y) ∂f ∂f
+ ∆y = dx + dy
∆y ∂x ∂y
That is, the total change in f is the sum of the change in f according to an infinitesimal change
in x and the change in f according to an infinitesimal change in y.
If the independent variables x, y for the function f are dependent variables of other variables,
the x and y variables lose their independency.
1. If y = y(x), y is not an independent variable anymore, f (x, g(x)). Now let’s calculate
the derivative of f over x. One way is to have h(x) = f (x, g(x)) and get its derivative,
dh(x)/dx,2 but it may be sometimes complicated. The other is to use the total differentia-
tion (2.3) and it is called a total derivative, which is calculated using the total differentiation:
df ∂f ∂f dy ∂f ∂f dg
= + = + (2.4)
dx ∂x ∂y dx ∂x ∂y dx
The total derivates is divided by two parts: one part affected by direct change in x and the
other part affected indirectly through the change y caused by the change in x.
2. If x = g(t) and y = h(t), x and y are not independent variables, f (g(t), h(t)). The total
derivative with respect to t is
df ∂f dx ∂f dy ∂f dg ∂f dh
= + = + (2.5)
dt ∂x dt ∂y dt ∂x dt ∂y dt
2 Sometimes, it is called an ordinary derivative, which is a derivative applied only to a single-variable function, like

h(x) here.
26 CHAPTER 2. BASIC CONCEPTS OF FLUID MECHANICS

Let c is the concentration of fish in a river. Because fish swim around, the fish concentration c will
be in general a function of position (x, y, z) and time t, c(x, y, z, t) and the total differentiation in c
(t와 x, y, z가 동시에 변화할 때 c의 총변화량) is
       
∂c ∂c ∂c ∂c
dc = dt + dx + dy + dz (2.6)
∂t x,y,z ∂x y,z,t ∂y x,z,t ∂z x,y,t
| {z } | {z } | {z } | {z }
the change in c the change in c the change in c the change in c
due to an infinitesimal due to an infinitesimal due to an infinitesimal due to an infinitesimal
change in t only change in x only change in y only change in z only

That is, the total change in c is the sum of the changes in c according to infinitesimal changes in t, x,
y, and z, respectively.
c(t, x, y, z) : the concentration of fish in a location

v
u
v
u

c(t, xfixed, yfixed, zfixed)


c(t, x(t), y(t), z(t)) c(t, x(t), y(t), z(t))
x′=ux, y′=uy, z′=uz x′=vx, y′=vy, z′=vz

z L (fixed) L(x(t), y(t), z(t))

Figure 2.3: Time derivatives: partial, total, and material derivatives.

Partial Time Derivative, ∂c/∂t (at a fixed location)


Suppose us in a boat that is anchored securely in a river, some distance from the shore. Now we
observe the concentration of fish just below us as a function of time and record the time rate of
change of c at a fixed location (고정된 위치에서 있는 관찰차가 관측한 시간에 대한 변화율). It is called
a partial time derivative: ∂c/∂t = dc
dt |x,y,z .

Total Time Derivative, dc/dt (boat velocity)


We pull up our anchor, switch on our outboard motor, and race about the river, sometimes upstream,
sometime downstream, or across the river current. Then the coordinates are functions of time: x =
f (t), y = g(t), z = h(t). The total time derivative is obtained directly from the total differentiation
(2.6). (임의적인 속도 u로 움직이는 관찰자가 관측한 시간의 대한 변화율):
       
dc ∂c ∂c dx ∂c dy ∂c dz
= + + +
dt ∂t x,y,z ∂x y,z,t dt ∂y x,z,t dt ∂z x,y,t dt
∂c
=
∂t
+ | ·{z∇c}
u (2.7)
net rate of change of
|{z}
local rate of c due to an observer’s (boat) motion
change of c = ċout − ċin
2.3. TIME DERIVATIVES: PARTIAL, TOTAL, MATERIAL (SUBSTANTIAL) 27

where u denotes the velocity of the observer which is the same as the boat. The x, y, and z components
of the velocity correspond to ux = dx/dt, uy = dy/dt, and uz = dz/dt, respectively. In fact, ∇c · u/|u|
is a directional derivative of c in the direction of s = u/|u|, which is the rate of change of c in the
direction given by a unit vector dc/ds. Here s is a magnitude in the direction of the unit vector
s = u/|u|.

Material(Substantial) Time Derivative, Dc/Dt (fluid velocity)


Suppose we now let our boat drift along with the river current. In this situation the velocity of the
observer is the same as the velocity v of the current (the fluid material). The time rate of change of
c at a moving location becomes (유체의 속도에 따라 움직이는 관찰자가 관측한 시간에 대한 변화율):

dc Dc ∂c
dt v =
Dt
=
∂t
+ | ·{z∇c}
v
|{z} net rate of change of (2.8)
local rate of c due to an observer’s (fluid) motion
change of c

and it is called a material(or substantial) time derivative (물질 시간도함수).

2.3.1 Relationship between total and partial time derivatives


Referring to Fig. 2.4, we can see that
• Region I is occupied by a square only at time t.
• Region II is occupied by the square only at time t + ∆t.
• Region III is common to the square at time t and t + ∆t.

Boundary of Boundary of the body


II
a body at time t 4 at time t + Δt

1
2
1 3 5

3 fish at time t + Δt
4
fish at III
time t 2
I 5

Figure 2.4: Fish at time t and t + ∆t in a square. The body moves in the velocity u due to the boat
velocity u.

The time rate of change of the number of fish in the square is


dc d c|t+∆t − c|t
= c(x(t), y(t), z(t), t) = lim
dt dt ∆t→0 ∆t
(cII + cIII )t+∆t − (cI + cIII )t
= lim (2.9)
∆t→0 ∆t
cIII |t+∆t − cIII |t cII |t+∆t − cI |t
= lim + lim
∆t→0 ∆t ∆t→0 ∆t
The first term indicates the rate of change of the number of fish in the control volume since
the region III becomes the control volume as ∆t → 0:
cIII |t+∆t − cIII |t dcIII ∂c
lim = = (2.10)
∆t→0 ∆t dt ∂t
28 CHAPTER 2. BASIC CONCEPTS OF FLUID MECHANICS

The next limiting expression is the net rate of fish efflux across the control surface during
the time interval ∆t since region I and II correspond to the control surface as ∆t → 0.

cII |t+∆t − cI |t
lim = ċout − ċin (2.11)
∆t→0 ∆t
Therefore, we can see
u · ∇c = ċout − ċin (2.12)
and
dc ∂c ∂c
= + u · ∇c = + ċout − ċin (2.13)
dt ∂t ∂t

2.3.2 Velocity and Acceleration


Velocity
the time derivative of position:
 
dL dx dy dz
v≡ , vx = , vy = , vz = (2.14)
dt dt dt dt

Acceleration
the time derivative of velocity:
dv
a≡ (2.15)
dt

2.3.3 Steady and Unsteady Flows


In the Eulerian representation, the fluid flow is a function of four independent variables (x, y, z, t). If
the flow at every point in the fluid is independent of time, the flow is termed steady.

∂c
=0 (2.16)
∂t
If the flow at a point varies with time, the flow termed unsteady.

2.4 Fundamental Physics Laws


The conservation laws such as mass, momentum, energy, and electric charge have been widely used in
many branches of engineering, as shown in Fig. 2.5. They provide global descriptions of large systems
without much regard for the details of the fluid dynamics inside the system. Often they are useful for
making an initial appraisal of an engineering problem and for making order-of-magnitude estimates
of various quantities.

2.4.1 Conservation of Mass (질량보존의 법칙)


The mass of a closed system remains constant, i.e., matter cannot be created/destroyed, although it
can be rearranged.3
dm
=0 (2.17)
dt
3 Wikipedia: The conservation of mass only holds approximately and is considered part of a series of assumptions

coming from classical mechanics. The law has to be modified to comply with the laws of quantum mechanics and
special relativity under the principle of mass-energy equivalence, which states that energy and mass form one conserved
quantity. For very energetic systems the conservation of mass-only is shown not to hold, as is the case in nuclear
reactions and particle-antiparticle annihilation in particle physics.
2.4. FUNDAMENTAL PHYSICS LAWS 29

Figure 2.5: Conservation of Mass, Momentum, and Energy (Bird et al., Fig. 7.0-1). Macroscopic flow
system with fluid entering at plane 1 and leaving at plane 2.

2.4.2 Newton’s Laws of Motion


• 운동량(momentum) = 질량 × 속도
• 힘(force) = 질량× 가속도 = 질량 × ∆속도/∆시간 = ∆운동량/∆ 시간
=⇒ 운동량 변화량(=충격량) = 힘 × 시간변화량

물체에 힘을 가하면 물체가 가속되면서 속도 및 운동량이 증가하는데 운동량은 힘× 시간 만큼


증가한다.

F m v

Figure 2.6: 질량 m인 물체에 힘(F)을 가하면 물체가 속도(v)가 변한다.

The First Law: Law of Inertia


Inertia: the resistance to change in state of motion of a body
• Linear inertia = mass(m): linear motion
For a given force F, the (linear) acceleration must be proportional to the inverse of the linear
inertia:
dv 1
a= ∝ (2.18)
dt m
As m(the resistance to change an object’s linear motion) increases, a ( the change in velocity)
decreases. Linear momentum is defined as P = mv
• Rotational inertia = moment of inertia (I): rotational motion
For a given rotational force (Torque, τ ), the angular acceleration must be proportional to the
inverse of the rotational inertia:
dω 1
α= ∝ (2.19)
dt I
30 CHAPTER 2. BASIC CONCEPTS OF FLUID MECHANICS

Angular momentum is defined as H = Iω.


1. An object at rest tends to stay at rest and an object in motion tends to stay in motion with the
same speed and in the same direction unless acted upon by a net external force.
2. If net external force on a body is zero, then its velocity remains constant or its acceleration
remains zero (v= a constant).
X
F = 0 ⇐⇒ a = 0 by 2nd law ⇐⇒ v = a constant (2.20)

The Second Law: Law of Acceleration, Conservation of Linear Momentum (운동량 보존법
칙)
1. Force acting on a body(고체 뿐만 아니라 유체도 포함) is the cause of its moving.
2. Force changes the direction of motion or the velocity of a body. It is possible for a body to keep
moving without force, but not possible to change the state of the motion.
3. Net force is equal to acceleration produced in unit mass
X
F = ma (2.21)

4. The net force acting on the body is also equal to the time rate of change of linear momentum
of a body.
X dv
F = ma = m
dt (2.22)
d(mv) dP
= = ∵ m=constant by conservation of mass
dt dt
where the linear momentum, P = mv, indicates the tendency of an object moving in a certain
direction to keep going at the same speed in the same direction. See Fig. 2.8.

Figure 2.7: 운동량보존. x 방향의 반대로 힘이 주어져서 노즐에서 나온 물의 x 방향의 운동량이 감소하
였고, y 방향으로 힘이 주어지지 않았기 때문에 전체 y 방향의 운동량은 0을 유지함.

5. For a given mass or force


1
a ∝ F, a∝ (2.23)
m

The Third Law (Law of Reciprocal Actions)


1. For every action, there is an equal in size and opposite reaction in direction.
2. All forces are interactions - that there is no such thing as a unidirectional force. If body A exerts
a force on body B, simultaneously, body B exerts a force of the same magnitude body A.
2.4. FUNDAMENTAL PHYSICS LAWS 31

2 m/s

500 kg 500 kg

5000 N 6000 N
5000 N 5000 N
Scale (5000N) Scale (6000 N)

Figure 2.8: 작용 반작용에 의하여 전자저울이 무게를 미는 힘은 무게가 전자저울을 미는 힘과 같다. 아


인슈타인 일반 상대성이론에 따르면 가속운동에 의하여 아래로 향한 쏠림과 지구가 잡아당기는 중력은
구별할 수 없다.

2.4.3 Thermodynamic Laws


The First Law (Conservation of Energy, 에너지보존법칙)

Etot W

F m, E v

Figure 2.9: Conservation laws of mass, momentum, and energy

1. In any process, the total energy of the universe (system + surrounding) remains the same.

dEtot
=0 (2.24)
dt

2. The time rate of change of energy of a body (closed system) is equal to the rate of energy
transmission to the body from its surroundings plus the rate of work done on the body by its
surrounding.
dE
= Q̇ − Ẇ (2.25)
dt

The Second Law


The entropy of an isolated system not in equilibrium will tend to increase over time, approaching a
maximum value at equilibrium.
dS
≥0 (2.26)
dt

The Third Law


As temperature approaches absolute zero, the entropy of a system approaches a constant minimum

Summary
32 CHAPTER 2. BASIC CONCEPTS OF FLUID MECHANICS

유체 및 역학

유체 (流體, Fluid) 역학 (力學, Mechanics)

움직인다 & 흐른다 힘에 의한 물체의 평형이나 운동을


배우는 학문

기체 & 액체
평형


고체의 반대이다 변형
운동

손으로 눌러서 모양이 변한다. 동역학(dynamics) + 정역학(statics)

응력 µ 변형율
Scalar, Vector, Tensor
(고체, 응력 µ 변형)

화학공학부 울산대학교
SCHOOL OF CHEMICAL ENGINEERING UNIVERSITY OF ULSAN

뉴튼의 운동법칙
!" 질량 ! !
∑ " = $% = $ = $& = ' , '(운동량) = $&
!# 보존 !# !#

• 외부에서 힘이 주어지지 않는 한 운동량의 변화는 없다. (운동량 보존 법칙)

• 외부에서 주어진 힘은 운동량의 변화량과 동일하다

• 노즐에서 나온 물의 운동량(속
도가 12m/s에서 0으로 변함)을
변화시키기 위해서는 힘이 필
요함.
• 여기서 힘의 방향은 – 방향임

화학공학부 울산대학교
SCHOOL OF CHEMICAL ENGINEERING UNIVERSITY OF ULSAN
Chapter 3

Conservation of Mass

The conservation of mass can be expressed within either a body or a control volume.

1. (Solid mechanics) The time rate of change of mass in a body or a system is zero.

dm
dt
motion (v)

body (t=t0) body (t=t1)

dm
=0 (3.1)
dt

2. (Fluid mechanics) The rate of accumulation(=time rate of change) of mass in a


control volume is equal to the net rate of mass flow into the control volume.

min
дm
дt mout
CS CV

∂m
= ṁin − ṁout (3.2)
∂t
|{z}
|{z} | {z }
mass flow rate mass flow rate
rate of mass accumulation
into control volume out of control volume
within control volume

Here m is the mass in the control volume which is a fixed region in space as mentioned in Sect.
2.2. Mass can be expressed as

m = density × volume = ρV (3.3)

When the density is not constant in the control volume, mas can be obtained from
Z
m = ρ dV (3.4)

Mass flow rate through a inclined surface


Mass flow rate entering into or exiting from the control volume through surface A can be
expressed as

ṁ = density × velocity × area = ρAv⊥ (3.5)

33
34 CHAPTER 3. CONSERVATION OF MASS

where v⊥ is the velocity component perpendicular to surface A. Generally, ρv⊥ is called mass
flux defined as the amount of mass flowing through a unit cross-sectional area per unit time
[g/cm2 ·s] and Av⊥ is called the volume flow rate.
When the velocity v⊥ is not constant on the surface A, the mass flow rate becomes
Z
ṁ = ρv⊥ dA (3.6)

n1
n2
v1y v2y
y
ρ1 v1 A1 v2 ρ2
A2
mass per v1x v2x
x
unit volume

(a) mass flow in (b) mass flow out

Figure 3.1: Fluid flow through a control surface

Figure 3.1 shows inclined surfaces through which some of mass enters or leaves. The mass flow
rate entering through surface A1 is

ṁin = density × velocity × area = ρ1 v1x A1 (3.7)

where v1y does not enter into the control volume.


The mass flow rate coming out through surface A2 is

ṁout = ρ2 v2y A2 (3.8)

For clear understanding, let’s consider a body composed of three fishes as shown in Fig. 3.2.
Their masses are 1, 2, and 3 kg, respectively.

Figure 3.2: Fish at time t and t + ∆t in a square.

Ex. 7. Find the amount of salt (mS ) in the tank at any time t. Initially, the amount of salt in the
tank was 100 kg out of 1000 kg brine.
3.1. MASS BALANCE FOR A STEADY-STATE FLOW 35

Figure 3.3: A mixing process.

Answer: Follow the sequence


1. Choose a control volume as the dot line.
2. Find the total amount of brine in the tank as a function of time using the mass balance for
the total amount of brine.
∂mT
= 20 − 10
∂t (3.9)
=⇒ mT = 10t + 1000 (kg) ∵ at t = 0, mt = 1000 kg
3. Express the amount of salt in the tank as a function of time using the mass balance.
 
∂mS mS
= (0.2)(20) − (10)
∂t 1000 + 10t
∂mS mS R 1
=⇒ =4− (a first-order linear ODE, IF =e 100+t dt ) (3.10)
∂t 100 + t
10000 + 400t + 2t2
=⇒ mS = (kg) ∵ at t = 0, mS = 100 kg
100 + t

3.1 Mass Balance for a Steady-State Flow


As mentioned in Sect. 2.3.3, the partial time derivative is zero at the steady-state flow and thus
∂m
=0 =⇒ ṁin = ṁout (3.11)
∂t

Ex. 8. Find the exiting water’s average velocity. Assume that the density is the same everywhere.

Answer: First, let control volume be the total fluid in the channel. Mass flow rate entering into the
control volume is
ṁin = ρvin Ain = 8ρ (3.12)
Mass flow rate exiting from the control volume is
ṁout = ρvout Aout = ρvout (3.13)
since ṁin = ṁout , vout = 8 m/s. However, it is the velocity perpendicular to surface Aout because
the velocity in (3.5) should be perpendicular to the area. Thus the outer velocity becomes
v2 = vout / cos 60 = 16 m/s (3.14)
36 CHAPTER 3. CONSERVATION OF MASS

1m

2m Aout
1m
2 m/s

60o

2m

Figure 3.4: Steady one dimensional flow into and out of a control volume.

3.2 Equation of Continuity


The differential form of mass balance (the differential mass balance or the equation of continuity)
can be derived by applying a mass balance over a differential element.
Consider the mass balance over a differential element, a special type of control volume. The mass

(x+∆x, y+∆y, z+∆z)

(x, y, z)

Figure 3.5: Mass flux through a differential control volume

balance(or the conservation of mass) within the differential element says


     
Rate of mass accumulation Rate of mass flow into Rate of mass flow out of
= − (3.15)
within a differential element the differential element the differential element

∂m
= ṁin − ṁout (3.16)
∂t
• Mass in the differential element shown in Fig. 3.5 is

m = density × volume = ρ∆x∆y∆z (3.17)

and the time rate of change of mass in the differential element is


(ρ∆x∆y∆z) (3.18)
∂t

• Mass flow rate entering into and leaving from the element
3.2. EQUATION OF CONTINUITY 37

The mass flow rate entering the differential element through the surface at x is
(ρvx )x ∆y∆z (3.19)
and the mass flow rate leaving the differential element through the surface at x + ∆x is
(ρvx )x+∆x ∆y∆z (3.20)
Thus the net mass flow rate out of the differential element in the x direction is written as
[(ρvx )x+∆x − (ρvx )x ] ∆y∆z (3.21)
Similarly we can get the net mass flow rates in the y and z directions.
[(ρvy )y+∆y − (ρvy )y ] ∆x∆z, and [(ρvz )z+∆z − (ρvz )z ] ∆x∆y (3.22)

Using the mass balance (3.15) and (3.18) through (3.22), we finally have

(ρ∆x∆y∆z) + [(ρvx )x+∆x − (ρvx )x ] ∆y∆z
∂t
+ [(ρvy )y+∆y − (ρvy )y ] ∆x∆z + [(ρvz )z+∆z − (ρvz )z ] ∆x∆y = 0 (3.23)
Since the volume does not change with time, we divide (3.23) by ∆x∆y∆z and take the limit as
∆x → 0, ∆y → 0, and ∆z → 0
∂ρ ∂ ∂ ∂
+ (ρvx ) + (ρvy ) + (ρvz ) = 0
∂t ∂x ∂y ∂z
∂ρ
=⇒ = −∇ · (ρv)
∂t
|{z} | {z }
net influx of mass per unit length in all direction
rate of mass per volume accumulation at a point
=한 점에서 각 방향으로 단위길이당 순유입 mass flux
=한 점에서 시간에 대한 단위부피당 질량(=밀도)의 변화율
(3.24)
This is called the differential mass balance or the equation of continuity which is the mass balance
required at a point.
With the help of the definition of divergence (1.21)
 
∂(ρvx ) ∂(ρvy ) ∂(ρvz ) ∂ρ ∂ρ ∂ρ ∂vx ∂vy ∂vz
∇ · (ρv) = + + = vx + vy + vz +ρ + +
∂x ∂y ∂z ∂x ∂y ∂z ∂x ∂y ∂z (3.25)
= v · ∇ρ + ρ(∇ · v)
The equation of continuity can be also expressed in terms of the material derivative (2.13)

+ ρ(∇ · v) = 0 (3.26)
Dt
As shown in Fig. 3.6, the Dρ/Dt is the time rate of change of density in a fluid body composed of
the material particles when the fluid particles are in a motion. Thus,
m', V'
m, V
motion (v)

Dρ/Dt
Body

Figure 3.6: Physical meaning of ∇ · v. Here V and V ′ are very small such as a point.

D
  
Dρ Dt m/V DV Volumetric increase
∇·v =− ρ=− = V = (3.27)
Dt m/V Dt Original volume
The physical meaning of divergence is well explained in movie clips (https://www.khanacademy.org
/math/multivariable-calculus/multivariable-derivatives/divergence-grant-videos/v/divergence-intuition-
part-1?modal=1). You can clearly see the concept of the divergence of velocity through Fig. 3.7
38 CHAPTER 3. CONSERVATION OF MASS

v=xex-yey
v=xex

(2,0) (2,0)

∇·v = 0 ∇·v > 0 ∇·v < 0 ∇·v > 0 ∇·v = 0

Figure 3.7: The divergences of the points which are composed of material particles. The point (2, 0)
is an example.

∇·v > 0

∇·v < 0

∇·v = 0

B point B: ∇·v > 0

A
point A: ∇·v = 0

C point C: ∇·v < 0

Figure 3.8: Divergences of fluid flows. The above three flows have the same sign at any points whereas
the forth flow has a positive divergence of v for upper region and negative for the lower region,
respectively. (see Khan Academy)
3.2. EQUATION OF CONTINUITY 39

3.2.1 The Differential Mass Balance for an Incompressible Flow


Suppose a fluid in a container like a balloon. If the container is not compressed by increasing pressure,
the fluid is incompressible. In other word, a fluid is considered as an incompressible fluid if its
density1 does not change when the pressure changes. In liquids, ρ is nearly independent of pressure
and spatially uniform unless there are significant variations in temperature. However, the density
of a gas is related to P and T by an equation of state ρ = M P/RT . If a fluid is incompressible,
ρ = constant, then the differential mass balance (3.24) for the flow of an incompressible fluid is
simplified to (see Appendix B)
∇·v =0 (3.29)
In order words, an incompressible fluid always undergoes an isochoric motion (no volume change) or
incompressible flow.
In reality, there is no incompressible fluid and all of fluids possess a certain amount of compress-
ibility.2 Fortunately, for many fluid flow situations, the changes in volume due to changes in pressure
associated with the flow are very small (∇ · v ≈ 0), even though the fluid is compressible. Therefore,
most of fluid flows can be treated as an incompressible (or isochoric) flow, which means the
volume of a body does not change so that the density is nearly constant (ρ ≈ constant) during the
flow regardless of the compressibility of the fluid. That is, even the compressible fluid can form an
incompressible flow and eventually its density keeps constant during the flow. Eventually, assuming ρ
to be constant, even in gases, usually leads to negligible error. These are illustrated in Fig. 3.9

no volume
Incompressible Fluid Incompressible flow (isochoric motion)
change
(ρ = constant) ∇·v = 0

Fluid
almost no
Incompressible flow ( ρ ≈ constant
vol. change
Compressible Fluid ∇·v ≈ 0 during the flow)
dependence of volume
ρ on pressure Compressible flow ( ρ ≠ constant
change ∇·v ≠ 0 during the flow)

Figure 3.9: Flows of incompressible and compressible fluids.

According to the coordinates used, equation (3.29) could be expressed in terms of velocity com-
ponents as

∂vx ∂vy ∂vz


In RCC + + =0
∂x ∂y ∂z
1 ∂ 1 ∂vθ ∂vz
In Cyl. (rvr ) + + =0 (3.31)
r ∂r r ∂θ ∂z
1 ∂ 1 ∂ 1 ∂vϕ
r 2 vr +

In Sph. 2
(vθ sin θ) + =0
r ∂r r sin θ ∂θ r sin θ ∂ϕ

Ex. 9. You can easily see that a fluid is not incompressible when its velocity profile is v = xex
because ∇ · v = 1.
1 The density at any point in a fluid is defined as
ρ = lim ∆m/∆V (3.28)
∆V →0

2 The compressibility is a measure of the relative volume change of a fluid or solid as a response to a pressure (or

mean stress) change:


1 ∂V
β=− (3.30)
V ∂P
The compressibility of an incompressible fluid is always zero.
40 CHAPTER 3. CONSERVATION OF MASS

Ex. 10. Filtration in a hollow fiber. A common type of membrane filtration unit consists of many
hollow fibers arranged in parallel. The wall of each fiber has fine pores, and a pressure difference causes
outward flow of the solvent, thereby concentrating macromolecules or particles that are retained inside
the fiber. Figure 3.10 shows a single fiber of radius R and length L. Predicting the performance of
the device requires knowledge of v inside the fiber. The cylindrical velocity components of interest are
vz (r, z) and vr (r, z). The flow is assumed to be axisymmetric (independent of θ) and without swirl
(vθ = 0). At any position along the fiber the mean axial velocity is u(z) and the radial velocity at
the wall is vw (z) = vr (R, z). The mean velocities at the inlet and outlet are u0 and uL , respectively.
Illustrate the three levels of detail at which conservation of mass can be applied: all liquid inside the
fiber, the dotted region, a point in the fibre.

Figure 3.10: Filtration in a hollow fiber.

Answer: Consider the mass balance over each region.


All liquid region: An overall relationship is obtained by choosing as the control volume all liquid
inside the fiber. The mass balance (3.3) is expressed as
Z L !
∂ 2 2
(ρπR L) + ρπR uL + 2πR ρvw dz − ρπR2 u0 = 0 (3.32)
∂t 0

With ρ constant, it becomes


Z L
πR2 u0 = πR2 uL + 2πR vw dz (3.33)
0
The volume inflow and outflow must be equal. This may be rearranged as
2 L
Z
uL = u0 − vw dz (3.34)
R 0
which shows how much the mean velocity at the outlet is reduced by the filtration.
Dotted region: An intermediate level of detail comes from selecting a control volume of radius R
and differential length ∆z that is located at some position z, as shown by the dashed lines. Because
3.2. EQUATION OF CONTINUITY 41

one of the dimensions is differential, what is to be constructed is a kind of shell balance. Here a shell
indicates a differential element of the flow. Equating volume inflow and outflow for this region gives

πR2 u(z) = πR2 u(z + ∆z) + (2πR∆z)vw (z) (3.35)

Dividing by ∆z and letting ∆z → 0 provides a differential equation that governs the mean velocity,
du 2vw
=− , u(0) = u0 (3.36)
dz R
If vw (z) is known, this can be solved for u(z). Thus, this approach provides the mean velocity at
intermediate locations along the fiber. The simplest possibility is a constant wall velocity, for which
2vw z
u(z) = u0 − , (constant vw ) (3.37)
R
A point: The most detailed information on v is contained in (3.29). With ρ constant and vθ = 0, the
cylindrical continuity equation reduces to
1 ∂ ∂vz
(rvr ) + =0 (3.38)
r ∂r ∂z
Equations (3.34) and (3.36) each could have been obtained also from a more detailed form of conser-
vation of mass. Equation (3.34) could have been found by integrating (3.36) over the channel length.
Likewise, Eq. (3.36) could have been derived by integrating (3.38) over the cross-section, as follows.
The differential area for the circular cross-section is dS = rdrdθ. Integrating each term in (3.38) gives
Z 2π Z R   r=R
1 ∂
(rvr ) rdrdθ = 2π(rvr ) = 2πRvw
0 0 r ∂r r=0
Z 2π Z R   Z R (3.39)
∂vz d du
rdrdθ = 2π vz r dr = πR2
0 0 ∂z dz 0 dz

and assembling these results leads to (3.36). Although either approach gives the same result, a shell
balance such as (3.35) is generally the simpler way to determine how a mean velocity varies along a
conduit.
Of the three ways to express conservation of mass, only the continuity equation permits evaluation of
an unknown velocity component. If vz (r, z) is given as
  2 
r
vz (r, z) = 2u(z) 1 − (3.40)
R

the velocity component vr can be calculated as follows.


  2    2 
1 ∂ ∂vz du r 4vw (z) r
(rvr ) = − = −2 1− = 1− (3.41)
r ∂r ∂z dz R R R

Multiplying by r, integrating, and then dividing by r gives

r3
 
2vw (z) C
vr (r, z) = r− + (3.42)
R 2R2 r

The constant C is evaluated by requiring that vr be finite everywhere, including r = 0. Accordingly,


C=0
r3
 
2vw (z)
vr (r, z) = r− (3.43)
R 2R2

Summary
42 CHAPTER 3. CONSERVATION OF MASS

질량보존의 법칙
!"
= 0, 물질입자로 이루어진 body내 질량은 시간에 대하여 변하지 않는다.
!#
!"
= #̇ $% − #̇ &'# , 고정된 공간인 control volume내 시간에 대한 질량 변화는
!#

순질량 유입속도와 같다.

• Body: 물고기 3마리로


이루어져 있음.
• CV: 관찰자가 정한 고정
된 공간임.

화학공학부 울산대학교
SCHOOL OF CHEMICAL ENGINEERING UNIVERSITY OF ULSAN

연속방정식 및 비압축성 흐름
• 연속방정식은 한점에서 만족해야 할 질량보존법칙이다.

%&
+ ∇ * &+ = 0
%'
- 비압축성 유체는 압력에 따라 밀도가 변하지 않는 유체이다. 따라서 속도는
발산하지 않는다. ∇ * + = 0
- 압축성 유체 또한 많은 경우 속도가 발산하지 않는 흐름을 보인다. ∇ * + = 0.
따라서 흐름 동안 밀도가 일정하게 유지된다.
• 속도의 발산은 유체가 흘러갈 때 부피의 변화 크기를 나타낸다.
- ∇ * + > 0, 들어오는 속도에 비하여 나가는 속도가 커 부피가 커지는 흐름
- ∇ * + = 0, 들어오는 속도와 나가는 속도가 동일하여 부피증가가 없는
isochoric 흐름

∇"#=0 ∇"#>0 ∇"#<0


화학공학부 울산대학교
SCHOOL OF CHEMICAL ENGINEERING UNIVERSITY OF ULSAN
Chapter 4

Conservation of Linear Momentum


and Its Derivatives

Newton’s second law of motion is


X dv d(mv) dP dm
F=m = = ∵ =0 by the conservation of mass (4.1)
dt dt dt dt
where P is called linear momentum. Thus equation (4.1) is also called the conservation of linear
momentum. Similar to the conservation of mass, the conservation of linear momentum can be expressed
within either a body or a control volume.
1. (Solid Mechanics) The time rate of change of linear momentum of a body or a
system is equal to the sum of forces acting on a body from environment.
dP X
= F (4.2)
dt
2. (Fluid Mechanics) The time rate of change of linear momentum within the control
volume is equal to the net rate of linear momentum influx into the control volume
plus the sum of forces acting on a control volume from environment.
∂P X
= Ṗin − Ṗout + F
∂t
|{z}
|{z} | {z } | {z }
rate of linear momentum rate of linear momentum sum of forces acting
rate of linear momentum
into control volume out of control volume on control volume
accumulation within control volume
(4.3)
Here P is the linear momentum in the control volume which is a fixed region in space as
mentioned in Sect. 2.2. Equation (4.3) is equivalent to (4.2).

4.1 Force Acting on a Fluid


All of the forces acting on a fluid are divided into three general groups if there is an interfacial tension
force acting on an interface contour: body, surface, and surface tension forces.
1. Body force is a force that acts throughout the volume of a fluid body without physical contacts.
- gravity, electrostatic force
2. Surface (or contact) force is a force that acts on the surface of a fluid body so that it requires
physical contact for transmission.
• Pressure force1 acts to the surface in the normal direction.
1 Positivepressures are compressive (rather than tensile). In other words, pressure pushes on (rather than pulls on)
a surface. Pressure is isotropic. That is, the pressure P (force per unit area) has a single value at any point in a fluid,
and tends to act equally in all directions. It will be shown now that pressure is an isotropic stress, a result known as
Pascal’s law (see p. 116 in Deen’s book).

43
44 CHAPTER 4. CONSERVATION OF LINEAR MOMENTUM AND ITS DERIVATIVES

• Friction force (Wikipedia) is a surface force resisting the relative motion of solid surfaces,
fluid layers, and material elements sliding against each other.2
– Fluid friction (viscous force): describes the friction between layers of a viscous fluid
that are moving relative to each other. Layers of a fluid can slide past each other, and
the viscous force resists the sliding. The internal resistance to flow is named viscosity.
– Skin friction: describes the surface force resisting the motion of a fluid across the
surface of a body. Fluids do not slide along surfaces, the so-called no slip condition.
This causes the skin friction.

Fs Fluid Fluid Fluid


50km/h 100km/h 50km/h
Pressure
+ Friction

Fb Fluid or Solid Fluid or Solid
Fluid or Solid
Friction(tangential) Friction(normal)

Figure 4.1: 유체에 작용하는 힘은 체적에 작용하는 body force (Fb ), 표면에 작용하는 surface force
(Fs ), 계면위 임의의 line 또는 contour에 작용하는 surface tension force(Fγ )로 이루어짐. Surface force
는 수직방향의 pressure과 수평 및 수직 방향의 friction의 합으로 표현됨.

3. Interfacial (surface) tension force is a tensile tangential force acting on an imaginary line
or contour within a interface between two different phases.
Ex. 11. Determine all forces acting throughout the volume and on the surface of a stationary fluid
in the container shown in Fig. 4.2

FP=압력힘
Patm FP1

FP4 FP2
ρ(밀도) h Fb=중력
y

S(밑면적) x

FP3

Figure 4.2: Forces acting throughout the volume and on the surface of a stationary fluid

Fbody : 중력 = 유체의 무게 = mg = ρShg


FP 1 : 공기가 윗면의 유체에 가하는 압력에 의한 힘 = Patm S
FP 2 : 용기의 오른쪽 벽면이 오른쪽 옆면의 유체에 가하는 압력에 의한 힘 = Pside hl
- 여기서 Pside 는 옆면의 평균 압력으로서 Patm + ρgh/2이다.
FP 3 : 용기의 아랫벽면이 아랫면의 유체에 가하는 압력에 의한 힘 = (Patm + ρgh)S
- 아랫면의 유체가 아랫면의 용기에 미치는 압력은 Patm + ρgh 이다. 따라서 아래면의 용기가
유체에 미치는 압력은 이와 똑같다고 할 수 있다.
FP 4 : 용기의 왼쪽 벽면이 왼쪽 옆면 유체에 가하는 압력에 의한 힘 = Pside hl

2 Besides the following two frictions, there are other types of friction. Dry friction is a surface force that opposes

the relative lateral motion of two solid surfaces in contact. Internal friction is the force resisting motion between the
elements making up a solid material while it undergoes deformation.
4.1. FORCE ACTING ON A FLUID 45
X
F = Fb + FP 1 + FP 2 + FP 3 + FP 4
(4.4)
= −ρShgey − Patm Sey − Pside hlex + (Patm + ρgh)Sey + Pside hlex = 0
P
즉, 외부에서 유체에 작용하는 모든 힘의 합은 0이다. (유체가 정지해 있기 때문에 a = 0 =⇒ F = 0)

Ex. 12. Using Fig. 4.3, show that the horizontal pressure force on the inclined surface is its projected
area times the pressure averaged over that projection.

Air
P = Patm

ρ FP1
h

x FP2

Figure 4.3: Horizontal forces acting on a stationary fluid

유체가 정지해 있으므로 수평힘 및 수직힘 각각의 합이 0이 되어야 한다. 따라서 기울어진 면에 작용하는
수평성분의 압력힘은 왼쪽 수직면에 작용하는 수평성분의 압력힘과 크기가 같아야 한다. 왼쪽면에 작용
하는 수평성분 압력힘 FP 1 은 평균압력 (Pside = Patm + ρgh/2)과 면적 hl의 곱이므로, 기울어진면에
작용하는 힘 FP 2 의 수평성분 또한

FP 2,x = Pside × hl = (기울어진면의 평균압력) × (기울어진면의 투사면적) (4.5)

Ex. 13. Show that the friction force acting on the upper surface of a moving fluid is equal to that
on the lower surface of the fluid (Fig. 4.4). Here the fluid moves due to the constant velocity of the
upper plate v.
Fb : 중력 = 유체의 무게 = mg = ρAw hg
FP 1 : 공기가 왼쪽면의 유체에 가하는 압력에 의한 힘 = Patm A
FP 2 : 공기가 오른쪽면의 유체에 가하는 압력에 의한 힘 = Patm A
FP 3 : 아랫판이 아랫면의 유체에 가하는 압력에 의한 힘 = (Pm + ρgh)Aw
FP 4 : 윗판이 윗면의 유체에 가하는 압력에 의한 힘 = Pm Aw
FP 5 : 공기가 앞면의 유체에 가하는 압력에 의한 힘 = Patm As
FP 6 : 공기가 뒷면의 유체에 가하는 압력에 의한 힘 = Patm As
Fτ 1 : 움직이는 윗판이 유체에 가하는 마찰력(no slip) = τ1 Aw
* 윗면의 유체가 움직이는 윗판에 가하는 마찰력은 크기는 같고 방향은 반대방향임 = −Fτ 1
Fτ 2 : 아랫판이 아랫면의 유체에 가하는 마찰력(no slip) = τ2 Aw
*흐르는 유체가 아랫판에 가하는 마찰력은 크기는 같고 방향은 반대방향임 = −Fτ 2

X
F = Fb + FP 1 + FP 2 + FP 3 + FP 4 + FP 5 + FP 6 + Fτ 1 + Fτ 2
= −ρhgAw ey + Patm Aex − Patm Aex + (Pm + ρgh)Aw ey − Pm Aw ey
(4.6)
− Patm As ez + Patm As ez + τ1 Aw ex − τ2 Aw ex = 0
=⇒ τ1 = τ2
P
유체가 일정속도로 움직이므로 운동량의 변화가 없어 a = 0 =⇒ F = 0.
46 CHAPTER 4. CONSERVATION OF LINEAR MOMENTUM AND ITS DERIVATIVES

Aw(판면적) v

Patm ρ(밀도) A(단면적)


h
As(앞면적)

FP4=압력
x 압력=FP6
z Pm
Fτ1=마찰력

FP2=압력
압력=FP1 Fb=중력

압력=FP5 Fτ2=마찰력

FP3=압력

Figure 4.4: Forces acting throughout the volume and on the surface of a moving fluid between two
plates

4.2 Interfacial Tension Force (Surface tension, Interfacial en-


ergy)
If a control surface includes a line or a curve encompassing an interface between any two different
phases, then we have to add the force associated only with the line or curve as well as body forces in
the control volume and surface forces on the control surface. It is called surface tension force.
X
F = Fb + Fs + Fγ (4.7)

Suppose an air-water interface shown in Fig. 4.5. Each molecule in water bulk is pulled equally
in every direction by neighboring water molecules, resulting in zero net force. On the other hand,
the interface molecules experience air-water adhesive forces with the above air molecules as well as
water-water cohesive forces with the below water molecules. Because the air-water adhesive force is
not as strong as the water-water cohesive force, the surface molecules feel a net force toward the
bulk of water. This inward force makes the interface to be tense, ultimately forcing liquid surface
to contract to the minimal area. In this viewpoint, the interfacial (or surface) tension3 (γ) is a
tensile force per unit length (N/m) that acts on an imaginary line or contour within a
interface. The direction of the surface tension force is tangent to the interface and away from the line
of contour. The forces (γL) are tensile, meaning that each pulls on the imaginary line. If γ is constant
and the interface is planar, the surface tension force will balance and the surface tension will not
be evident. However, at an interface with curvature, the surface tension generally creates a pressure
difference between the phases. For example, the pressure inside a small drop or bubble exceeds that
in the surrounding fluid. (See Ex. 14)
Two examples on the effects of surface tension are also shown in Fig. 4.5. The above shows water
adhering to the faucet gaining mass until it is stretched to a point where the surface tension can no
longer keep the drop linked to the faucet. It then separates and surface tension forms the drop into a
sphere. If a stream of water was running from the faucet, the stream would break up into drops during
its fall. Gravity stretches the stream, then surface tension pinches it into spheres. Water striders use
3 In general, the interfacial tension is called surface tension in the case that the interface is composed of gas-liquid

and two immiscible liquid-liquid phases.


4.2. INTERFACIAL TENSION FORCE (SURFACE TENSION, INTERFACIAL ENERGY) 47

Air

Interface
γL
γL

Bulk
Liquid

Figure 4.5: Surface tension of the interface between air and water. A force of magnitude γL pulls away
from each side of the imaginary cut shown by the dashed line. Two effects of surface tension are also
shown; the formation of a drop from faucet and the flotation of water strider (Wikipedia).

surface tension to walk on the surface of a pond in the following way. The nonwettability of the water
strider’s leg means there is no attraction between molecules of the leg and molecules of the water,
so when the leg pushes down on the water, the surface tension of the water only tries to recover
its flatness from its deformation due to the leg. This behavior of the water pushes the water strider
upward so it can stand on the surface of the water as long as its mass is small enough that the water
can support it. The surface of the water behaves like an elastic film (Wikipedia).
Table 4.1 shows values of γ for several liquids in contact with air. The surface tension for water is
about three times that for benzene, ethanol, or other common organic solvents. Mercury is unusual in
that γ is about 20 times that for the organics. Surface tensions decrease with increasing temperature,
as emplified by the entries for water and mercury.

Table 4.1: Surface tension for liquid-air interfaces

Liquid T (K) γ(mN/m)


Water 293 72.88
303 71.40
313 69.92
Benzene 293 28.88
Ethanol 293 22.39
Mercury 293 486.5
303 484.5

Besides, we can see the surface tension at work when we fill a glass. Even if the water is at the rim
of the glass, we can add just a few more drops so that the water is slightly taller than the rim. When
surfactants are added to water, the surfactants gather at the surface of water because they are lighter
than water. The intermolecular force between water molecules becomes weaker due to the existence
of surfactant molecules between them. The surfactants actually reduce the surface tension of water
by a factor of three or more. Thus it is possible to make thin soap bubble.
Sometimes, surface tension is viewed as energy per unit area (J/m2 ). A molecule in a bulk phase
is in a lower state of energy due to balancing with neighbor molecules, whereas the surface molecules
are missing neighbors and thus have higher energy. For the liquid to minimize its energy state, the
number of higher energy surface molecules should be minimized. Additionally, in order to have new
surface molecules by creating new interface, some of energy should be required. With this viewpoint,
the surface tension is called the interfacial energy (or surface energy only in the case of gas-liquid
or liquid-liquid interface), which is an energy per unit area (J/m2 ) required to increase  the
interfacial area in thermodynamics. It can be thermodynamically expressed as γ = ∂G ∂A T,P,n .
More details on the surface tension is described with mathematical approach in ”Interfacial Trans-
48 CHAPTER 4. CONSERVATION OF LINEAR MOMENTUM AND ITS DERIVATIVES

port Phenomena” written by J. Slattery, L. M. C. Sagis, and Eun-Suok Oh (Springer 2007).

Ex. 14. Young-Laplace equation. Determine the pressure difference across the surface of a sta-
tionary spherical bubble of radius R in terms of the surface tension and bubble size. It is assumed
that the internal and external pressures are nearly uniform. The limitation of that assumption will
be examined later.
Answer: A convenient control volume is a hemispherical shell of infinitesimal thickness that encloses
half of the gas-liquid interface, as illustrated in Fig. 4.6. With this choice, the surface tension force
that the surroundings exert on the control volume is downward in the diagram, which is 2πRγ. The
pressure force is simply calculated by the projected-area concept. The uniform pressures each act on
a projected area πR2 . Gravity need not be considered because the mass in the thin control volume is
negligible. The net force acting downward is then

Pout πR2 − Pin πR2 + 2πRγ = 0 (4.8)

and thus the pressure difference is



∆P = Pin − Pout = (4.9)
R
which is called the Young-Laplace equation. This holds for a spherical droplet in a gas. The pressure
inside the fluid sphere is higher than that outside. The analogous results for a cylindrical interface of
radius R is ∆P = γ/R under the assumption of l ≫ R.

Liquid
Pout
Pin R

γ Gas

Figure 4.6: Pressure and surface tension acting on a spherical bubble. The control volume is the
hemispherical shell indicated by the dashed curves.

The assumption of uniform pressures requires the bubble (or droplets) to be small. More precisely,
the pressure variation within each fluid must be negligible relative to ∆P . In the case of liquid, the
gravity-induced pressure variation over a height 2R of liquid is 2ρL gR. Accordingly, the Young-Laplace
equation requires that

2γ ρL gR2 Gravitational force


2ρL gR ≪ ⇒ Bo = = ≪1 (4.10)
R γ Surface tension force
where Bo is the Bond number. For an air-water interface at room temperature, the radius for which
Bo=1 (the capillary length) is 3 mm. Radii smaller than this are needed for a static bubble or drop
to be spherical.

Ex. 15. Capillary rise. Capillary action (sometimes, capillarity or capillary motion) is the ability
of a liquid to flow in narrow spaces without the assistance of, or even in opposition to, external
forces like gravity. A consequence of the pressure difference resulting from the surface tension is the
phenomenon of capillary action.4 A common apparatus used to demonstrate the first phenomenon is
the capillary tube. When the lower end of a vertical glass tube is placed in water, a concave meniscus
forms (capillary rise). On the other hand, a convex meniscus forms when it is placed in mercury
4 This effect is related to how well a liquid wets a solid boundary. The indicator for wetting or nonwetting is the

contact angle α. A nonwetting case is associated with α > 90◦ , such as mercury in contact with a clean glass tube,
α ≈ 130◦ . For a wetting case α < 90◦ .
4.2. INTERFACIAL TENSION FORCE (SURFACE TENSION, INTERFACIAL ENERGY) 49

(capillary depression) due to its strong intermolecular forces. This is caused by the difference between
the liquid cohesion and the liquid adhesion with the solid tube.
In the case of water, the adhesion between water molecules and the glass tube is stronger than the
cohesion between water molecules at the surface, resulting in the formation of thin film of water on
the tube wall. Then, the surface tension pulls water molecules at the center to minimize the interfacial
area. The tube suck up water to a certain height where the weight of the sucked water reaches to the
adhesion, as shown in Fig. 4.7.

γ α
CV1
Air 1 R γ
P = P0 CV2
2 α
Rm
H
α
α

Water

Figure 4.7: Rise of water inside a small, wettable glass tube. 1 and 2 indicates the tops of control
volumes CV1 and CV2, respectively.

Show how H depends on the tube radius (R), surface tension (γ), and contact angle (α).

Answer: The analysis can be done using force balances on either of two control volumes; CV1 and
CV2. The difference between them is that the top of CV1 is on the air side of the meniscus and that
of CV2 is on the liquid side.
• Method 1: With CV1 the control surface cuts through the air-water interface. The surroundings
therefore exert a surface tension force on the liquid, the upward component of which is 2πRγ cos α.
The gravitational force on the liquid of CV1 acts in the downward direction, which is ρgπR2 H
under the assumption that H ≫ R. There is no net pressure force, because the top and bottom
of CV1 are the same pressure (P0 ) and have the same projected arer (πR2 ). Requiring that the
upward forces in this static system sum to zero, we find that
2πRγ cos α − ρgπR2 H = 0 (4.11)
Solving for H gives
2γ cos α
H= (4.12)
ρgR
Thus, the smaller the tube radius and the smaller the contact angle, the higher the liquid will
rise.
• Method 2: With CV2 the control surface does not cut through the air-water interface, and
therefore there is no surface tension force from the surroundings. However, there is a net pressure
force, because surface tension within the curved surface lowers the pressure at the top of the
liquid column. If Bo ≪ 1, the gas-water interface will resemble part of a spherical bubble and
the Young-Laplace equation will apply. From Eq. (4.9), the pressure on the liquid side is then
P0 − 2γ/Rm . Accordingly, the vertical force balance for CV2 is
 
2 2γ cos α
P0 πR − P0 − πR2 − ρgπR2 H = 0 (4.13)
R
This will give the same result as Eq. (4.12).
It is equally valid to attribute capillary rise to (1) surface tension pulling upward on the liquid, or (2)
surface tension creating a partial vacuum at the top of the liquid column.
50 CHAPTER 4. CONSERVATION OF LINEAR MOMENTUM AND ITS DERIVATIVES

4.3 Linear Momentum and Angular Momentum


Before we discuss the momentum balance, let’s see the definition of linear and angular momentums.
Momentum can be considered as the power residing in a moving object.
Linear momentum is a vector quantity associated with how a mass moves along a straight path
and the product of the mass and velocity of an object:

P = mv [momentum = inertia × velocity] (4.14)

As shown in Fig. 4.8(a), the linear momentum represents the effect of mass and velocity simultaneously.
By the Newton’s second law of motion, the linear momentum should be conserved if the external forces
are not given. The linear momentum flow rate becomes

m
v
r mv

+
mv mv
r

(a) Linear momentum (b) Angular momentum

Figure 4.8: Physical meaning of linear and angular momentums.

Ṗ = ṁv = ρAv⊥ v (4.15)

where v⊥ is the velocity component perpendicular to surface A. When the velocity v⊥ is not constant
on the surface A, the linear momentum flow rate becomes
Z
Ṗ = ρv⊥ v dA (4.16)

Now consider an object rotating with an equal speed shown in Fig. 4.8(b). In this case, the linear
momentum mv is continuously changed due to the change of direction of motion. However the vector
quantity
H ≡ r × P = r × mv = Iω (4.17)
keeps constant during the rotational motion. H is called the angular momentum (moment of momen-
tum) which measures how much the linear momentum is rotating around a certain point called the
origin. Similar to the linear momentum expressed as linear inertia(m) and linear velocity(v), the angu-
lar(rotational or moment of) momentum can be written as rotational inertia times rotational(angular)
velocity since
I = m|r|2 (4.18)
and
r× v
v=ω × r → ω= (4.19)
|r|2
The angular momentum should be conserved if the external rotational forces(moment) are not
given.5
5 피겨스케이팅 선수가 빙판위에서 회전운동을 할때는 팔을 몸에 바짝 붙어서 회전을 한다. 회전할 때 외부에서 토크(돌림 힘)

가 주어지지 않기 때문에 각운동량 H = Iω은 보존된다. 따라서 rotational inertia(회전관성)을 I = mr2 을 줄여서 각속도가
크게 하여야 하므로 팔이 몸쪽에 붙도록, 즉 r 적어지도록 하여야 한다. 반면에 회전을 멈추고자 할때는 손을 펼쳐서 회전관성을
최대한 크게 해야 각속도가 줄어들게 된다.
4.4. LINEAR MOMENTUM BALANCE FOR A STEADY FLOW 51

4.4 Linear Momentum Balance for a Steady Flow


Since the partial time derivative for a steady state flow is zero, the conservation of linear momentum
(or the linear momentum balance) (4.3) becomes
X
Ṗout − Ṗin = F (4.20)

It means that
X X X
(Ṗout,x − Ṗin,x )ex + (Ṗout,y − Ṗin,y )ey + (Ṗout,z − Ṗin,z )ez = Fx ex + Fy ey + Fz ez (4.21)

and thus
X X X
Ṗout,x − Ṗin,x = Fx , Ṗout,y − Ṗin,y = Fy , Ṗout,z − Ṗin,z = Fz (4.22)

Ex. 16. A jet of water exits a nozzle and strikes a vertical plane surface as shown in Fig. 4.9.
Determine the force required to hold the plate stationary.

A1 A2

Figure 4.9: A fluid jet striking a vertical plate. 유체로만 CV를 정하면 x 방향의 반대로 720 N 힘이 plate
오른쪽에 가해지고 이로 인하여 plate 왼쪽면이 유체에 720 N힘을 가해서 노즐에서 나온 물의 x 방향의
운동량이 720 N 만큼 감소하였고, y 방향으로 힘이 주어지지 않았기 때문에 전체 y 방향의 운동량은 0을
유지함.

• Choose a control volume: Fig. 4.9.


• Use (4.22) to solve the problem.
Consider x direction only because no force in y direction needed. The linear momentum balance
in the x direction becomes
X
−Ṗin,x = −ρAj vj (vj ) = Fx = −F + Patm A1 − Patm A2 (4.23)

where F is the external (surface) force acting on the control surface, which is given from outside.
Therefore
F = ρAj vj 2 = 1000 kg/m3 (0.005 m2 )(12 m/s)2 = 720 N (4.24)
The control volume shown in Fig. 4.9 for which the above solution was obtained represents only
possible choice. Another can be chosen, even though wrong choice of the control volume may not give
the solution.
Ex. 17. Water flows steadily through the 90◦ reducing elbow shown in Fig. 4.10. At the inlet to the
elbow, the absolute pressure is 220 kPa and the cross-sectional area is 0.01 m2 . At the outlet, the
cross-sectional area is 0.0025 m2 and the velocity is 16 m/s. The elbow discharge to the atmosphere.
Determine the force required to hold the elbow in place, Rx and Ry . Here the density of incompressible
water is 999 kg/m3 . Neglect weight of elbow and water in elbow.
52 CHAPTER 4. CONSERVATION OF LINEAR MOMENTUM AND ITS DERIVATIVES

Pabs=220 kPa Patm

As2
As1
y P1 1 W Rx

R1 Ry 2
R2
x
P2

Figure 4.10: Flow in a reducing elbow. Here we choose the dot line as a control volume. The second
figure shows all forces acting on the control volume. 파이프 내 유체만을 CV로 정하면 다음과 같은
해석이 가능함. 입구에서 유체의 x방향 운동량과 gauge 압력힘이 1.35 kN 이었는데 출구에서는 0이 되
었으므로 −x 방향으로 1.35 kN 힘이 파이프의 안쪽면에서 유체로 가해졌음. 이와 같은 힘은 외부에서
파이프의 바깥쪽면으로 동일한 힘(크기 및 방향)이 주어졌기 때문임. 입구에서 y방향 운동량 0 이었는데
출구에서는 -639 N이 되었으므로 −y 방향으로 639 N 힘이 가해졌음. 이 힘에는 중력도 포함되어 있음.
예를 들어 유체의 중력이 10 N 작용한다면 파이프 안쪽면에서 유체에 가해진 −y 방향 총힘은 629 N임.

Answer: • Choose a control volume : the dot line


• Use (4.22) to solve the problem.
In order to determine the inlet velocity v1 , use the mass balance. Since this is a steady-state flow,

ṁin = ṁout =⇒ ρA1 v1 = ρA2 v2 =⇒ v1 = 16 × 0.0025/0.01 = 4 m/s (4.25)

The linear momentum balance in the x direction becomes


X
− Ṗin,x = −ρA1 v1 (v1 ) = Fx = Rx + Patm (As1 − A1 ) + P1 A1 − Patm As1
(4.26)
=⇒ Rx = −ρA1 v1 2 − P1g A1

where the gauge pressure at position 1 is P1g = P1 − Patm . The Rx and Ry are the external
(body) forces acting on the control volume from outside (more exactly, the force given to the
elbow in order to hold it in place).
The linear momentum balance in the y direction becomes
X
Ṗout,y = ρA2 v2 (−v2 ) = Fy = Ry + Patm As2 − Patm As2 − W
(4.27)
=⇒ Ry = −ρA2 v2 2 + W

By neglect the weight, W = 0. Thus

Rx = −999 × 0.01 × 42 − (220 − 101) × 1000 × 0.01 = −1.35 kN


(4.28)
Ry = −999 × 0.0025 × 162 = −639 N

Thus, 1.35 kN and 639 N are the forces given to the elbow in the negative x and y directions in order
to hold the elbow.

Ex. 18. In the steady-state flow system of incompressible water shown in Fig. 4.11, the total drag on
the object (흐르는 유체가 고체에 미치는 흐름방향 힘) is measured to be 100,000 N/m of length normal
to the direction of flow, and frictional force at the walls are neglected, find the pressure difference
between inlet and outlet sections
4.4. LINEAR MOMENTUM BALANCE FOR A STEADY FLOW 53

2m 4m

5 m/s
P3

τw

P1 τo P2

τw

P4
Figure 4.11: A two-dimensional object is placed in a 4-ft-wide water tunnel.

Answer: The conservation of mass of a fluid in the steady-state incompressible flow says that at
Z
ṁin = ṁout (Qin = Qout ) =⇒ v1 A1 = vout dA
Z 1 Z 2 
=⇒ 4v1 = 2 v2 y dy + v2 dy (4.29)
0 1
4 20
=⇒ v2 = v1 = m/s
3 3
The linear momentum balance in the x direction is
X
Ṗout,x − Ṗin,x = Fx = P1 A − P2 A − τo Ao − 2τw Aw (4.30)

Since τw is negligible and τo Ao = 100, 000 N/m, we will have

(P1 − P2 )A = Ṗout,x − Ṗin,x + τo Ao (4.31)

The incoming linear momentum is

Ṗin,x = ṁv1 = ρAv12 = 103 × 4 × 52 = 100, 000 N/m (4.32)

The outgoing linear momentum is


Z Z 1 Z 2 
2 2
Ṗout,x = ρvx vx dA = 2ρ (v2 y) dy + (v2 ) dy
0 1
2 (4.33)
v22
  
8 20
= 2ρ + v22 = × 103 × = 118, 519 N/m
3 3 3

Therefore the pressure difference between inlet and outlet is

P1 − P2 = 118, 519/4 = 29, 630 N/m2 (4.34)


54 CHAPTER 4. CONSERVATION OF LINEAR MOMENTUM AND ITS DERIVATIVES

4.4.1 Linear Momentum Balance over a Moving Control Volume


See the following examples. Note that if we sit on the moving control volume and see the control
volume, it looks like a fixed volume to our eyes.

Ex. 19. (Control volume is moving with a constant velocity) Consider the steam locomotive tender
schematically illustrated in Fig. 4.12, which obtains water from a trough by means of a scoop. Find
the force on the train due to the water. Here the force due to pressure and friction are to be neglected.

Figure 4.12: Schematic of locomotive tender scooping water from a trough.

Answer: The logical choice for a control volume in this case is the water-tank/scoop combination.
Our control volume will be selected as the interior of the tank and scoop. As the train is moving with
a uniform velocity, there are two possible choices of coordinate systems. We may select a coordinate
system either fixed in space or moving with the velocity of the train v0 . The control volume is inertial,
since it is not accelerating.
• Case 1: Stationary coordinate system but moving control volume (Fig. 4.13).

Figure 4.13: Stationary coordinate system and moving control volume.

All the velocities must be measured relative to the stationary coordinate system. The x compo-
nent of the momentum balance is
∂Px X ∂
= Ṗin,x − Ṗout,x + Fx =⇒ (mvx ) = ṁin (0) + Fx (4.35)
∂t ∂t
Here Fx the total force exerted on the fluid by the train and scoop. The entering velocity of the
water is zero relative to the stationary coordinate. Also the velocity of the control volume in the
x direction vx is
vx = v0 (4.36)

From the conservation of mass, we have

∂m
= ṁin − ṁout = ρhv0 (4.37)
∂t | {z }
0

Finally, the force acting on the train by the fluid R can be also expressed as

R = −Fx = −ρhv02 (4.38)


4.4. LINEAR MOMENTUM BALANCE FOR A STEADY FLOW 55

Figure 4.14: Moving coordinate system and control volume.

• Case 2: Moving coordinate system (Fig. 4.14).


All the velocities must be measured relative to the moving coordinate system. The x component
of the momentum balance is
∂Px X ∂
= Ṗin,x − Ṗout,x + Fx =⇒ (mvx ) = ṁin (−v0 ) + Fx (4.39)
∂t ∂t
Here Fx the total force exerted on the fluid by the train and scoop.
The velocity of the control volume in the x direction vx
vx = 0 (4.40)
since the control volume is moving with the same velocity of the coordinate. Finally with the
help of (4.37), we have the force acting on the train by the fluid R as
R = −Fx = −ρhv02 (4.41)

Ex. 20. (Control volume is moving with an acceleration) A small rocket, with an initial mass of 400
kg, is to be launched vertically. Upon ignition the rocket consumes fuel at the rate of 5 kg/s and ejects
gas at atmospheric pressure with a speed of 3500 m/s relative to the rocket. Determine the initial
acceleration of the rocket and the rocket speed after 10 sec, if air resistance is neglected.

v(t) v(t) v(t)=0


a(t)
y
M0=400 kg
x

ve-v(t) ve
ve=3500 m/s vs rocket x
Stationary Moving
me=5 kg/s coordinate coordinate

Figure 4.15: A rocket moving with an acceleration

Answer: • Case 1: Stationary coordinate system but moving control volume


All the velocities must be measured relative to the stationary coordinate system. The y compo-
nent of the momentum balance is
∂Py X ∂
= Ṗin,y − Ṗout,y + Fy =⇒ (mvy ) = −ṁe (vout,y ) − mg (4.42)
∂t ∂t
56 CHAPTER 4. CONSERVATION OF LINEAR MOMENTUM AND ITS DERIVATIVES

Since the mass m and the velocity vy of the control volume are changed with time, it becomes
∂m ∂v
v +m = −ṁe (−ve + v) − mg (4.43)
∂t ∂t
The conservation of mass says
∂m
= ṁin −ṁout = −ṁe =⇒ m = M0 − ṁe t (4.44)
∂t |{z}
0

and thus we have


∂v
m = ṁe ve − mg (4.45)
∂t
At last, the acceleration of the rocket can be written as
∂v ṁe ve ṁe ve
a(t) = = −g + = −g + (4.46)
∂t m M0 − ṁe t
and at t = 0
5 × 3500 2
a(0) = −9.8 + = 33.9 m/s (4.47)
400
The velocity becomes
Z t Z t 
ṁe ve M0 − ṁe t
v(t) = a(t) dt = −g+ dt = −gt − ve ln (4.48)
0 0 M0 − ṁe t M0
• Case 2: Moving coordinate system
All the velocities must be measured relative to the moving coordinate system. Thus the velocity
of the control volume is zero. The y component of the momentum balance is
∂Py X ∂
= Ṗin,y −Ṗout,y + Fy =⇒ (mvy ) = − ṁout vout,y −m(g + a) (4.49)
∂t | {z } ∂t
| {z } | {z }
0 −ṁe ve
0, ∵v=0

Here we used that the body force acting on an accelerating object is


Fb = m(g − a) (4.50)
if the rocket is considered as stationary in viewpoint of the observer on the rocket.6 Equation
(4.49) is exactly the same as (4.45).

4.5 Stress : Surface Force per Unit Area at a Point


Now, let’s discuss the concept of stress that is the most important in analyzing Fluid dynamics.

4.5.1 Stress Vector and Stress Tensor


Consider the surface force ∆Fs acting on a surface element ∆A of a body shown in Fig. 4.16. As ∆A
approaches to the smallest meaningful area of statistical averages δA, the surface force per unit area
becomes a vector at a point, which is generally called the stress (or surface force) vector :
Z
∆Fs
t ≡ lim , and thus Fs = t dS (4.51)
∆A→δA ∆A

6 가속되는 좌표계 속에 놓여 있는 물체는 좌표계가 가속되는 방향과 반대 방향으로 힘을 받는 것처럼 느낀다. 이 힘을 관성력

이라고 한다. 가속도 a로 가속되는 좌표계 속에 있는 질량이 m인 물체는 f=ma 만큼의 힘을 가속도의 반대 방향으로 받는 것처럼
느낀다. [네이버 지식백과] 관성력 [慣性力] (Basic 고교생을 위한 물리 용어사전, 2002. 4. 15., 신근섭). 가속도가 중력과 반대
방향으로 작용하는 물체에 작용하는 body force는 중력방향으로 질량×(중력가속도+가속도)로 작용하며, 만약 가속도가 중력과
같은 방향이면 질량×(중력가속도-가속도)의 body force가 중력방향으로 작용한다. 예를 들어 윗 방향으로 2 m/s2 로 가속하고
있는 승강기에 타고 있는 100kg의 질량은 아랫방향으로 약 1200N 의 힘을 가한다. 아인슈타인의 일반 상대성이론에 따르면 가속에
이한 관성력과 질량에 의한 중력은 구분이 안됨.
4.5. STRESS : SURFACE FORCE PER UNIT AREA AT A POINT 57

ΔFs t
tp+tτn
tτt
ΔA
ΔA→0

Figure 4.16: Surface force on an element of fluid. The stress vector is a surface force per unit area
acting at a point on the surface.

The stress vector defined as (4.51) does not include the direction of the surface on which the
surface forces act. As depicted in Fig. 4.17, however, a force at a point exerted by an external surface
force depends on the direction of the surface as well as the direction of the external force. Thus it
is necessary to add information on the direction of the surface to the stress vector. This could be
achieved by introducing the stress tensor (or just stress) T (σ in our textbook p. 132) as

∆F
∆F ∆F

∆A ∆A ∆A

Figure 4.17: A force at a point exerted by an external force depends on both the direction of the
external force and the direction of the surface.

t = TT · n (4.52)
Here n is the unit normal vector to the surface, which gives the information on the direction of the
surface. The unit normal vectors are evaluated generally as
∇G
n= (4.53)
|∇G|
where G(x, y, z) = 0 defines the location of the surface.
In order to know physical meaning of stress T, we expand (4.52) in terms of components:
tx ex + ty ey + tz ez ≡ (Txx ex ex + Tyx ex ey + Tzx ex ez + Txy ey ex + Tyy ey ey + Tzy ey ez
(4.54)
+ Txz ez ex + Tyz ez ey + Tzz ez ez ) · (nx ex + ny ey + nz ez )
and thus
tx = Txx nx + Tyx ny + Tzx nz
ty = Txy nx + Tyy ny + Tzy nz (4.55)
tz = Txz nx + Tyz ny + Tzz nz
For reference, suppose that a stress vector acting on the surface shown in Fig. 4.18. Since n = ex ,
we have from (4.55)
Txx = tx > 0, and Txy = ty > 0 (4.56)
In other words, Txx is the magnitude of the surface force in the x direction per unit surface area of
which the unit normal vector is ex and Txy is the magnitude of the surface force in the y direction
per unit surface area of which the unit normal vector is ex .
More generally,

∆Fj i − the direction of n
Tij = lim ,
∆Ai →0 ∆Ai j − the direction of t (or ∆F)
(4.57)
Tij > 0 if both of n and t are either in positive or in negative directions.
Tij < 0 if either of n and t is in the positive direction.
58 CHAPTER 4. CONSERVATION OF LINEAR MOMENTUM AND ITS DERIVATIVES

n
y t
tyey

x txex

Figure 4.18: The stress vector acting on a point. (한점에 작용하고 있는 단위면적당 표면힘)

Figure 4.19 shows positive stress components acting at a point on the surfaces of a differential control
volume. When n and t are in the same direction regardless of their signs, it is call the normal stress

Tyy

y Tyz Tyx
Txy
Tzy
Txx
Tzx
Txz
Tzz

z
Figure 4.19: Positive stress components acting at a point.

and in the different direction the shear stress:


Tii = normal stress, and Tij (i ̸= j) = shear stress (4.58)
At this point it should be noticed that the stress tensor is symmetric, i. e.,
T = TT , or Tij = Tji (4.59)
It can be proven from the conservation of angular momentum even though it is beyond undergraduates’
ability. In addition, it is common to speak the stress acting at a point of fluid surface in terms
of the pressure portion and the viscous portion as
   
Txx Txy Txz −P + τxx τxy τxz
T = −P I + τ ⇐⇒ Tyx Tyy Tyz  =  τyx −P + τyy τyz  (4.60)
Tzx Tzy Tzz τzx τzy −P + τzz
where P is the thermodynamic pressure and τ is the viscous portion of the stress tensor. The ther-
modynamic pressure for a static fluid (v = 0 and thus τ = 0) is equal to the average normal stress,
that is
1
P = − (Txx + Tyy + Tzz ) (4.61)
3
The shear stresses are the same as the viscous portions of the stress.
Tij = τij , if i ̸= j (4.62)

4.5.2 Viscosity
As written in (4.60), the viscous portion of the stress has both normal and shear components of the
stress.  
τxx τxy τxz
τ = τyx τyy τyz  (4.63)
τzx τzy τzz
4.5. STRESS : SURFACE FORCE PER UNIT AREA AT A POINT 59

As previously mentioned in the surface force acting on fluid surface, when a shear force (viscous
force) is applied to the layers of a fluid, one of the fluid layer slides against the other layer and thus
forms a velocity gradient, more exactly shear rate (전단률)7 (Fig. 4.20). Similar to the voltage-current
system8 , the resistance of a fluid to the shear force (or sliding) is called viscosity defined as

shear stress τ F/A h g i


viscosity(µ) ≡ = = = Poise (4.64)
shear rate γ̇ ∆v/∆h cm · s

For a given shear force, thicker viscosity(=larger resistance) leads to smaller velocity gradient.

F/A F/A

Δv
Fluid Δh

Figure 4.20: The concept of viscosity.

Tar and honey (approximately 10,000 cP) as highly viscous; air and water are low viscous. In
Table 4.2, the viscosities of water and air are shown with respect to temperature.

Table 4.2: Viscosity of water and air at different temperatures

20 ◦ C 60 ◦ C 100 ◦ C
Water (cP = mPa·s) 1 0.46 0.28
Air (cP) 0.018 0.02 0.021

A simple theory for noninteracting gas molecules says that the viscosities of the gases are

(M T )1/2
µ = 0.00267 (4.65)
σ2
where µ is in cP, M is molecular weight, T is absolute temperature, and σ is the molecular diameter
in Å. The viscosities of liquids are much greater than those of gas at the same temperature. A good
approximation for the viscosities of liquid below the normal boiling point is

B
ln µ = A + (4.66)
T
As the temperature increases, the viscosity of gas increases due to increase in the molecular inter-
actions. However, the viscosity of liquid is highly dependent of cohesion between molecules and thus
decreases as the temperature increases. In Fig. 4.21, the viscosities of some gases and liquids are shown
with respect to temperature.
Sometimes, we use the other terminology of viscosity, the kinematic9 viscosity defined as

m2
 
µ
ν≡ (4.67)
ρ s

It is like the diffusion coefficient in mass transfer. As the diffusion coefficient increases, the amount of
mass transfer increases. The rate of momentum transfer can be determined by the kinematic viscosity
(ν), not viscosity (µ). More details are in Sect. 5.1.
7 Shear rate is defined as the flow velocity gradient in the direction perpendicular to the flow direction. γ̇ = ∇v+∇vT

where γ̇ is called the rate of deformation tensor.


8 The electric potential difference (voltage) in a conducting material leads to the current flow. The R in Ohm’s law

is the resistance of the conducting material to the voltage, R = V /I.


9 Kinematics is the branch of mechanics that focuses on the description of motion, without concern for the associated

forces.
60 CHAPTER 4. CONSERVATION OF LINEAR MOMENTUM AND ITS DERIVATIVES

Figure 4.21: Viscosity-temperature variation for some liquids and gases

4.5.3 Types of Fluids (Newtonian and non-Newtonian)


For water or air, the velocity gradient (more exactly shear rate) linearly increases with increasing a
shear force, indicating that its viscosity(= the slope of the shear rate-shear stress line) is independent
of the given shear force, as shown in Fig. 4.22. Such fluid is called Newtonian fluid. As a matter
of fact, most of gases and liquids with molecular weights less than 103 , and suspensions of non-
aggregating particles are newtonian except extremely viscous liquids. On the other hand, some of
fluids such as polymer melts and concentrated suspension of particles are deviated from the linearity.
Besides, some liquids like sewage sludge do not flow until the yield stress is attained and flow linearly.
These are called non-Newtonian fluids.

Bingham
F/A plastic Real plastic
Shear Shear
stress thinning
τ Newtonian
F/A F/A
Yield μ Shear
Δv
stress thickening
Fluid Δh

Shear rate γ (Δv/Δh)

Figure 4.22: Shear stress versus shear rate for newtonian and non-newtonian fluids.

For the non-Newtonian fluids, experimental data show nonlinear relationships between the shear
force and the velocity gradient, as shown in Fig. 4.22. In order words, their viscosities vary with
the shear force (or the velocity gradient because an increase in the shear force increases the velocity
4.5. STRESS : SURFACE FORCE PER UNIT AREA AT A POINT 61

gradient). A material that obeys


n−1
dv
µ = K = K γ̇ n−1 (4.68)
dh
is called a power-law fluid. Here K and n are positive constants that can be determined by fitting
F/A as a function of ∆v/∆h in (4.64). These values are specific to a given material and temperature.
In polymer solutions, they both depend on concentration. A material A fluid with n < 1, such that
µ decreases with increasing the shear force (and thus the velocity gradient), is referred to as shear-
thinning. For polymer solutions, n often ranges from 0.2 to 0.8. In the rarer situations where n > 1,
such that µ increases with increasing the shear force, it is shear-thickening. The n value for the
Newtonian fluid is 1, i.e., independent of the shear force. Equation (4.68) implies that µ → ∞ as
dv/dh → 0 for a shear-thinning fluid. In reality, µ tends to be constant so that it behaves like a
Newtonian fluid at very low shear rates.
Another class of materials flows only when the shear stress (force per unit area) exceeds a certain
threshold, called the yield stress τ0 . One way to describe its viscosity is


(
for τ < τ0
µ= τ0 (4.69)
µ0 + for τ ≥ τ0
dv/dh

A material that obeys Eq. (4.69) is called a Bingham fluid or Bingham plastic. The infinite viscosity
for τ < τ0 means simply that the material behaves as a rigid solid when subjected to insufficient
shear. If dv/dh greatly exceeds τ0 /µ0 , the fluid becomes Newtonian with µ = µ0 . House paint and
various foods behave much like this.
In summary, the shear stress for a fluid, which can be described by (4.68) and (4.69), can be
expressed as n−1
dv dv
τ (= F/A) = τ0 + K = τ0 + K γ̇ n (4.70)
dh dh
| {z }
µ

• Newtonian (τ0 = 0 and n = 1): Most of gases and liquids with molecular weights less than 103 ,
and suspensions of non-aggregating particles
• Shear-thinning (τ0 = 0 and n < 1): lava, blood, polymer melt (see Oobleck2008.wmv at Yotube)
• Shear-thickening (τ0 = 0 and n > 1): cornstarch + water mixture
• Bingham plastic (τ0 ̸= 0 and n ≤ 1): at low stress it is like a rigid body, at high stress it flows
like a viscous fluid. ex, slurry, toothpastes, ketchup, paint

Ex. 21. The functional relationship between stress and shear rate of a fluid is experimentally de-
termined as shown below. Plot shear stress vs. shear rate and finally determine a specific rheological
model for this fluid.
Answer: Bingham plastic, τ = 14.5 + 0.2γ̇

Shear stress (N/m2 ) Shear rate (1/s) Shear stress (N/m2 ) Shear rate (1/s)
14.54 0.2 16.52 10.1
14.56 0.3 18.52 20.1
14.6 0.5 22.52 40.1
14.66 0.8 26.52 60.1
14.72 1.1 30.52 80.1
14.92 2.1 46.52 160.1
15.32 4.1 66.52 260.1
15.52 5.1 86.52 360.1
62 CHAPTER 4. CONSERVATION OF LINEAR MOMENTUM AND ITS DERIVATIVES

Ex. 22. Some of non-Newtonian fluids do not follow the power-law given in (4.68). Other models
are available for a shear-thinning fluid such as polymer melt.
µ − µ∞ 1
Cross model : = , Sisko model : µ = µ∞ + K γ̇ n−1 (4.71)
µ0 − µ∞ 1 + (λγ̇)m
Viscosity of a polymer solution is measured over a wide range of shear rate as shown below. Plot shear
stress versus shear rate. Use Cross, Power law, and Sisko viscosity models to fit the viscosity-shear
rate relationship.
Viscosity (Pa·s) Shear rate (1/s) Viscosity (Pa·s) Shear rate (1/s)
0.0000012 60000 0.07 80
0.000004 70000 0.2 30
0.000008 30000 0.8 9
0.00002 28000 2.3 3.5
0.00012 11000 8 1
0.00031 7000 20 0.75
0.0008 3500 100 0.2
0.0013 1300 170 0.105
0.0036 900 800 0.11
0.008 350 1600 0.095
0.02 180 7000 0.105

65000 − 0.1
Cross : µ = 0.1 + , Power-law : µ = 8γ̇ −0.84 Sisko : µ = 0.1 + 8γ̇ −0.84 (4.72)
1 + (50000γ̇)0.84

Viscous Portion of Stress for a Newtonian Fluid


Until now, we have discussed the shear stress including viscosity in the one-dimension. More generally,
in the three-dimension the viscous portion stress for a Newtonian fluid is expressed as
 
2
τ = µ ∇v + ∇vT − (∇ · v)I (4.73)
3
where µ is independent of the shear stress (thus the velocity gradient or shear rate). If the Newtonian
fluid is incompressible or flows in an incompressible motion, the stress is further reduced to

τ = µ ∇v + ∇vT
 
(4.74)
4.6. EQUATION OF MOTION 63

It is clear that the viscous stress tensor is symmetric. The components of viscous stress for an incom-
pressible Newtonian fluid becomes
    
∂vx ∂vx ∂vy ∂vx ∂vz
2µ µ + µ +
   ∂x ∂y ∂x  ∂z ∂x 
τxx τxy τxz    
τyx
 ∂vx ∂vy ∂vy ∂vy ∂vz 
τyy τyz = µ
  + 2µ µ +  (4.75)
τzx τzy τzz   ∂y ∂x   ∂y  ∂z ∂y 
 ∂vx ∂vz ∂vy ∂vz ∂vz 
µ + µ + 2µ
∂z ∂x ∂z ∂y ∂z

in the rectangular cartesian coordinate or


    
∂vr vθ 1 ∂vr ∂vθ ∂vr ∂vz
2µ µ − + + µ +
   ∂r r r ∂θ ∂r  ∂z ∂r 
τrr τrθ τrz     
 vθ 1 ∂vr ∂vθ vr 1 ∂vθ ∂vθ ∂vz 
τθr τθθ τθz = µ − +
  + 2µ + µ + 
r r ∂θ ∂r  r r ∂θ  ∂z ∂θ 
τzr τzθ τzz  
 ∂vr ∂vz ∂vθ ∂vz ∂vz 
µ + µ + 2µ
∂z ∂r ∂z ∂θ ∂z
(4.76)
in the cylindrical coordinates or
     
∂vr vθ 1 ∂vr ∂vθ 1 ∂vr ∂vϕ vϕ
2µ µ − + + µ + −

  ∂r r r ∂θ ∂r  r sin θ ∂ϕ ∂r r 
v 1 ∂v ∂v

v 1 ∂v

1 ∂v sin θ ∂ h v i

 µ − + θ r θ r θ θ ϕ 
+ 2µ + µ + 

  r r ∂θ ∂r   r r ∂θ  r sin θ ∂ϕ
 r ∂θ sin θ   
 1 ∂vr ∂vϕ vϕ 1 ∂vθ sin θ ∂ h vϕ i 1 ∂vϕ vr + vθ cot θ 
µ + − µ + 2µ +
r sin θ ∂ϕ ∂r r r sin θ ∂ϕ r ∂θ sin θ r sin θ ∂ϕ r
(4.77)
in the spherical coordinate.

4.6 Equation of Motion


As the differential form of mass balance, the differential form of linear momentum balance (the
differential linear momentum balance or the equation of motion) can be also derived by applying
a linear momentum balance over a differential element.
Consider a linear momentum balance over a differential element, a special type of control volume.

(x+∆x, y+∆y, z+∆z)

(x, y, z)

Figure 4.23: Linear momentum flux through a differential control volume

The linear momentum balance(or the conservation of linear momentum) within the differential
64 CHAPTER 4. CONSERVATION OF LINEAR MOMENTUM AND ITS DERIVATIVES

element says that


   
Rate of linear momentum Rate of linear momentum
=
accumulation within the DE into the DE
    (4.78)
Rate of linear momentum Sum of forces acting
− +
out of the DE on a differential element(DE)

∂P X
= Ṗin − Ṗout + F (4.79)
∂t
• Linear momentum in the differential element shown in Fig. 4.23 is

P = mass × velocity = ρv∆x∆y∆z (4.80)

and the time rate of change of linear momentum in the differential element is


(ρv∆x∆y∆z) (4.81)
∂t

• Linear momentum rate entering and leaving the element


The linear momentum rate entering the differential element through the surface at x is

Ṗin = ṁin v = (ρvvx )x ∆y∆z (4.82)

and the linear momentum rate leaving the differential element through the surface at x + ∆x is

Ṗout = ṁout v = (ρvvx )x+∆x ∆y∆z (4.83)

Thus the net linear momentum rate out of the differential element in the x direction is written
as
[(ρvvx )x+∆x − (ρvvx )x ] ∆y∆z (4.84)
Similarly we can get the net linear momentum rates in the y and z directions:

[(ρvvy )y+∆y − (ρvvy )y ] ∆x∆z, and [(ρvvz )z+∆z − (ρvvz )z ] ∆x∆y (4.85)

Finally, the net rate of linear momentum from the differential element becomes

[(ρvvx )x+∆x − (ρvvx )x ] ∆y∆z + [(ρvvy )y+∆y − (ρvvy )y ] ∆x∆z + [(ρvvz )z+∆z − (ρvvz )z ] ∆x∆y
(4.86)

• Sum of forces acting on the differential element

1. Body force acting on the differential element is

ρb∆x∆y∆z (4.87)

where b is the body force per mass and thus ρb is the body force per unit volume. In case
there is the gravitational force only as a body force, b = g.
2. Surface forces acting on the surface of the differential element.
From Fig. 4.24, we can see that sum of the surface forces in the x direction is

(Txx |x+∆x − Txx |x )∆y∆z + (Tyx |y+∆y − Tyx |y )∆x∆z + (Tzx |z+∆z − Tzx |z )∆x∆y (4.88)

The surface forces in the y and z directions can be obtained in similar forms:

(Txy |x+∆x − Txy |x )∆y∆z + (Tyy |y+∆y − Tyy |y )∆x∆z + (Tzy |z+∆z − Tzy |z )∆x∆y
(4.89)
(Txz |x+∆x − Txz |x )∆y∆z + (Tyz |y+∆y − Tyz |y )∆x∆z + (Tzz |z+∆z − Tzz |z )∆x∆y
4.6. EQUATION OF MOTION 65

y Tyx y+∆y

Tzx z
∆y
Txx x
Txx x+∆x
Tzx z+∆z
Tyx y ∆z
∆x

z
Figure 4.24: Surface forces in the x direction acting on the surface of a differential element.

Using the linear momentum balance (4.78) and (4.81) through (4.89), we finally have


(ρv∆x∆y∆z) + [(ρvvx )x+∆x − (ρvvx )x ] ∆y∆z
∂t X
+ [(ρvvy )y+∆y − (ρvvy )y ] ∆x∆z + [(ρvvz )z+∆z − (ρvvz )z ] ∆x∆y = F (4.90)

Since the volume does not change with time, dividing (4.90) by ∆x∆y∆z gives

∂ (ρvvx )x+∆x − (ρvvx )x (ρvvy )y+∆y − (ρvvy )y (ρvvz )z+∆z − (ρvvz )z


(ρv) + + +
∂t ∆x ∆y ∆z
 
Txx |x+∆x − Txx |x Tyx |y+∆y − Tyx |y Tzx |z+∆z − Tzx |z
= + + ex
∆x ∆y ∆z
  (4.91)
Txy |x+∆x − Txy |x Tyy |y+∆y − Tyy |y Tzy |z+∆z − Tzy |z
+ + + ey
∆x ∆y ∆z
 
Txz |x+∆x − Txz |x Tyz |y+∆y − Tyz |y Tzz |z+∆z − Tzz |z
+ + + ez + ρb
∆x ∆y ∆z

and take the limit as ∆x → 0, ∆y → 0, and ∆z → 0, this becomes


 
∂ ∂ ∂ ∂ ∂Txx ∂Tyx ∂Tzx
(ρv) + (ρvvx ) + (ρvvy ) + (ρvvz ) = + + ex
∂t ∂x ∂y ∂z ∂x ∂y ∂z
   
∂Txy ∂Tyy ∂Tzy ∂Txz ∂Tyz ∂Tzz
+ + + ey + + + ez + ρb
∂x ∂y ∂z ∂x ∂y ∂z

=⇒ (ρv) = −∇ · (ρvv)
∂t
| {z } | {z }
net influx of momentum per unit length in all direction
rate of momentum per volume accumulation at a point
=한 점에서 각 방향으로 단위길이당 순유입 momentum flux
=한 점에서 시간에 대한 단위부피당 운동량의 변화율
T
+ |∇ ·{zT } + ρb
|{z}
surface force per unit area per length in all direction body force per unit volume at a point
=한 점에서 작용하는 각 방향으로 단위길이당 표면힘/단위면적 =한 점에서 작용하는 단위부피당 body force
(4.92)

This is called the differential linear momentum balance or the equation of motion that should be
applied at any differential elements, i.e., points.
Since
 
∂ ∂ρ ∂v ∂v
(ρv) + ∇ · (ρvv) = v +ρ + v∇ · (ρv) + ∇v · (ρv) = ρ + ∇v · v (4.93)
∂t ∂t ∂t ∂t
66 CHAPTER 4. CONSERVATION OF LINEAR MOMENTUM AND ITS DERIVATIVES

from the differential mass balance (3.24) and the symmetry of stres (4.58), the equation of motion
becomes  
∂v
ρ + ∇v · v = ρb + ∇ · T (4.94)
∂t
It can be also expressed in terms of the material (or substantial) time derivative

Dv
ρ = ρb + ∇ · T (4.95)
Dt
from the definition of substantial time derivative (2.13).
From (4.60), the equation of motion (4.94) becomes
 
∂v
ρ + ∇v · v = −∇P + ∇ · τ + ρb (4.96)
∂t

because
∇ · (−P I) = −∇P · I − P (∇ · I) = −∇P (4.97)

4.6.1 The Euler Equation for an Inviscid Flow (∇ · τ ≈ 0)


Clearly, all fluids are viscous, but in certain situations and under certain conditions, a fluid may be
considered ideal or inviscid (µ ≈ 0). In the inviscid flow (∇ · τ ≈ 0), the viscous forces are negligible.
The equation of motion (4.96) is further simplified to
 
∂v
ρ + ∇v · v = −∇P + ρb (4.98)
∂t

which is known as the Euler equation for inviscid flows. This Euler equation has been found to be
useful for describing the flows of low-viscosity (µ ≈ 0) at Re≫ 1 (see Sect. 5.2.1). Of course, we
know that this equation will be inadequate in the neighborhood of solid surface, so called boundary-
layer (see Sect. 5.2.3). The subject of inviscid flow has particular application in aerodynamics, which
handles mainly the external flows having a very high Re number.

4.6.2 The Navier-Stokes Equation for an Incompressible Flow of a New-


tonian Fluid
If a fluid
 is Newtonian
 and moves in an incompressible flow, the viscous stress tensor is expressed as
τ = µ ∇v + (∇v)T and thus the equation of motion (4.96) reduces to
 
∂v
ρ + ∇v · v = −∇P + µ∇2 v + ρb (4.99)
∂t

This is called the Navier-Stokes equations and is the differential form of Newton’s second law of motion
for an incompressible Newtonian fluid. In deriving (4.96), we used that

∇ · (∇v) = ∇2 v and
T
(4.100)
∇ · (∇v ) = ∇(∇ · v) = 0 ∵ ∇ · v = 0 for an incompressible flow

Creeping flow (viscous force ≫ intertia)


When the acceleration terms in the Navier-Stokes equation are neglected, it reduces to

0 = −∇P + µ∇2 v + ρb (4.101)

which is called the Stokes flow equation or the steady-state creeping flow equation.
4.7. MECHANICAL ENERGY BALANCE 67

Irrotational flow (∇ × v = 0 → ∇ · τ = 0)
Using vector calculation, we can show
1
∇v · v = ∇ (v · v) −[v × (∇ × v)] (4.102)
2 | {z }
v2

and
∇2 v = ∇(∇ · v) −∇ × (∇ × v) (4.103)
| {z }
0, incompressible
Thus, the viscous force acting on a fluid becomes
∇ · τ = −µ[∇ × (∇ × v)] (4.104)
Indeed, the curl of velocity is twice the vorticity vector, which is a measure of rotational motion (see
Appendix B). If a fluid flow is irrotational, the curl of velocity becomes zero:
∇× v=0 (4.105)
The physical meaning of curl of velocity is well explained in movie clips (https://www.khanacademy.org/
math/multivariable-calculus/multivariable-derivatives/curl-grant-videos/v/2d-curl-intuition?modal=1).
Therefore, the flow of a fluid is irrotational, the viscous effect is negligible: ∇ · τ = 0.

∇×v

Figure 4.25: Curl of fluid flows. The positive curl corresponds to counter-clockwise rotation, whereas
the negative curl corresponds to clockwise rotation. The zero curl indicates no rotation flow. (see
Khan Academy)

Overall, when the flow of a Newtonian fluid is incompressible and irrotational, the differential form
of linear momentum balance (4.99) reduces to
v2
 
∂v
ρ +∇ = −∇P + ρb (4.106)
∂t 2
This is further simplified for a steady-state flow to
 2 
ρv
∇ + P = ρb (4.107)
2
It is nothing but the differential form of linear momentum balance for the steady-state incompressible
and irrotational flow of a Newtonian fluid.

Potential flow (v = −∇ϕ → ∇ × v = 0 → ∇ · τ = 0)


When the velocity distribution is given by an equation of the form:
v = −∇ϕ (4.108)
It is referred to a potential flow. It is easy to prove that a type of potential flows is irrotational, i.e.,
∇ × v = ∇ × ∇ϕ = 0 (4.109)
Therefore, the viscous effect is negligible in the potential flow so that no dissipation of mechanical
energy into heat occurs.
68 CHAPTER 4. CONSERVATION OF LINEAR MOMENTUM AND ITS DERIVATIVES

4.7 Mechanical Energy Balance (NOT Conserved): v·2nd Law


In general form, the first law of thermodynamics shows how the total energy of an open system is
affected by the flow of internal, kinetic, and potential energy in or out, by heat gain or loss, and by work
done on the system by its surroundings. The temperature variations that affect the internal energy
are crucial for heat transfer, but less important for fluid dynamics. Only kinetic and gravitational
potential energies are considered. In science, mechanical energy is the sum of them. It is the energy
associated with the motion and position of an object. This mechanical energy is NOT conserved in
all real systems since it is converted to heat by non-conservative forces such as a viscous force. With
conservative forces10 only, the mechanical energy can be conserved. Thus the mechanical energy is
not conserved in a flow system having friction.
Taking a dot product of a velocity vector v, with the conservation of linear momentum balance,
we have
d(mv) X
v· =v· F (4.110)
dt
Since
dv 1 d(v · v) 1 dv 2
v· = = (4.111)
dt 2 dt 2 dt
Equation (4.110) becomes  
d 1 2 dEK
mv = = v · (Fs + Fb ) (4.112)
dt 2 dt
where EK is called the kinetic energy.
If we can find a potential Φ̂ satisfying11

b = −∇Φ̂ (4.113)

Then  
dΦ̂ ∂ Φ̂
v · Fb = v · m(−∇Φ̂) = −m(∇Φ̂ · v) = −m − (4.114)
dt ∂t
in view of (2.13). If the potential is independent of the time, ∂ Φ̂/∂t = 0 (it is true for the gravitational
field for systems; Φ̂ = gy), this becomes

dΦ̂ d
v · Fb = −m = − (mΦ̂) (4.115)
dt dt
from the conservation of mass dm/dt = 0. Finally (4.112) can be rewritten as
 
d 1 2 d
mv + mΦ̂ = (EK + EP ) = v · Fs , if b = −∇Φ̂ (4.116)
dt 2 dt

where EP = mΦ̂ is called the potential energy. In the gravitational field only,

b = g = −gey =⇒ Φ̂ = gy (4.117)

Let’s consider a falling object in a fluid. It is assumed that all the surface forces acting on the
object caused by the fluid are negligible, compared to the gravitational body force. Thus, (4.116)
becomes
 
d 1 2 d ∂
mv + mgh = (EK + EP ) = (EK + EP ) + v · ∇(EK + EP ) = 0 (4.118)
dt 2 dt ∂t

This is the conservation of mechanical energy for a falling object in an inviscid fluid.
10 a force with the property that the work done in moving a particle between two points is independent of the path

taken. In order words, a force acting on an object is a function of position olny.


11 Body force is representable by a potential Φ̂ because it is a kind of conservative forces. For example, if the gravita-

tional force is the only body force, then b = g and Φ̂ can be expressed as gy so that g = −∇Φ̂ = −gey .
4.8. EQUATION OF MECHANICAL ENERGY 69

h
y

Figure 4.26: Conservation of mechanical energy for a falling object. Here, the drag force (air resistance)
on the object is assumed to be negligible.

The directional derivative of f , the rate of change of f in the direction given by a unit vector u is
defined as
df
= u · ∇f (4.119)
dh
where h is a magnitude in the direction of u. Thus,
v d
· ∇(EK + EP ) = (EK + EP ) (4.120)
v dh
With neglecting the time derivatives, we get using the definition of the directional derivative
d 1
(EK + EP ) = 0 ⇐⇒ mv 2 + mgh = constant (4.121)
dh 2
where h is the magnitude along the direction of the velocity, the falling direction of a body:

4.8 Equation of Mechanical Energy


Taking a dot product of a velocity vector v with the equation of motion (4.96), we have
   
∂v
v· ρ + v · ∇v = −∇P + ∇ · τ + ρb (4.122)
∂t

With the assumptions that b = −∇Φ̂, neglecting the time derivatives (steady-state), and ρ constant
(incompressible), this becomes using (4.102)
ρv 2
 
v· ∇ + ∇P + ∇ρΦ̂ = v · [v × (∇ × v)] + v · (∇ · τ ) (4.123)
2
Using the directional derivative (4.119) and (4.104), we get
 
d 1 v v
P + ρv 2 + ρΦ̂ = · [v × (∇ × v)] − · {µ[∇ × (∇ × v)]} (4.124)
ds 2 v v
where s is the magnitude along the velocity direction of a particle, a streamline. It is called the
differential mechanical energy balance or the equation of mechanical energy that should be applied at
any differential elements, i.e., points on the same streamline. If a fluid flow is also irrotational and
thus no viscous effect occurs, we finally have
1
P + ρv 2 + ρΦ̂ = constant, for a steady-state incompressible irrotational flow (4.125)
2
which is called the (streamline) Bernoulli equation, a cornerstone of classical fluid dynamics, applied
at a point.12 For a steady-state incompressible irrotational(or inviscid) flow, the total
12 Daniel Bernoulli (1700-1782), Swiss mathematician, was one of the early researchers in fluid dynamics. He showed

that as the velocity of fluid increases, the pressure decreases, a statement known as the Bernoulli principle. He was a
friend of Leonhard Euler (1707-1783), who formulated the continuity and momentum equations for an inviscid fluid.
70 CHAPTER 4. CONSERVATION OF LINEAR MOMENTUM AND ITS DERIVATIVES

energy (pressure work+kinetic energy+potential energy) remains constant along any


streamline. It describes the reversible interconversion of kinetic energy and gravitational potential
energy, as influenced by pressure-volume work. Although the sum is constant at any points along a
streamline, it may vary from one streamline to another.

Figure 4.27: Streamline: an imaginary line in a fluid such that the tangent at any point indicates
the direction of the velocity of a particle of the fluid at that point. In a steady flow, streamlines
corresponds to path lines of particles. (유동에서 입자의 속도벡터에 접하는 선)

4.9 Angular Momentum Balance: r × 2nd Law


Newton’s second law of motion can be applicable to a rotational motion as well as a straight motion
of an object. However, we can’t directly apply the Newton’s 2nd law to the rotational motion.

Figure 4.28: A rotational motion of an object.

Take a cross product of a position vector, r, with the Newton’s 2nd laws, we have

X dv d dP d dr
r× F=r× m =r× (mv) = r × = (r × P) − × P
dt dt dt dt dt
X d dr
=⇒ r × F = (r × P) ∵ × P = v × mv = 0 (4.126)
dt dt
X dH
=⇒ M=
dt

where M is the moment (torque) and H is the angular momentum(moment of momentum), which
measures how much the linear momentum is rotating around a certain point called the origin. This is
called the conservation of angular momentum. Torque is usually used to describe a rotational force of
a shaft (the tendency of a force to rotate an object about an axis), for example a turning screw-driver
(a measure of the turning force on an object).
4.9. ANGULAR MOMENTUM BALANCE: R × 2ND LAW 71

Similar to the conservation of linear momentum, it says in fluid mechanics that


∂H X
= Ḣin − Ḣout + M
∂t
|{z}
|{z} | {z } | {z }
rate of angular momentum rate of angular momentum sum of moments acting
rate of angular momentum
into control volume out of control volume
accumulation within control volume on control volume
(4.127)
By taking a cross product of a position (constant) vector, r with the differential linear momentum
balance (4.95), we have
D
ρ (r × v) = r × (∇ · T) + r × ρb (4.128)
Dt
This is called the differential angular momentum balance or the equation of angular momentum.

Summary
72 CHAPTER 4. CONSERVATION OF LINEAR MOMENTUM AND ITS DERIVATIVES

운동량 (Momentum)
운동량: 움직이는 물체가 지니고 있는 power.

Ø 선형운동량(P) = mv Ø 회전운동량(H) = r´mv= r´P

m
v
mv r´mv
mv
r

Ø 운동량 보존법칙 Ø 각(회전)운동량 보존법칙

SF = dP/dt r´SF = SM = dH/dt

유체의 운동량을 변화시키기 위해 유체의 각운동량을 변화시키기 위해서는 외부


서는 외부에서 힘을 가하여야 함 에서 모멘트(거리´힘)을 가하여야 함

화학공학부 울산대학교
SCHOOL OF CHEMICAL ENGINEERING UNIVERSITY OF ULSAN

운동량보존 법칙
!"
= Σ#, 물질입자로 이루어진 body내 선형운동량의 시간변화율은 body에
!#
작용하는 힘들의 합과 같다.
!"
̇ − #&'#
= #$% ̇ + Σ&, 고정된 공간인 control volume내 시간에 대한 선형운동량
!#

변화는 순 선형운동량 유입속도와 CV에 작용하는 힘들의 합과 같다.

!$
= Σ$, 물질입자로 이루어진 body내 각운동량의 시간변화율은 body에
!#
작용하는 모멘트의 합과 같다. (' = ( × *, + = ( × &)
!(
= ,̇ $% − ,̇ &'# + Σ+, 고정된 공간인 control volume내 시간에 대한 각운동량
!#

변화는 순 각운동량유입속도와 CV에 작용하는 모멘트들의 합과 동일하다.

화학공학부 울산대학교
SCHOOL OF CHEMICAL ENGINEERING UNIVERSITY OF ULSAN
4.9. ANGULAR MOMENTUM BALANCE: R × 2ND LAW 73

운동량 보존법칙 예
!"
!#
= #̇ $% − #̇ &'# + Σ&

• Control volume을 정함.


• CV 및 CS에 작용하는 힘과 각 방향 운동량
• CV의 v는 0이므로 운동량의 시간변화량은 0임.
• ̇
x방향 : "!",$ ̇
= 12 #̇ 인데 "%&',$ = 0 이므로
–x 방향으로 12 #̇ 힘이 가해져야 함
• ̇
y방향: "!",( ̇
= 0 임. 위와 아래가 동일한 질량 및 속력으로 나가므로 "%&',( 의
합 또한 0임. 따라서 y 방향으로 힘은 필요 없음.
• 각 방향에서 CS에 작용하는 압력힘은 서로 상쇄됨에 따라서 남은 힘은 –x
방향으로 주어지 F 밖에 없음.

화학공학부 울산대학교
SCHOOL OF CHEMICAL ENGINEERING UNIVERSITY OF ULSAN

힘 (Force)
Interfacial
Body force Surface force tension force

계면에 작용
물체의 Body(체적)에 작용한 힘 물체의 표면에 작용하는 힘
하는 힘

물리적인 접촉 없이 작용하는 힘 물리적인 접촉이 필요한 힘 표면장력

압력 + 마찰력(점성력)
중력, 정전기력 = 응력(stress)

점성력 : 유체 층들 사이 미끄러짐
에 대항하는 힘 표면장력 (g)
m q1

압력 (P)
중력
정전기력 점성력 (t)

M q2

화학공학부 울산대학교
SCHOOL OF CHEMICAL ENGINEERING UNIVERSITY OF ULSAN
74 CHAPTER 4. CONSERVATION OF LINEAR MOMENTUM AND ITS DERIVATIVES

등속운동하는 유체에 작용하는 힘


v=일정 (운동량 P=mv=일정) Þ SF =dP/dt = 0 P3
S(단면적) t1

W
h(높이) P1 P2
r(밀도)

L(너비) t2
SF= 중력 + 압력 + 마찰력
P4
W = 유체의 무게에 의한 중력 =mg=rShg
FP1 = 좌측면과 접하고 있는 유체가 좌측면의 유체에 가한 압력힘 = P1 hL
FP2 = 우측면과 접하고 있는 유체가 우측면의 유체에 가한 압력힘 = P2 hL
FP3 = 용기의 윗면이 윗면의 유체에 가한 압력힘 = P3 S
FP4 = 용기의 아랫면이 아랫면의 유체에 가한 압력힘 = P4 S
Ft1 = 용기의 윗면이 윗면의 유체에 가한 마찰힘 = t1 S
Ft2 = 용기의 아랫면이 아랫면의 유체에 가한 마찰힘 = t2 S

수평방향의 힘의 합: FP1 – FP2 - Ft1 – Ft2=(P1–P2)hL–(t1+t2)S=0 Þ P1 = P2 + (t1 + t2)S/hL

수직방향의 힘의 합: FP4 – FP3 – W = P4 S – P3 S - rShg = 0 Þ P4 = P3 + rgh


화학공학부 울산대학교
SCHOOL OF CHEMICAL ENGINEERING UNIVERSITY OF ULSAN

표면장력
• 계면분자들은 벌크분자들과 달리 분자간 인력이 균형을 이
Air
루지 않아 순힘이 작용하며, 이를 계면장력힘이라 함.
Interface

• 힘: 계면의 윤곽선을 늘리는데 필요한 단위길이당 장력힘


Air Liquid
(N/m)이다. 계면장력힘은 표면장력과 길이의 곱임 (Fg = gL)
Interface
Bulk
• 에너지: 계면을 늘리는데 필요한 단위면적당 에너지 (J/m2)
이다. 따라서 interfacial energy라고도 불린다. Liquid
Air
• 표면분자들의 에너지가 벌크분자보다
Interface 크므로 에너지를
Bulk 최

소화하기 위하여 표면을 최소화한다.


• Young-Laplace 식: 유체에 잠긴 기포의 내부압력이 Liquid
외부압력
보다 표면장력이 견딜 만큼 크다. 구형 기포
Bulk
Pin-Pout = 2g/R
• 모세관 상승은 모세관-물간 인력이 물 분자간 인력보다 크
기 때문. 중심 부분은 표면을 최소화하기 위하여 올라간다.
• 모세관 하강은 모세관-수은의 인력이 수은분자간 인력보다
작기 때문. 중심 부분은 표면을 최소화하기 위하여 내려감
화학공학부 울산대학교
SCHOOL OF CHEMICAL ENGINEERING UNIVERSITY OF ULSAN
4.9. ANGULAR MOMENTUM BALANCE: R × 2ND LAW 75

응력 (Stress)
• 한점에 작용하는 단위면적당 표면의 힘 (압력+점성력)
• 응력벡터는 t로 표시하며 힘의 방향만 있음.
t = T・n
• 응력텐서 (일반적으로 응력이라 함)는 T로 표시하고
힘 및 단면의 방향 정보가 있음.
Tij i: n의 방향, j: 표면힘(t) 방향
Tii: 수직응력, Tij (i ≠ j) 전단응력
• Txy : x축에 수직인 면에 작용하는 y방향의
단위면적당 표면힘.
• 응력은 대칭임 (Tij=Tji)
• T = -P I + t (응력 = 압력 +점성력)
• 압력은 수직응력만 존재하고 점성응력은 수직응력과 전단응력 모두 존재함.
따라서 전단응력은 점성력에 의해서만 존재함.
화학공학부 울산대학교
SCHOOL OF CHEMICAL ENGINEERING UNIVERSITY OF ULSAN

점도(Viscosity)
유체의 한 층에 shear force(전단응력)를 가하면 유체의 층이 다른 층에서 미끄러지
며, 속도구배를 형성한다. 유체의 점도란 이와 같은 shear force (or sliding)에 대항
하는 저항을 일컫는다. 이는 도선에 전위차를 가하여 전류가 흐르게 하는 것과 같은
원리로서, 전위차에 대한 저항을 도선의 저항이라 칭함 (; = ∆< ⁄=).

F/A F/A

Δv
Fluid Δh

shear force 4 ⁄5 g
점도 ! = = [ ] = Poise
velocity gradient ∆7⁄∆ℎ cm : s

• 때때로 동점도 (속도 or 운동량전달계수)를 사용하기도 함.

점도(!)
동점도 > =
!
[! Bs]
밀도(A)

화학공학부 울산대학교
SCHOOL OF CHEMICAL ENGINEERING UNIVERSITY OF ULSAN
76 CHAPTER 4. CONSERVATION OF LINEAR MOMENTUM AND ITS DERIVATIVES

점도와 온도

Ø 일반적으로 온도가 증가하면 액체의 경우 분자간


cohesion이 감소하여 점도는 감소한 반면에 기체는 분
자간상호작용의 증가로 인하여 점도가 증가한다.

Ø 상온에서 물의 점도(1 cP) > 공기의 점도 (0.018 cP)


Þ 동일한 속도기울기를 형성하기 위해서 물에 훨씬 큰
힘을 가해야 한다.

Ø 상온에서 물의 동점도 (0.01 cm2/s) < 공기 (0.15 cm2/s)


Þ 점성에 의한 운동량전달은 공기가 훨씬 빠르다.

Fig. 7.5 in Welty’s Book

화학공학부 울산대학교
SCHOOL OF CHEMICAL ENGINEERING UNIVERSITY OF ULSAN

점도에 따른 뉴튼 및 비뉴튼 유체
tic
두 평판 사이에 유체를 두고 힘을 가하 Bingham
p las
F/A plastic al
Re Shear thinning
면서 속도기울기를 측정함.
Newtonian
F
d" Shear
$
d# thickening
y 항복
! d& 응력
유체 = $
" d'
x

d"
Ø Newtonian – 점도가 주어진 전단응력에 따라 변하지 않는 일정한 유체
d#
(공기와 물과 같은 대부분의 기체 및 액체)

Ø Pseudo plastic (shear thinning) – 전단응력이 증가함에 따라 점도가 감소하는 유체


(페인트 수지, 혈액)

Ø Dilatant(shear thickening) – 전단응력이 증가함에 따라 점도가 증가하는 유체


(물과 전분의 혼합물)

Ø Bingham plastic - 일정한 힘(항복응력) 이상을 가하지 않으면 속도의 기울기가 생기지
않는 유체 (치약, 케찹, 등)
화학공학부 울산대학교
SCHOOL OF CHEMICAL ENGINEERING UNIVERSITY OF ULSAN
4.9. ANGULAR MOMENTUM BALANCE: R × 2ND LAW 77

점성 응력(Viscous stress)
F
y ! d(
& = $!" = %!" = & ∵ T = -P I + t
" d)
x

• 비압축성 뉴턴유체의 한점에 작용하는 점성 응력 (단위면적당 점성힘)

! = #(∇& + ∇&! )

• 점성력의 원인은 물질특성인 점도와 유체의 운동특성인 속도차이에 의함.


• 참고로 정지해 있는 유체에 대한 압력은 각 방향에 대한 수직응력의 평균
이라 할 수 있음.
화학공학부 울산대학교
SCHOOL OF CHEMICAL ENGINEERING UNIVERSITY OF ULSAN

운동방정식 및 비회전성 흐름
• 운동방정식은 한점에서 만족해야 할 운동량보존법칙(뉴튼 2법칙)이다.
• 뉴튼유체가 비압축성흐름을 보이면 운동방정식은 Navier-Stokes 식이 됨.

*+
) + + - .+ = −∇0 + #∇" + + )1
*,
- 어느 한점에서 시간에 대한 운동량의 변화량 (관성력)은 그 점에 작용하는
압력 + 점성력 + 중력의 합과 같다.
- Creeping flow : 속도가 매우 천천히 변하여 관성력을 무시할 수 있는 흐름
!"
(# !# =0), 이와 같은 흐름을 Stokes flow라고도 함.
- Irrotational flow: 유체입자들이 회전없이(속도의 curl이 0임) 흘러 점성력이
0이 되는 흐름
- Potential flow: 속도가 potential 함수 (+ = −∇2)로서 표현되어 비회전성 흐
름을 형성, 결국 점성력이 0 (마찰이 생기지 않는 흐름)이 됨.
- Inviscid flow: 모든 유체는 점도를 있지만 점도가 거의 없는 것처럼 흐름.
화학공학부 울산대학교
SCHOOL OF CHEMICAL ENGINEERING UNIVERSITY OF ULSAN
78 CHAPTER 4. CONSERVATION OF LINEAR MOMENTUM AND ITS DERIVATIVES

역학적에너지 방정식(베르누이 식)
• 베르누이식(equation of mechanical energy)은 속도와 운동방정식 (한점에 작
용하는 운동량보존법칙)의 dot product로 부터 유도됨.
• 베르누이식은 유체가 정상상태 비압축성 및 비점성 흐름을 보일 때 한 유선
상의 어느 점들에서도 총에너지 (압력일 + 운동에너지 + 위치포텐셜에너지)
는 항상 일정함.
#! $!" ## $#"
!! + + #! &ℎ! = !# + + ## &ℎ#
2 2

b
a

a
b

* 유선: 유체 내 상상의 line으로서 접선이 그 점에서 유체입자의 속도의 방향을 나타냄.


정상상태에서는 유선이 실제 입자들이 지나는 path line로 동일함.
화학공학부 울산대학교
SCHOOL OF CHEMICAL ENGINEERING UNIVERSITY OF ULSAN

질량, 운동량, 각운동량 비교


Mass Linear Momentum Angular Momentum
Definition m (= rV ) P = mv H = r ´ P = r ´ mv (= Iω)
m! (= r Av^ ) P! = m! v (= r Av^ v ) ! = r ´ P! = r ´ m! v (= r ´ r Av v )
H ^

Conservation dm dP dH
body =0 ma = å F Û = åF r ´ ma = r ´ å F Û = åM
dt dt dt
F M
¶m ¶P ¶H
min mout Pin Hin
¶t ¶t Pout ¶t Hout

¶m ¶P ¶H
control volume + mout - min = 0 + Pout - Pin = å F + Hout - Hin = å M
¶t ¶t ¶t
Integral m = òòò r dV P = mv = òòò r v dV H = r ´ P = òòò r ´ r v dV
Balance CV CV CV

mout - min = òò r ( v × n) dA Pout - Pin = òò r v( v × n) dA Hout - Hin = òò r ´ r v( v × n) dA


CS CS CS

å F = Fb + Fs
= òòò r b dV + òò t dA å M = r ´ å F = r ´ (F + F ) b s

= òòò r ´ r b dV + òò r ´ t dA
CV CS

CV CS
t º T×n

Differential dr dv d (r ´ v)
Balance + rÑ × v = 0 r = Ñ × T + rb r = r ´ (Ñ × T) + r ´ r b
dt dt dt
화학공학부 울산대학교
SCHOOL OF CHEMICAL ENGINEERING UNIVERSITY OF ULSAN
Chapter 5

Momentum Transfer and Various


Flows

In what follows, we will mostly use the differential mass (equation of continuity) and momentum
balances (equation of motion) to analyze fluid flows. These will give some information on every point
in a control volume.
The differential mass balance is
∂ρ Dρ
+ ∇ · (ρv) = 0, or + ρ∇ · v = 0 (5.1)
∂t Dt
and for incompressible fluids or flows it reduces to

∇·v =0 (5.2)

The equation of motion is


 
Dv ∂v
ρ = ρb + ∇ · T or ρ + ∇v · v = −∇P + ∇ · τ + ρb (5.3)
Dt ∂t

For the incompressible flow of a Newtonian fluid, it becomes the Navier-Stokes equation:
 
∂v
ρ + ∇v · v = −∇P + µ∇2 v + ρb (5.4)
∂t

5.1 Momentum Transfer by Velocity and Viscosity


As shown in Fig. 5.1, when a fluid in the left corner of the bottom is in motion, the fluid transfers its

Momentum transfer τ1
in the vertical direction
Δy
by viscosity Δx
τ2

fluid in motion
Momentum(=mv) transfer in the
at t=0
horizontal direction by motion v

Figure 5.1: Momentum transfer of a moving flud in the left corner of the bottom.

momentum to the neighborhood of the fluid in both horizontal and vertical directions. The momentum
transfer in the horizontal direction is dependent upon the velocity of the fluid, whereas the momentum

79
80 CHAPTER 5. MOMENTUM TRANSFER AND VARIOUS FLOWS

transfer in the vertical direction is due to the viscosity of the fluid, represented by the cohesion force.
In other words, fluid layers can slide each other and the viscosity is the resistance for the sliding,
leading to momentum transfer from one to the other layer.
The conservation of linear momentum over a fluid layer in Fig. 5.1 is
X d ∂
F= (mv) = (mv) + (ṁv)out − (ṁv)in (5.5)
dt ∂t | {z }
=0

For the x direction, this becomes


  
dvx dvx ∂
(τ1 − τ2 )∆x = (Tyx |y+∆y − Tyx |y )∆x = µ − µ ∆x = (ρ∆x∆yvx ) (5.6)
dy y+∆y,t
dy y,t
∂t

where pressure is assumed to be the same everywhere. It reduces to with ∆y → 0

∂vx µ ∂ 2 vx
= (5.7)
∂t ρ ∂y 2

Therefore the time rate of change of vx at a point is dependent of µ/ρ, not µ only. That is, the rate
of momentum transfer in the vertical direction is dependent upon kinematic viscosity ν = µ/ρ. The
viscosities of water and air are 1 cP and 0.018 cP, respectively, whereas the kinematic viscosities of
water and air are 1 × 10−6 m2 /s and 15 × 10−6 m2 /s. This indicates that the momentum transfer of
air in the vertical direction is much faster than that of water.

5.2 Laminar Flows


The existence of laminar and turbulent flows was first described by Reynolds in 1883. In his experi-
ments, at low rate of flow the dye pattern was regular and formed a single line of color as shown in
Fig. 5.2. It was called a laminar flow (See also https://www.youtube.com/watch?v=p08 KlTKP50).
While the dye became dispersed throughout the pipe cross section at high flow rates. It is called a
turbulent flow.
Others found that this type of flow is dependent of pipe diameter, fluid density and viscosity as
well as the velocity of the fluid. These four variables were combined and form the Reynolds number,

ρDv
Re ≡ (5.8)
µ

It is a criterion for laminar and turbulent flows. The critical Re in a circular pipe is 2100. As the Re
increases, the flow transits from laminar to turbulent. When the Re is much larger than this value,
the flow becomes really turbulent.

Re < 2100

Re > 2100

Figure 5.2: Reynold’s experiment.


5.2. LAMINAR FLOWS 81

5.2.1 Reynolds Number


As shown in 5.3, a layer of fluid feels both the inertia force to keep the state of motion from the back
layer (horizontal momentum transfer)and the viscous force to adapt the state of motion of the lower
layer (vertical momentum transfer). Reynolds number is the ratio of the inertia force to the viscous
force:
dv
• Inertia force = ma = ρV ∝ ρv 2 L2 (L: characteristic length)
dt
dv
• Viscous force τ A = µ ∝ µvL
dy

Fluid
Inertia force v→
( ρv2 L2 )
Viscous force
(μvL)
Figure 5.3: Physical meaning of Reynolds number.

Inertia force ρv 2 L2 ρvL


Re = = = (5.9)
Viscous force µvL µ
A fluid flow having large Re is determined by mainly the inertia force, whereas a low-Re fluid flow is
determined by the viscous force.

5.2.2 No Slip Condition


The layer of fluid adjacent to a surface has zero velocity relative to the surface, so-called no slip
condition, because fluids do not generally slide along surfaces. Particles close to a surface do not move
along with a flow when adhesion is stronger than cohesion. At the fluid-solid interface, the force of
attraction between the fluid particles and solid particles (Adhesive forces) is greater than that between
the fluid particles (Cohesive forces). This force imbalance brings down the fluid velocity to zero. The
no slip condition is only defined for viscous flows and where continuum concept is valid. The no-slip
condition is a result of the viscous nature of the fluid.

5.2.3 Boundary Layer


The boundary layer1 was first defined by Ludwig Prandtl2 in 1904. It simplifies the equations of
fluid flow (Fig. 5.4) by dividing the flow field into two areas: one inside the boundary layer (adjacent
to a solid boundary), dominated by viscosity and creating the majority of drag experienced by the
boundary body; and one outside the boundary layer, where viscosity can be neglected (treated as
inviscid) without significant effects on the solution. This allows a closed-form solution for the flow in
both areas, a significant simplification of the full Navier–Stokes equations. The majority of the heat
transfer to and from a body also takes place within the boundary layer, again allowing the equations to
be simplified in the flow field outside the boundary layer. The boundary layer thickness is arbitrarily
taken as the distance away from the surface where the velocity reaches 99% of the free-stream velocity.
Von Kármán3 developed momentum integral theory in 1921. He analyzed a fluid flow over flat
plate using a control volume analysis technique. More advanced method was developed Prandtl and his
another student Blasius. He used the boundary layer as a control volume and applied the conservation
1 Re수가 큰 유동의 경우 점성력은 무시할 수 있다. 그러나 이러한 경우에도 고체면과 접하는 유체는 no-slip 조건을 따른다.

따라서 고체면에 가까운 얇은 유체층에는 점성력이 여전히 중요하다. 이러한 얇은 유체층을 경계층(boundary layer) 라고 부른다.
2 (4 February 1875 – 15 August 1953) was a German fluid dynamicist, physicist and aerospace scientist.
3 was a Hungarian-American mathematician, aerospace engineer, and physicist who was active primarily in the

fields of aeronautics and astronautics. He joined Ludwig Prandtl at the University of Göttingen, where he received his
doctorate in 1908.
82 CHAPTER 5. MOMENTUM TRANSFER AND VARIOUS FLOWS

Figure 5.4: Prandtl boundary layer on a flat plate.

of mass and momentum. Also, the quadratic velocity profile in the laminar boundary area was used
and determined by the boundary conditions, which are no-slip at the surface, and v = v∞ at the
boundary, and zero velocity derivative at the boundary. From this, he derived the boundary thickness
is expressed as
δ 5.5 ρv∞ x
= 1/2
where Rex = in a laminar flow (5.10)
x Rex µ

Ex. 23. Calculate the thickness of boundary layers for air and water when the fluids flow with
v∞ = 0.5 m/s over a flat pate L = 1m. ρair = 1.20 kg/m3 and µ = 1.80 × 10−5 Pa·s and ρwater = 998
kg/m3 and µ = 1.0 × 10−3 Pa·s.

Answer: In the case of air


1.2 × 0.5 × 1 δair 5.5
Rex = −5
= 0.3 × 105 (Laminar) =⇒ =√ = 30.1 mm (5.11)
1.8 × 10 L 3333.3
For water
998 × 0.5 × 1 δwater 5.5
Rex = = 4.99 × 105 (close to Laminar) =⇒ =√ = 7.9 mm (5.12)
1 × 10−3 L 499000
The boundary layer grows slowly even for these small velocities. We can conclude that the velocity
normal to the wall is much less than that parallel to the wall.
Figure 5.4 illustrates how the thickness of the boundary layer increases with distance x from the
leading edge. As x increases, flow within the boundary layer becomes turbulent from laminar. Even
in the turbulent boundary-layer region there is a very thin film of fluid called the laminar sublayer in
which flow is still laminar and large velocity gradients exist.
In the laminar boundary layer, the thickness increases x0.5 The experimental data for flow past a
flat plate indicate that for the local Reynolds number Rex ≡ xvρ/µ
• Rex < 2 × 105 : laminar flow in the boundary layer, δ ∝ x0.5
• 2 × 105 <Rex < 3 × 106 : transition, δ ∝ x1.5
• 3 × 106 <Rex : turbulent flow in the boundary layer except the laminar sublayer, δ ∝ x0.8 .
The transition occurs at the lower Rex when the plate is rough.

5.2.4 Fully Developed Flow


At a point well downstream from the entrance, the boundary layer reaches the center of the tube
and the boundary layer occupies the entire cross section of the stream. From this point, the velocity
distribution remains unchanged, which is called fully developed flow. That is, the velocity distribution
is not influenced by entrance effects.
5.3. FLUID FLOW DUE TO PRESSURE DIFFERENCE 83

Figure 5.5: Development of boundary layer flow in pipe: Fully developed flow

Experimental results and numerical simulations for laminar flow of Newtonian fluids indicate that
LE
= 0.49 + 0.11Re (5.13)
R
where LE is the distance at which the centerline velocity reaches 99% of its final value. Thus, the
entrance region will be as long as 230 radii if laminar flow is maintained up to Re = 2100.

5.3 Fluid Flow Due to Pressure Difference


Consider the steady-state incompressible flow of an fluid through a horizontal cylindrical tube of
radius R shown in Fig. 5.6. The flow is assumed to be in the axial direction only and its velocity is
not a function of θ, that is,
vr = vθ = 0, vz ̸= f (θ)
There is no-slip at wall of the pipe. We have two pressure gauges mounted on the tube which show
P = P0 at z = 0 and P = PL at z = L.

P=P0 P=PL

r R vz
z
∆r ∆z

θ = 90o vz=parabolic Trz=linear < 0


z =0 z =L

Figure 5.6: Flow in a pipe.

1. Prove that vz = vz (r) (vz is a function of r only)


In the cylindrical coordinate, the differential mass balance for an incompressible flow says that
1 ∂ 1 ∂vθ ∂vz ∂vz
(rvr ) + + = 0 =⇒ = 0 =⇒ vz = f (r) (5.14)
r ∂r r ∂θ ∂z ∂z
2. Calculate the shear stress at wall and the direction.
The viscous shear stress at the wall is Tzr |r=R (= τrz |r=R ), which can be obtained from the
differential form of linear momentum balance:
 
∂v
ρ
∂t
+ |∇v{z· v} = −∇P + ∇ · τ + ρb
|{z}
(5.15)
|{z} 0, vr = vθ = 0, vz ̸= f (θ) ρge y
0, st-st

· r-component
 
∂P 1 ∂ 1 ∂τθr ∂τzr τθθ
0=− + (rτrr ) + + − + ρbr =⇒ P = ρgr sin θ + f (θ, z)
∂r r ∂r r ∂θ ∂z r |{z}
| {z } ρg sin θ
0, τrr = τθr = τθθ = 0, τzr ̸= f (z)
84 CHAPTER 5. MOMENTUM TRANSFER AND VARIOUS FLOWS

· θ-component
 
1 ∂P 1 ∂  1 ∂τθθ ∂τzθ
0=− + 2 r2 τrθ + + + ρbθ =⇒ P = ρgr sin θ + g(r, z)
r ∂θ r ∂r r ∂θ ∂z |{z}
| {z } ρg cos θ
0, τrθ = τθθ = τzθ = 0

Therefore we can express pressure as

P = ρgr sin θ + h(z)

· z-component4
 
∂P 1 ∂ 1 ∂τθz ∂τzz 1 ∂ ∂P
0=− + (rτrz ) + + + ρbz =⇒ (rτrz ) = (5.16)
∂z r ∂r r ∂θ ∂z |{z} r ∂r ∂z
0
| {z }
0, τθz =τzz =0

Since τrz ̸= f (z) from the viewpoint of flow and ∂P


∂z = h′ (z) ,

1 ∂ ∂P ∆P r c1
(rτrz ) = = constant =⇒ τrz = − + (5.17)
r ∂r ∂z L 2 r
At r = 0 the shear stress has to be finite. This gives c1 = 0 and thus

∆P r
τrz = Trz = − (5.18)
L 2
It is VERY important that the shear stress in a flow by the pressure difference is linearly
distributed from the wall to the center. The viscous force acting on the fluid at the
wall by the cylindrical pipe becomes

∆P R
τzr |r=R = − <0 (5.19)
L 2
It indicates that the shear force acting on the fluid at the wall by the solid cyclinder is in
the −z direction because the positive direction of τzr |r=R is +z.

3. Calculate vz , vmax , ⟨v⟩, Q(= V̇ ), when the fluid is a Newtonian fluid and forms an incompressible
flow.
The shear stress τrz is expressed as
 
∂vr ∂vz
τrz =µ + (5.20)
∂z
|{z} ∂r
0

4 Without knowing the differential form of linear momentum balance, this equation can be derived. Because the mass

flow rates and velocities


P at both z and z + ∆z are equal, the conservation of linear momentum over the differential
element(Fig. 5.6) is F = 0. In the z direction, it becomes
(P |z − P |z+∆z )π[(r + ∆r)2 − r2 ] − τ |r+∆r 2π(r + ∆r)∆z − τ |r 2πr∆z = 0
where because
τ |r+∆r = −Trz |r+∆r and τ |r = Trz |r
Dividing by π[(r + ∆r)2 − r2 ]∆z gives
P |z+∆z − P |z Trz |r+∆r (r + ∆r) − Trz |r r Trz |r+∆r (r + ∆r) − Trz |r r
=2 =2
∆z (r + ∆r)2 − r2 (2r + ∆r)∆r
When ∆r and ∆z approach to zero, it becomes
P |z+∆z − P |z Trz |r+∆r (r + ∆r) − Trz |r r ∂P 1 ∂
lim = lim 2 =⇒ = (rTrz )
∆z→0 ∆z ∆r→0 (2r + ∆r)∆r ∂z r ∂r
which is the same as (5.60)
5.3. FLUID FLOW DUE TO PRESSURE DIFFERENCE 85

Using (5.18) and (5.20), we have

∂vz ∆P r ∆P r2
µ =− =⇒ vz = − + c2 (5.21)
∂r L 2 L 4µ

By the no-slip boundary condition, the velocity at wall is zero so that

∆P R2
  r 2 
vz = 1− (5.22)
L 4µ R

It is VERY important that the velocity in a flow by the pressure difference is expressed as a
parabolic function. It should be noticed that the velocity can be directly derived from the Naiver-
Stoke’s equation, the linear momentum balance for an incompressible flow of a Newtonian fluid.
Using the velocity, we can also obtain the shear stress Trz .
The maximum velocity occurs at the center of the pipe:

R2 ∆P
vmax = vz |r=0 = (5.23)
4µ L

The average velocity becomes


R
R
∆P R2
  r 2 
vz dA
Z
A 1
⟨v⟩ = = 1− 2πrdr
A πR2 0 L 4µ R
(5.24)
R2 ∆P vmax
= =
8µ L 2

From this we can get the pressure drop per unit length as

∆P 8µ⟨v⟩ 32µ⟨v⟩
= 2
= (5.25)
L R D2
It is known as the Hagen-Poiseuille equation, which is applied to the steady-state fully-
developed incompressible flow of a Newtonian fluid.
The volumetric flow rate is
πR4 ∆P
Z
V̇ = ⟨v⟩A = vz dA = (5.26)
A 8µ L

4. Repeat 3 if the fluid is one such that (non-Newtonian fluid)

dvz n−1 dvz



τrz = K (5.27)
dr dr
| {z }
µ

Using (5.18) and (5.27), we have

dvz n−1 dvz


 n  n
dvz n−1 ∆P r dvz ∆P r
K
=K (−1) =− =⇒ = (−1)n
dr dr dr L 2 dr LK 2
 1/n  1/n (5.28)
dvz ∆P n ∆P
=⇒ =− r1/n =⇒ vz = − r(n+1)/n + c2
dr 2LK n + 1 2LK

By the no-slip condition, we finally have


 1/n   r (n+1)/n 
n ∆P
vz = R(n+1)/n 1 − (5.29)
n+1 2LK R

If n = 1, i. e., a Newtonian Fluid, equation (5.29) corresponds to (5.22).


86 CHAPTER 5. MOMENTUM TRANSFER AND VARIOUS FLOWS

Table 5.1: Orders with respect to r.

τrz = µ dv st
dr = 1 -order
z
n µ dvz
dr (r
1/n
) vz (r1+1/n )
Newtonian 1 0 (constant) 1 2
Shear thinning <1 -0.a 1+0.a >2
Shear thickening >1 +0.b 1-0.b <2

Shear thickenning
Trz vz

Newtonian Shear thinning

Figure 5.7: Velocity profiles for Newtonian and Non-Newtonian fluids. Newtonian 유체의 경우 점도가
일정하므로 속도기울기가 r의 1차 함수이다. 따라서 속도는 r에 대하여 2차 함수로 나타내어 진다. 반면
에 shear thinning 유체의 경우 속도의 기울기가 1차보다 높은 차수로 증가하므로 속도는 2차 보다 높은
차수로 증가한다. 따라서 중심부분이 2차 곡선인 Newtonian 경우 보다 뭉뚝해 진다. Shear thickening의
경우에는 2차보다 뽀쪽해 진다.

5. Estimate the average velocity on cross section if the Newtonian fluid flows in a turbulence range,
instead of a Laminar flow.
As show in Fig 5.8, turbulence causes fluid elements at different radial positions to intermingle.
In other words, in turbulent flow the local velocity fluctuates from moment to moment and thus
the profile becomes more blunted. For a turbulent flow in a pipe at the relatively small Re,
experimental results are well represented by a power-law relationship
 1/7
v̄z 60 r
= 1− (5.30)
⟨v⟩ 49 R
Given the relatively uniform velocity in the central part of the tube, there must be a larger
gradient near the wall, if the mean velocity is the same as for laminar flow. This leads to larger
shear stress at the wall in the turbulent flow (τrz = µdvz /dr in (5.20)).

5.4 Fluid Flow Due to Gravity


Consider the steady-state incompressible flow of an fluid down an inclined surface shown in Fig. 5.9.
Pressure is assumed to be a function of y only.
vy = vz = 0, P ̸= g(x)
There is no-slip at the urface.
1. Prove that vx = vx (y) (vx is a function of y only).
In the rectangular coordinate, the differential mass balance for an incompressible flow says that
∂vx ∂vy ∂vz ∂vx
+ + = 0 =⇒ = 0 =⇒ vx = f (y) (5.31)
∂x ∂y ∂z ∂x

2. Calculate the shear stress at plane surface τyx |y=0 .


The shear stress can be obtained from the differential form of linear momentum balance:
 
∂v
ρ + ∇v
| {z· v} = −∇P + ∇ · τ + ρb (5.32)
∂t
|{z} 0, vy = vz = 0, vx ̸= f (z)
0, st-st
5.4. FLUID FLOW DUE TO GRAVITY 87

(a) (b)
Turbulent
vz
uz
vz
Laminar Turbulent
t0 time
(c)

vavg vavg
vz vz

Laminar Turbulent
Figure 5.8: (a) Fully developed state state flow on average. The two curves in turbulent flow correspond
to fluid elements starting at the same position at different time. (b) Velocity fluctuations at a fixed
position in a turbulent flow. The instant velocity is vx (solid curve), its-smoothed value is v¯x (dashed
curve), and the fluctuation is uz . (c) Velocity profiles for laminar or turbulent flow in a tube. The
mean velocity in each case is indicated by the thick red dot lines.

h
vx

Tyx =line
ar
>0

Figure 5.9: Laminar flow down an inclined-plane surface.


88 CHAPTER 5. MOMENTUM TRANSFER AND VARIOUS FLOWS

· y-component
 
∂P ∂τxy ∂τyy ∂τzy
0=− + + + + ρby =⇒ P = −ρg cos θy + g(z)
∂y ∂x ∂y ∂z |{z}
| {z } −ρg cos θ
0, τyy = τzy = 0, τxy ̸= f (x)

· z-component
 
∂P ∂τxz ∂τyz ∂τzz
0=− + + + + ρbz =⇒ P = f (y) (5.33)
∂z ∂x ∂y ∂z |{z}
| {z } 0
0, τxz = τyz = τzz = 0

Therefore we can express pressure as

P = −ρgy cos θ + P0 (5.34)

where P0 is a constant. Since the pressure at the free surface (y = h) is the atmospheric
pressure, we have
P = Patm + ρg(h − y) cos θ (5.35)
· x-component
 
∂P ∂τxx ∂τyx ∂τzx ∂τyx
0= − + + + + ρbx =⇒ = −ρg sin θ
∂x}
| {z ∂x
| {z } ∂y | ∂z
{z } |{z} ∂y
ρg sin θ
0, P ̸=f (x) 0, τxx = 0 0, τzx = 0

=⇒ τyx = −ρgy sin θ + c1

If the fluid surface at y = h is assumed to be free to friction. This gives that


 y
τyx = ρgh sin θ 1 − =⇒ τyx |y=0 = ρgh sin θ > 0 (5.36)
h
It is VERY important that the shear stress in a flow by the gravity is linearly distributed
from the wall to h. The viscous force acting on the fluid at the wall by the plate is
in the −x direction because the normal vector of the surface in the −y direction.
3. Calculate vx , vmax when the fluid is a Newtonian fluid and forms an incompressible flow.
For ompressilbe Newtonian fluid, Tyx is expressed as
 
∂vx ∂vy
Tyx = µ + (5.37)
∂y ∂x
|{z}
0

Using (5.36) and (5.37), we have

y2
 
∂vx  y ρgh sin θ
µ = ρgh sin θ 1 − =⇒ vx = y− + c2 (5.38)
∂y h µ 2h
By the no-slip boundary condition, the velocity at the plane surface is zero so that

ρgh2 sin θ y
 
1  y 2
vx = − (5.39)
µ h 2 h
It is VERY important that the velocity in a flow by the gravity is expressed as a parabolic
function.
The maximum velocity occurs at the top surface:

ρgh2 sin θ
vmax = vx |y=h = (5.40)

5.5. FLUID FLOW DUE TO MOVING PLATES (COUETTE FLOW) 89

v0

y vx
h Tyx=const.
x >0

Figure 5.10: Laminar flow between two plates with the one stationary and the other moving.

5.5 Fluid Flow Due to Moving Plates (Couette Flow)


Figure 5.10 shows an incompressible laminar flow of a fluid between two parallel plates. One of the
surface is stationary, the other surface is moving to the right with the velocity v0 . On both surfaces
no-slip conditions are applied. Pressure doesn’t vary in the x direction. From the viewpoint of flow,
we can easily see that
vy = vz = 0, vx = f (y)

1. Prove that vx = vx (y) (vx is a function of y only).


In the rectangular coordinate, the differential mass balance for an incompressible flow says that

∂vx ∂vy ∂vz ∂vx


+ + = 0 =⇒ = 0 =⇒ vx = f (y) (5.41)
∂x ∂y ∂z ∂x

2. Calculate the shear stress at both plane surfaces.


The shear stress can be obtained from the differential form of linear momentum balance:
 
∂v
ρ + ∇v · v = −∇P + ∇ · τ + ρb (5.42)
∂t | {z }
|{z} 0, vy = vz = 0, vx ̸= f (z)
0, st-st

· y-component
 
∂P ∂τxy ∂τyy ∂τzy
0=− + + + + ρby =⇒ P = −ρgy + g(z)
∂y ∂x ∂y ∂z |{z}
| {z } −ρg
0, τyy = τzy = 0, τxy ̸= f (x)

· z-component
 
∂P ∂τxz ∂τyz ∂τzz
0=− + + + + ρbz =⇒ P = f (y) (5.43)
∂z ∂x ∂y ∂z |{z}
| {z } 0
0, τxz = τyz = τzz = 0

Therefore we can express pressure as

P = −ρgy + P0

· x-component
 
∂P ∂τxx ∂τyx ∂τzx ∂τyx
0= − + + + + ρbx =⇒ =0
∂x} | ∂x ∂y | ∂z |{z} ∂y
0
| {z {z } {z }
0, P ̸=f (x) 0, τxx = 0 0, τzx = 0

Finally we have
τyx = Tyx = c1 (5.44)
90 CHAPTER 5. MOMENTUM TRANSFER AND VARIOUS FLOWS

It is VERY important that the shear stress in a Couette flow is constant, and that the flow
becomes inviscid
∇·τ =0 (5.45)
though the fluid is not inviscid (µ ̸= 0).
3. Calculate vx if the fluid is a Newtonian fluid and forms an incompressible flow.
Using the relation the shear stress and the velocity and (5.44), we have
 
∂vx ∂vy ∂vx c1
τyx = µ + =⇒ µ = c1 =⇒ vx = y + c2 (5.46)
∂y ∂x
|{z} ∂y µ
0

No-slip boundary condition at both of the surfaces says

at y = 0, vx = 0, and at y = h, vx = v0 (5.47)

Thus we can obtain the constants as


v0 µ
c1 = , and c2 = 0 (5.48)
h
and the velocity as
y
vx = v0 (5.49)
h
It is VERY important that the velocity in a Couette flow is expressed as a linear function.
The shear stress at everywhere becomes
v0 µ
τyx = c1 = >0 (5.50)
h
The viscous force acting on the fluid at the top by the upper plate is in the x direction
due to +y normal direction, whereas the viscous force acting on the fluid at the bottom
by the lower plate is in the −x direction due to −y normal direction.

5.6 Superposition Principle of Fluid Flows


A fluid between two vertical plates flows due to one moving plate as well as gravity (Gravity flow +
Couette flow). Using the differential momentum balance and the stress-velocity relation for Newtonian
fluid (or the Navier-Stokes equation), we can obtain the velocity profile for the incompressible flow of
a Newtonian fluid as  2 
ρgh2 x

x x
vy = − − + v0 (5.51)
2µ h h h
|{z}
| {z }
by gravity by a moving plate

More cases about the flow of an incompressible Newtonian fluid on plates are shown in Fig. 5.12.

Ex. 24. As shown in Fig. 5.11, an incompressible flow of a Newtonian fluid is formed between two
vertical wide plates. One of the plates are stationary and the other is moving upward with the velocity
of v0 . Calculate the velocity of the moving plate v0 for the total flow rate to be zero.

Answer: The total volumetric flow rate becomes


Z h  2 
ρgh2 x ρgh3
Z  
x x h
Q = vy dA = − − + v0 dx = − + v0 (5.52)
o 2µ h h h 12µ 2
Since Q = 0,
ρgh2
v0 = (5.53)

5.6. SUPERPOSITION PRINCIPLE OF FLUID FLOWS 91

v0 v0
y

h
τ g
+ =

Flow by gravity and


Flow by gravity Flow by a moving plate
a moving plate

Figure 5.11: Velocity profiles for one surface moving upward.

Flow by moving plates


vx = U y/h As time increases, the velocity in
some thickness becomes constant.
U

y Air
h + = vx = U
vx = U
x U U
vx = U(1-y/h)

Presssure difference with a moving plate Gravity with a moving plate


vx =- ∆P (y2 - hy) vx =
2µL - ρg s Air
i
2µ nθ ( 2
y -
2hy
+ = U + )
=
U
vx = U(1-y/h) θ

vx = - ∆P (y2 - hy) + U(1-y/h) vx =- ρg sinθ (y2 - 2hy) - U


2µL 2µ

Figure 5.12: Various flows on the surface of plates.


92 CHAPTER 5. MOMENTUM TRANSFER AND VARIOUS FLOWS

5.7 Fluid Flow by a Rotating Shaft


A rotating shaft with an angular velocity ω, as illustrated in Figure 5.13, causes an incompressible
flow of a Newtonian fluid to move in a circular motion. Calculate the velocity vθ and the pressure
gradient ∇P .
From the viewpoint of flow we can see that

vr = vz = 0, vθ ̸= vθ (θ), P ̸= P (θ)

It is clear that this velocity satisfies the differential mass balance. The differential form of linear

Figure 5.13: Rotating shaft in a fluid

momentum balance is  
∂v
ρ +∇v · v = −∇P + ∇ · τ + ρb (5.54)
∂t
|{z}
0, st-st

· θ-component
1 ∂ c1
r2 τrθ =⇒ r2 τrθ = c1 =⇒ τrθ = 2

0= 2
r ∂r r
Since the fluid is a Newtonian one, we have
    
vθ 1 ∂vr ∂vθ ∂ vθ
τrθ = µ − + + =µ r (5.55)
r |r {z
∂θ} ∂r ∂r r
0,vr =0

Therefore     
∂ vθ c1 ∂ vθ c1 c1 1
µ r = 2 =⇒ = 3 =⇒ vθ = − + c2 r
∂r r r ∂r r µr 2µ r
we know that at infinity there is no fluid moving (no momentum transfer by viscous effect)

at r = ∞, vθ = 0 =⇒ c2 = 0

and by no-slip at the shaft

at r = R, angular velocity = ω (vθ = Rω) =⇒ c1 = −2µωR2

Thus
ωR2
vθ = (5.56)
r
· r-component
vθ 2 ρω 2 R4
 
∂P ∂P
ρ − =− =⇒ =
r ∂r ∂r r3
5.7. FLUID FLOW BY A ROTATING SHAFT 93

· z-component
∂P ∂P
0=− + ρbz =⇒ = −ρg (5.57)
∂z |{z} ∂z
−ρg

Thus the pressure gradient becomes

ρω 2 R4
∇P = er − ρgez (5.58)
r3

Ex. 25. A fluid of constant density and viscosity is in a cylindrical container of radius R, as shown in
Fig. 5.14. The container is caused to rotate about its own axis at an angular velocity Ω. The cylinder
axis is vertical. Find the shape of the free surface when steady state has been established.

Figure 5.14: Rotating liquid with a free surface.

Answer: At steady state we know that

vr = vz = 0, vθ = vθ (r) (5.59)

We also know that pressure will depend upon r because of the centrifugal force and upon z because
of gravitational force.
Because the fluid is a Newtonian fluid and forms an incompressible flow, the velocity becomes
c1 1
vθ = − + c2 r
2µ r
we know that at r = 0, vθ should be finite and thus c1 = 0.

at r = R, vθ = RΩ =⇒ c2 = Ω

The r-component
vθ 2
 
∂P ∂P
ρ − =− =⇒ = ρΩ2 r
r ∂r ∂r
The z-component
∂P ∂P
0=− + ρbz =⇒ = −ρg (5.60)
∂z |{z} ∂z
−ρg

Thus
1
P = −ρgz + ρΩ2 r2 + c (5.61)
2
94 CHAPTER 5. MOMENTUM TRANSFER AND VARIOUS FLOWS

Since at r = 0 and z = z0 , P = Patm , we have


1
P − Patm = −ρg(z − z0 ) + ρΩ2 r2 (5.62)
2
The equation for the free surface is the surface where pressure is the atmosphere and thus
 2
1 2 2 Ω
0 = −ρg(z − z0 ) + ρΩ r =⇒ z = z0 + r2 (5.63)
2 2g

This is the equation of a parabola.

5.8 Time-dependent Flow


In this section, we will show how we can solve the unsteady-state velocity. A semi-infinite body of a
Newtonian fluid with constant density and viscosity is bounded below by a plate as shown in Fig. .
Then at time t = 0, the plate is set in motion in v0 . We can approximately draw the unsteady-state
flow as shown in Fig 5.15. For this system, it can be easily seen that

t<0 ∞ ∞ ∞
y
Fluid
at rest

x v0 v0 F/A
at t=0 at t=t1 at t=∞

Figure 5.15: Viscous flow of a fluid near a wall suddenly set in motion

vy = vz = 0, vx = vx (y, t) (5.64)

It is clear that this velocity satisfies the differential mass balance. The components of the differential
linear momentum balance are
· x-component
∂vx ∂τyx ∂vx ∂ 2 vx
ρ = =⇒ =ν (5.65)
∂t ∂y ∂t ∂y 2
It says that the time rate of change in vx is dependent upon the kinematic viscosity ν = µ/ρ
· y-component
∂P
0=− + ρby
∂y |{z}
−ρg

· z-component
0=0 (5.66)
The initial and boundary conditions are summarized as

at t ≤ 0, vx = 0 for all y
at y = 0, vx = v0 for all t > 0 (5.67)
at y → ∞, vx = 0 for all t > 0

By solving these, we can obtain the unsteady-state velocity as well as the shear stress.
Let’s introduce a dimensionless velocity ψ = vx /v0 . Then (5.65) and (5.67) become

∂ψ ∂2ψ
=ν 2 (5.68)
∂t ∂y
5.8. TIME-DEPENDENT FLOW 95

and

at t ≤ 0, ψ=0 for all y


at y = 0, ψ = 1 for all t > 0 (5.69)
at y → ∞, ψ = 0 for all t > 0

Now we use the method of combination of variables (change of variables). This is useful only for
semi-infinite regions, such that the initial condition and the boundary condition at infinity may be
combined into a single new boundary condition. To solve this, we let
y
η≡√ =⇒ ψ(t, y) → ψ(η) (5.70)
4νt

then we have
∂ψ ∂ψ ∂η 1 η dψ
= =−
∂t ∂η ∂t 2 t dη
(5.71)
∂ψ ∂ψ ∂η 1 dψ ∂2ψ 1 d2 ψ
= =√ and 2 = 4νt
∂y ∂η ∂y 4νt dη ∂y dη 2

Substitution of these into (5.68) and (5.69) then gives

d2 ψ dψ
2 + 2η dη = 0, =⇒ ψ(η) = c1 + c2 erf(η) (5.72)

and

at η = 0, ψ = 1
(5.73)
at η → ∞, ψ = 0

Here the error function is defined as


Z x Z ∞
2 −t2 2 2
erf(x) = √ e dt and erfc(x) = √ e−t dt = 1 − erf(x)
π 0 π x (5.74)
erf(0) = 0, erf(∞) = 1 and erfc(0) = 1, erfc(∞) = 0

Its solution is
ψ(η) = 1 − erf(η) (5.75)
that is   
y
vx = v0 1 − erf √ (5.76)
4νt

1.0

v=v0 (1-erf(y/(4 νt)1/2)


0.8

0.6
y t increases
erf(x)
0.4

0.2

v0
1 2 3 4 5
x

Figure 5.16: Error function and the velocity profiles as time increases
96 CHAPTER 5. MOMENTUM TRANSFER AND VARIOUS FLOWS

5.9 A Creeping Flow (Re ≪ 1)


Sometimes, the Navier-Stokes equation can be simplified enough for special cases. When the acceler-
ation terms (inertial effect) in the differential momentum balance are neglected, we get

0 = −∇P + ∇ · τ + ρb
(5.77)
= −∇P + µ∇2 v + ρb if an incompressible flow of a Newtonian fluid

which is called the Stokes flow equation. It is sometimes called the steady creeping flow equation,
because the term ρ(v · ∇v), which is quadratic in the velocity, can be discarded when the flow is
extremely slow. The Stokes flow equation is very important in lubrication theory.
As shown in Fig. 5.17, a solid sphere of radius R is rotating slowly at a constant angular velocity
Ω in a large body of quiescent Newtonian fluid. Let’s determine P , vϕ , and Trϕ . It is assumed that
the sphere rotates sufficiently slowly so that it is appropriate to use the creeping flow equation. From

Figure 5.17: A slowly rotating sphere in an infinite expanse of fluid.

the viewpoint of flow we can see that

vr = vθ = 0, vϕ = vϕ (r, θ), P = P (r, θ) (5.78)

It is clear that this velocity satisfies the differential mass balance. The Stokes equation becomes

· r-component
∂P
0=− + ρbr (5.79)
∂r |{z}
−ρg cos θ

· θ-component
1 ∂P
0=− + ρbθ (5.80)
r ∂θ |{z}
ρg sin θ

where b = −gez = −g [cos θer + (− sin θ)eθ ] from Fig. 1.6(c). Thus we can see

P = P0 − ρgr cos θ

· ϕ-component
1 ∂ f (θ)
r3 τrϕ =⇒ r3 τrϕ = f (θ) =⇒ τrϕ = 3

0= 3
(5.81)
r ∂r r
The bounday condition for τrϕ to determine f (θ) is

at r → ∞, τrϕ = 0 (5.82)

Unfortunately, this is automatically satisfied with (5.81).


5.10. TWO DIMENSIONAL FLOWS 97

Thus, we use
 
1 ∂vr ∂vϕ vϕ
τrϕ = µ + − (5.83)
r sin θ ∂ϕ ∂r r

Using (5.78), (5.81) and (5.83)


 
∂vϕ vϕ ∂  vϕ  1 f (θ)
− =r =
∂r r ∂r r µ r3
(5.84)
1 f (θ)
=⇒ vϕ = + rg(θ)
−3µ r2

The boundary conditions are summarized as

at r = R, vϕ = RΩ sin θ ∵ R sin θ = a distance from z-axis


(5.85)
at r → ∞, vϕ = 0

Applying the boundary conditions (5.85) shows that g(θ) = 0 and f (θ) = −3µΩR3 sin θ. Finally
we have
 2
R
vϕ = ΩR sin θ < 0 (5.86)
r

The shear stress acting on the fluid at the wall by the solid Trϕ = τrϕ can be obtained using (5.83)
 3
R
τrϕ = −3µΩ sin θ
r (5.87)
=⇒ τrϕ |r=R = −3µΩ sin θ

Thus the viscous force at the wall is in the −ϕ direction due to +r normal direction.

5.10 Two Dimensional Flows


For the unidirectional flows shown from Sect. 5.3 to 5.9, the continuity and Navier-Stokes equations
can be solved exactly. Although unidirectional flow has important application, flows are generally
more complex. Fortunately, the Navier-Stokes equation can be simplified enough for some special
cases. In this section, two-dimensional flows are introduced in order to show how to simplify the
Navier-Stokes equation.

5.10.1 Flow in a Tapered Channel (Lubrication Approximation for small-


to-moderate Re)
Consider a flow at small-to-moderate Re that is nearly unidirectional, the flow of fluids in a geometry
in which one dimension is significantly smaller than the others. Consequently one component of
the velocity is much larger than the others. The resulting simplification is called the lubrication
approximation. This was first used by Reynolds in 1886 in a study of lubrication, hence the name.
This approximation is fundamental to the study of polymer processing, where it forms the basis for
the analysis of extrusion, coating, calendering, and molding operations. As shown in Fig. 5.18, the
flow in a tapered channel with planar walls are governed by the lubrication approximation, in which
the x component of velocity is much larger than the others. That is, vx ≫ vy and vz = 0, and thus
the continuity equation becomes ∂vx /∂x ≈ 0.
When an compressible Newtonian fluid flows in a tapered channel,
x
h(x) = h0 + (hL − h0 ) (5.88)
L
98 CHAPTER 5. MOMENTUM TRANSFER AND VARIOUS FLOWS

y hL
h0 h(x)
x

Figure 5.18: Flow in a tapered channel with planar walls at small-to-moderate Re.

the differential momentum balance becomes


 2
∂ 2 vx
  
∂vx ∂vx ∂P ∂ vx
ρ vx + vy =− +µ +
∂x ∂y ∂x ∂x2 ∂y 2
 2 (5.89)
∂ 2 vy
  
∂vy ∂vy ∂P ∂ vy
ρ vx + vy =− +µ + + ρby
∂x ∂y ∂y ∂x2 ∂y 2
Note that all variables vx , vy , and P are functions of x and y.
Using the lubrication approximation, vy ≈ 0 and ∂vx /∂x ≈ 0, the equations can be simplified to

∂P ∂ 2 vx
0≈− +µ 2
∂x ∂y
(5.90)
∂P
0≈− + ρby
∂y
Later, we will confirm the validity of the approximation.
From the second equation, we have
∂P
P (x, y) = −ρgy + f (x) ⇒ = f ′ (x) (5.91)
∂x
and thus we have from the first equation
 
1 ∂P
vx = y 2 + a(x)y + b(x) (5.92)
2µ ∂x

Under the no-slip condition, vx (x, ±h) = 0, the velocity finally is expressed as
" 2 #
h2 (x)
 
∂P y
vx (x, y) = − 1− (5.93)
2µ ∂x h(x)

On the other hand, the volumetric flow rate becomes


Z h
h2 (x)
 
∂P 4h(x)
Q= vx dy = − (5.94)
−h 2µ ∂x 3

and thus
∂P 3 µQ
=− 3 (5.95)
∂x 2 h (x)
and "  2 #
3Q y
vx (x, y) = 1− (5.96)
4h(x) h(x)
Equations (5.88), (5.91) and (5.95) lead to
Z  
3 µQ 3 µQL 1
f (x) = − dx = +c (5.97)
2 h3 (x) 2 hL − h0 2h2 (x)
5.10. TWO DIMENSIONAL FLOWS 99

Finally, the pressure can be written as


 
3 µQL 1
P (x, y) = −ρgy + +c (5.98)
2 hL − h0 2h2 (x)
and the pressure drop at the same height in the tapered channel is
 
3 h0 + hL
∆P = P0 − PL = µQL (5.99)
2 2h0 2 hL 2
Now let’s determine the y-component of velocity vy . The continuity equation is
∂vx ∂vy
+ =0 (5.100)
∂x ∂y
and thus
" 2 #
Z y Z y 
∂vx 3Q y
vy (x, y) = − dY = 2 (x)
1−3 h′ (x)dY
0 ∂x 0 4h h(x)
"  2 # (5.101)
3Q ′ y y
= h (x) −
4h(x) h(x) h(x)

because at y = 0, vy (x, 0) = 0. The y-component velocity is proportional to dh/dx.

Ex. 26. Verification of the Lubrication Approximation. Although the above solutions (5.96),
(5.98), and (5.101) are plausible, we should examine more carefully when they will be accurate. We
have to check the magnitudes of the terms which are omitted in (5.102).
∂vx ∂vx ∂P ∂ 2 vx ∂ 2 vx
ρvx + ρvy =− + µ 2 +µ 2
| {z∂x} | {z∂y} ∂x | ∂x
{z } | ∂y
{z }
T4 T3 T2 T1
(5.102)
∂vy ∂vy ∂P ∂ 2 vy ∂ 2 vy
ρvx + ρvy =− + µ 2 + µ 2 +ρby
| {z∂x} | {z∂y} ∂y | ∂x
{z } | ∂y
{z }
T8 T7 T6 T5

Answer: It has to be proven that T1 is much larger than the other T’s. To facilitate comparisons, we
will average each term over the channel height and consider only absolute values.
∂ 2 vx 3µQ 3µQ
T1 = µ 2
=− 3 ⇒ ⟨T1 ⟩ = (5.103)
∂y 2h 2h3
 2   2
∂ 2 vx 3µQ dh  y 2  3µQ dh
T2 = µ 2 = 1−6 ⇒ ⟨T2 ⟩ = (5.104)
∂x 2h3 dx h 2h3 dx
2
 
∂vx 3ρQ dh
T3 = ρvy ⇒ ⟨T3 ⟩ = ⟨T4 ⟩ = 3
(5.105)
∂y 20h dx
Thus 2
⟨T2 ⟩ dh ⟨T3 + T4 ⟩ Re dh
= and = (5.106)
⟨T1 ⟩ dx ⟨T1 ⟩ 5 dx
On the other hand,
⟨T5 ⟩ 3 dh
= (5.107)
⟨T1 ⟩ 2 dx
There is no necessary to check the others, because they are smaller than T5 .
In summary, the lubrication approximation will be accurate if

dh
≪ 1 and Re dh ≪ 1

dx dx (5.108)
100 CHAPTER 5. MOMENTUM TRANSFER AND VARIOUS FLOWS

5.10.2 Irrotational Flow


For a steady-state incompressible and irrotational flow of an Newtonian fluid, as derived in Sect. 4.6.2,
the differential form of linear momentum balance can be written as
 2 
ρv
∇ + P = ρb (5.109)
2

It should be noticed here again that the viscous effect is negligible due to the irrotational motion of
the fluid. Thus, the differential momentum balance cannot give an information on the velocity profiles
as we did up to now. Instead, the differential mass balance can be used for determining the velocity.
As shown in Fig. 5.19, a Newtonian fluid flows incompressiblely and irrotationally around a cylinder
of infinite length. Therefore the viscous transfer does not occur from the surface of the cylinder and
the velocity profile is only caused by geometry of the cylinder. In this case, there are two nonvanishing
components of the fluid velocity: vr and vθ . Let’s determine them with P .

Figure 5.19: Cylinder in a uniform flow

From the viewpoint of flow we can see that

vr = vr (r, θ) ̸= 0, vθ = vθ (r, θ) ̸= 0, and vz = 0 (5.110)

The boundary conditions are

at r = a, vr = 0
at θ = 0 and π, vθ = 0 ∵ symmetry
(5.111)
at r → ∞, |v| = v∞
at r → ∞, P = P∞

Notice that for a flow in which the viscous effect is negligible, no-slip condition cannot be applied.
Thus the tangential component of velocity vθ at wall of the cylinder may not be zero, but the normal
component of velocity at the wall is still zero.
Since the fluid is incompressible, the differential mass balance (equation of continuity) must satisfies

1 ∂ 1 ∂vθ ∂vz
∇·v = (rvr ) + + =0 (5.112)
r ∂r r ∂θ ∂z
|{z}
0, vz =0

Let
∂Ψ
rvr ≡ (5.113)
∂θ
Then (5.112) becomes  
∂ ∂Ψ
+ vθ =0 (5.114)
∂θ ∂r
5.10. TWO DIMENSIONAL FLOWS 101

For this to be true in general


∂Ψ
vθ = − + f (r) (5.115)
∂r
Let’s set f (r) = 0 and find vθ satisfying the boundary conditions. Instead of having 2 unknowns vr , vθ ,
we have only one unknown Ψ(r, θ), called the stream function (see Appendix D). The stream function
can be used to visualize a velocity field that has been found either analytically or numerically.
Since the flow is irrotational, the curl of the velocity must be zero:
      
1 ∂vz ∂vθ ∂vr ∂vz 1 ∂ ∂vr
∇× v= − er + − eθ + (rvθ ) − ez = 0 (5.116)
r ∂θ ∂z ∂z ∂r r ∂r ∂θ
| {z } | {z }
0, vz =0,vθ ̸=f (z) 0, vz =0,vr ̸=f (z)

Using (5.113), (5.115), and (5.116), we can write


   
∂ ∂Ψ ∂ 1 ∂Ψ
r + =0 (5.117)
∂r ∂r ∂θ r ∂θ
This can be written as
∂ 2 Ψ 1 ∂Ψ 1 ∂2Ψ
+ + =0 (5.118)
∂r2 r ∂r r2 ∂θ2
Equation (5.111) becomes
1 ∂Ψ
at r = a, =0
a ∂θ
∂Ψ
at θ = 0 and π, =0 ∵ symmetry
s ∂r  (5.119)
2  2
1 ∂Ψ ∂Ψ
at r → ∞, + − = v∞
r ∂θ ∂r
at r → ∞, P = P∞

Now we can obtain Ψ by solving (5.118) with (5.119)

Method of Separation of Variables


Ψ = f (r)g(θ) (5.120)
Equation (5.118) becomes

f′ f f ′′ + f ′ /r g ′′
f ′′ g + g + 2 g ′′ = 0 =⇒ 2
=− ≡ λ2 (5.121)
r r f /r g
Therefore we have
g ′′ + λ2 g = 0
f′ f (5.122)
f ′′ + − λ2 2 = 0
r r
We can obtain solutions of these equations as

g(θ) = c1 cos(λθ) + c2 sin(λθ)


(5.123)
f (r) = c3 rλ + c4 r−λ

by using a trial solution f (r) = rm

m(m − 1) + m − λ2 = 0 ⇒ m = ±λ (5.124)

and thus
Ψ = c3 rλ + c4 r−λ c1 cos(λθ) + c2 sin(λθ)
 
(5.125)
102 CHAPTER 5. MOMENTUM TRANSFER AND VARIOUS FLOWS

Using (5.119), we determine λ and c1 · · · c4 . From (5.119)2 ,



∂Ψ ∂
c3 rλ + c4 r−λ
 
= c1 cos(0) + c2 sin(0)
∂r θ=0 ∂r
(5.126)
=0
∴ c1 =0
and

∂Ψ ∂
c3 rλ + c4 r−λ
 
= c2 sin(λπ)
∂r θ=π
∂r
(5.127)
=0
∴ λ =0, 1, 2, · · ·
But λ should be 1 because
when λ = 0 ⇒ Ψ = 0 ⇒ vr = vθ = 0 incorrect trivial solution

∂Ψ (5.128)
when λ ≥ 2, ⇒ = 0 ⇒ vθ |θ=π/λ = 0 incorrect solution
∂r θ=π/λ

Now equation (5.125) becomes


Ψ = sin θ c2 c3 r + c2 c4 r−1

(5.129)
In addition, from (5.119)3 , we can get
"r #
1 2 2
cos2 θ c2 c3 r + c2 c4 r−1 + sin2 θ c2 c3 − c2 c4 r−2

v∞ =
r2 (5.130)
r→∞
∴ c2 c3 = v∞
Using (5.119)1 gives
1
cos θ(v∞ a + c2 c4 a−1 ) = 0 ⇒ c2 c4 = −v∞ a2 (5.131)
a
Finally we have
a2
 
Ψ = v∞ r sin θ 1 − 2 (5.132)
r
and thus
a2
 
1 ∂Ψ
vr = = v∞ cos θ 1 − 2
r ∂θ r
(5.133)
a2
 
∂Ψ
vθ = − = −v∞ sin θ 1 + 2
∂r r
In order to obtain pressure, we use the differential form of linear momentum balance for this flow
(5.109). Its r- and θ-components are
∂ ρv 2 1 ∂ ρv 2
   
+ P = ρgr , + P = ρgθ (5.134)
∂r 2 |{z} r ∂θ 2 |{z}
−ρg sin θ −ρg cos θ

and thus
ρv 2
P =− − ρgr sin θ + P0 (5.135)
2
where P0 is constant and
" 2 2 #
a2 a2
 
2 2 2 2
v =v·v = v∞ cos θ 1 − 2 + sin θ 1 + 2 (5.136)
r r

It is clear from (5.133) that the curl of the viscous portion of the stress is zero.
5.10. TWO DIMENSIONAL FLOWS 103

5.10.3 Creeping Flow


A Newtonian fluid flows incompressibly past a solid sphere of radius R in the z direction. The fluid
flows sufficiently slowly so that it is appropriate to use the creeping flow equation. From the viewpoint

z
r
θ

Moving
fluid v∞

Figure 5.20: Creeping flow past a sphere of radius R.

of flow we can see that


vr ̸= 0, vθ ̸= 0, vϕ = 0, P = P (r, θ) (5.137)
Since no-slip occurs at the surface of the sphere, the boundary conditions are

at r = R, vr = 0
at r = R, vθ = 0
(5.138)
at r → ∞, |v| = v∞
at r → ∞, P = P∞

The differential mass balance (equation of continuity) must satisfies


1 ∂ 1 ∂ 1 ∂vϕ
r 2 vr +

∇·v = (vθ sin θ) + =0 (5.139)
r2 ∂r r sin θ ∂θ r sin θ ∂ϕ
| {z }
0, vϕ =0

Let
1 ∂Ψ
vr = (5.140)
r2 sin θ ∂θ
Then (5.139) becomes  
∂ 1 ∂Ψ
+ vθ sin θ =0 (5.141)
∂θ r ∂r
For this to be true in general
1 ∂Ψ
vθ = − (5.142)
r sin θ ∂r
Instead of having 2 unknowns vr , vθ , we have only one unknown Ψ(r, θ), called the stream function
(see Appendix D).
Now we use the Stoke’s flow equation to solve Ψ. Having both velocity and pressure as dependent
variables in the equation of motion gives difficulty in multidimensional flow. It is therefore convenient
to eliminate the pressure by taking the curl:

0 = ∇ × (−∇P + µ∇2 v + ρb) = − ∇ × ∇P +µ∇ × ∇2 v + ρ∇ × b


| {z } | {z }
0 0 (5.143)
=⇒ ∇ × ∇2 v = 0
104 CHAPTER 5. MOMENTUM TRANSFER AND VARIOUS FLOWS

Using (5.140), (5.142), and (5.143) we have


 2  2
∂ sin θ ∂ 1 ∂
+ Ψ=0 (5.144)
∂r2 r2 ∂θ sin θ ∂θ
This is consistent with the boundary conditions:
1 ∂Ψ
at r = R, =0
R2 sin θ ∂θ
1 ∂Ψ
at r = R, − =0
s θ ∂r
R sin
2  2
1 ∂Ψ 1 ∂Ψ (5.145)
at r → ∞, + − = v∞
r2 sin θ ∂θ r sin θ ∂r
1
⇒ Ψ → v∞ r2 sin2 θ
2
at r → ∞, P = P∞
Now we postulate a solution of the form based on the third boundary condition as
Ψ(r, θ) = f (r) sin2 θ (5.146)
Substituting this into (5.144) gives
d2 d2
  
2 2
− 2 − 2 f =0 (5.147)
dr2 r dr2 r
and finally we get
Ψ(r, θ) = (c1 r−1 + c2 r + c3 r2 + c4 r4 ) sin2 θ (5.148)
Applying the boundary conditions (5.145), we can obtain
"    3 #
3 R 1 R
vr = v∞ 1 − + cos θ
2 r 2 r
"    3 # (5.149)
3 R 1 R
vθ = −v∞ 1 − − sin θ
4 r 4 r

Thus, the shear stress can be written as


   3
vθ 1 ∂vr ∂vθ 3  µv∞  R
τrθ = µ − + + =− sin θ (5.150)
r r ∂θ ∂r 2 R r
To obtain the pressure distribution, we substitute (5.149) into the Stoke’s equation and get
1 ∂2
  
∂P 1 ∂ ∂vr
+ µ 2 2 r 2 vr + 2

0=− sin θ + ρbr
∂r r ∂r r sin θ ∂θ ∂θ |{z}
−ρg cos θ
      (5.151)
1 ∂P 1 ∂ 2 ∂vθ 1 ∂ 1 ∂ 2 ∂vr
0=− +µ 2 r + 2 (vθ sin θ) + 2 + ρbθ
r ∂θ r ∂r ∂r r ∂θ sin θ ∂θ r ∂θ |{z}
ρg sin θ

where b = −gez = −g [cos θer + (− sin θ)eθ ] from Fig. 1.6(c). Thus we can see
 2
3  µv∞  R
P = P∞ − ρgr cos θ − cos θ (5.152)
2 R r
The pressure and shear stress acting on fluid at the surface by the sphere are
3  µv∞ 
P (R, θ) = P∞ − ρgR cos θ − cos θ
2 R (5.153)
3 µv∞
 
τrθ (R, θ) = − sin θ
2 R
5.10. TWO DIMENSIONAL FLOWS 105

Note that the normal stress is zero everywhere at the sphere surface from (5.149)

∂vr
τrr (R, θ) = 2µ =0 (5.154)
∂r

Summary
106 CHAPTER 5. MOMENTUM TRANSFER AND VARIOUS FLOWS

운동량 전달 및 Reynolds 수

수직방향 Dx Dy !" !!"


운동량 전달 L =% !
!# !&

v
흐름(수평)방향 운동량 전달

(유동에 의한) 수평방향 운동량 전달 속도 = v


(점도에 의한) 수직방향 운동량 전달 속도 = µ/r (µ/r 의 차원은 cm2/s 이므로)
L

Reynolds 수 = 유동운동량 전달속도/점도운동량 전달속도 = rvL/µ

화학공학부 울산대학교
SCHOOL OF CHEMICAL ENGINEERING UNIVERSITY OF ULSAN

유체의 유동
!"# !" ! 관성력
Reynolds 수 = = =
Fluid $ $" ⁄# 점성력
Inertia force v→
( ρv2 L2 ) 움직이는 유체는 같은 상태로 계속 움직이려
Viscous force
는 관성력과 주변유체의 영향을 받아 주변의
(μvL)
상태에 적응하려는 점성력을 갖는다.

점성유동 비점성유동
(creeping flow) (inviscid flow) 층류 유동 난류 유동
관성력 무시 점성력 무시
(µ=0) 유체입자들이 서로 엉
- v, L: small 유체입자들이 서로 엉
키지 않고 층을 이루
키면서 흐르는 유동
며 흐르는 유동
Re ® ¥
Re <<1

Potential flow

Re 수 = 2100
화학공학부 울산대학교
SCHOOL OF CHEMICAL ENGINEERING UNIVERSITY OF ULSAN
5.10. TWO DIMENSIONAL FLOWS 107

경계층 (Boundary layer)


• 고체 표면 위를 지나는 흐름은 표면 근방에 점성력이 지배하는 boundary layer와
비점성 흐름을 보이는 경계층 밖, 두 구역으로 구별됨 (Prandtl 1904).
• Karman momentum integral theory로부터 laminar boundary layer의 두께는 아래
와 같이 유도됨.

" 5.5 "#$


= where Rex=
# Rex %

• 유체가 0.5 m/s로 평판 위를 흐를 때


공기: " ⁄# = 3 cm (단위 m당)

물 : " ⁄# = 0.79 cm (단위 m당)


Fig. 12.5 in Welty’s Book

ü Rex < 2✕105 : laminar flow in the boundary layer, " ∝ # !.#
ü 2✕105 < Rex < 3✕106 : transition, " ∝ # $.#
ü 3 ✕ 106 < Rex : turbulent flow in the boundary layer except the laminar sublayer, " ∝ # !.%

화학공학부 울산대학교
SCHOOL OF CHEMICAL ENGINEERING UNIVERSITY OF ULSAN

뉴튼 유체의 층류 유동 (정상상태)
Ø 압력차 유동 Ø 중력 유동 Ø Couette 유동

g t F
.+
P1 P2 t

t
t

1 ) .+
.&'(
. = .&'( 1 − . = .= >
2 2 ℎ

3$ − 3) 4 = t ×4*
;4:< = t ×4*
7.
= −6 1 = 2 4* 7.
71 = −6 1 = 2 4*
71
∆32 ) ∆" 32' (
.&'( = = ;<2 )
46: # )! .&'( =
46
Hagen-Poiseuille Eq.
화학공학부 울산대학교
SCHOOL OF CHEMICAL ENGINEERING UNIVERSITY OF ULSAN
108 CHAPTER 5. MOMENTUM TRANSFER AND VARIOUS FLOWS

비뉴튼 유체의 층류 유동
Shear force는 중심에서 r방향으로 선형적으로 증가함 ! d&
= $
" d'
Shear force 속도

' $%$/'
& = &!"# 1 −
*
Newtonian: n=1
Shear thinning: 0<n<1
Shear thickening: n>1
Newtonian Shear Shear
thinning thickening

ü Newtonian: 점도가 일정하고 shear force가 일차이므로 속도는 반지름에 대하여 2차식임
ü Shear thinning: 중심에서 벽으로 갈수록 속도기울기가 커져서 점도가 줄어듦으로 Newtonian
유체보다 상대적으로 관성력이 커져서 벽쪽의 속도가 커진다. 따라서 속도의 차원은 2차보다
커짐.
ü Shear thickening: 중심에서 벽으로 갈수록 속도기울기가 커져서 점도가 커짐으로 Newtonian
유체보다 상대적으로 점성력이 커져서 벽쪽의 속도가 줄어든다. 따라서 속도의 차원은 2차보
다 적어짐.
화학공학부 울산대학교
SCHOOL OF CHEMICAL ENGINEERING UNIVERSITY OF ULSAN

뉴튼유체의 난류 유동
Ø 난류 유동
(a) (b)
Turbulent
vz
uz
vz
Laminar Turbulent
t0 time
(c)

vavg vavg
vz vz

Laminar Turbulent

$/(
&̅ 60 '
= 1−
& 49 *

층류보다 관성력이 증가하므로 벽쪽의 속도가 층류보다 커지고 중심부분이 뭉툭해짐

화학공학부 울산대학교
SCHOOL OF CHEMICAL ENGINEERING UNIVERSITY OF ULSAN
5.10. TWO DIMENSIONAL FLOWS 109

속도의 중첩원리

화학공학부 울산대학교
SCHOOL OF CHEMICAL ENGINEERING UNIVERSITY OF ULSAN

Semi-infinite region에서 운동량 전달


y !"! !"!
=% "
t<0 !# !&
Fluid
x
at rest &
⟹ "! = "# 1 − erf
∞ ∞ ∞ 4%#
t=0 시간에 대한 x방향의 속도 변화는
Wall set in
Force, 동점도가 클수록 크다.
v0 v0 F/A
at t=0 at t=t1 at t=∞
è 동점도가 클수록 y방향으로 운동
량 전달이 빨라진다.

1.0

v=v0 (1-erf(y/(4 νt)1/2)


0.8

0.6
y t increases
erf(x)
0.4

0.2

v0
1 2 3 4 5
x
화학공학부 울산대학교
SCHOOL OF CHEMICAL ENGINEERING UNIVERSITY OF ULSAN
110 CHAPTER 5. MOMENTUM TRANSFER AND VARIOUS FLOWS

윤활근사법에 의한 근사 1차원 유동 해석
- 윤활근사 (lubrication approximation)은 1886년 Reynolds 가 lubrication 유동을
해석하면서 시작되었음.

- 고분자 가공 공정 (extrusion, coating, calendaring, and molding operations)을 해


석하는데 주로 쓰임

- 2차원 흐름이지만 한방향의 속도가 다른 방향에 비하여 매우 커 거의 일차원 흐


름과 유사한 흐름. 즉, 채널 직경 또는 채널간 거리의 변화가 작고 Re수가 크지 않
는 경우에 해당함.

y hL ℎ! ≪ 1 & Re×ℎ! ≪ 1
h0 h(x)
x u(x) +)" +)"
⟹ )" ≫ )# & ≫
+, +-

L
화학공학부 울산대학교
SCHOOL OF CHEMICAL ENGINEERING UNIVERSITY OF ULSAN

2차원 Creeping, 비회전성 흐름


P∞ - ρgR

z
r FPstat FPmov Fτ
P∞ P∞
θ

!! 가 매우 적어서
R
Re ≪ 1인 흐름
P∞ + ρgR

3μv∞ 3μv∞
Pstat = P∞ - ρgR cosθ Pmov = - cosθ τrθ = - sinθ
2R 2R

∇×( = 0인 흐름
Moving FPstat FPmov Fτ

fluid v∞

Creeping (or Stokes flow) : 관성력 무시 Irrotational flow: 점성력이 0임

- 2차원 흐름은 흐름함수(stream function)을 도입하여 연속방정식으로 속도성분들


을 흐름함수로 표현해야 함.

- 운동방정식 및 creeping 또는 속도의 curl=0 조건들을 활용하여 흐름함수를 구할


수 있음. 또한 이로 부터 유체 움직임으로 인한 압력들도 구할 수 있음.

- 비회전성 흐름의 경우 고체 표면에서 미끄러짐 발생함.

화학공학부 울산대학교
SCHOOL OF CHEMICAL ENGINEERING UNIVERSITY OF ULSAN
Chapter 6

Application of Momentum Transfer

Until Ch. 5, we have used the conservation laws to analyze fluid flows macroscopically (conservation
of mass, momentums, and mechanical energy) and microscopically (equation of continuity, motion,
and mechanical energy-Bernoulli). In this chapter, we will discuss how to expand them to analyze
forces acting on a solid, energy losses during fluid flow, pumps, drag, and flow in packed and fluidized
beds.

6.1 Correction of the Bernoulli Equation


Let’s go back to the equation of mechanical energy in Sect. 4.8 for a steady-state incompressible and
inviscid fluid or flow, so-called the streamline Bernoulli equation.
 
d 1
P + ρv 2 + ρgh = 0 (6.1)
ds 2
This must be applied any points on the same streamline. Integrating this along a streamline from
Point 1 and Point 2 shown in Fig. 6.1, we get
1 1
P1 + ρv12 + ρgh1 = P2 + ρv22 + ρgh2 (6.2)
2 2
In other words, the sum of pressure, kinetic energy, and potential is the same at any points on a
streamline. It should be noticed that each value in (6.2) is a local one on each point. As mentioned
in Sect. 4.8, they may vary from one streamline to another, though the sums are constant along a
streamline.

v2
P2

v1
P1 h2

h1
P+1/2ρv2+ρgh = consant at points on a streamlne

Figure 6.1: Streamline Bernoulli Theorem: the total energy of a fluid flowing from Cross section 1 to
Cross section 2 remains constant, though one energy form can be converted into another.

Most of fluid flow problems encountered in engineering involve streams that are influenced by solid
boundaries and therefore contain boundary layers. This is especially true in the flow of fluids through

111
112 CHAPTER 6. APPLICATION OF MOMENTUM TRANSFER

pipes and other equipment, where the entire stream may be in boundary layer flow. To extend the
Bernoulli equation to cover these practical situations, two modifications are needed. The first, usually
of minor importance, is a correction of the kinetic energy term for the variation of local velocity with
position in the boundary layer; the second, of major importance, is the correction of the equation for
the existence of fluid friction, which appears whenever a boundary layer forms.

6.1.1 Bernoulli Equation : Kinetic Energy Correction


A difficulty to use (6.2) is that the values are local values and thus could be different at any points
on cross section. Instead of using the local values, we express the Bernoulli equation using their mean
values. This can be achieved by integrating the energies of the equation along the cross-section of
a pipe. The average of pressure and potential energy of all particles passing through a cross section
become R R
(P + ρgh)dṁ ρvdA
= (P + ρgh) = P + ρgh (6.3)
ṁ ρ⟨v⟩A
because P + ρgh is constant in the cross section of a gradually varied flow. On the other hand, the
kinetic energy should be integrated by differential mass flow rate dṁ through the differential cross
section dA. Thus, the average kinetic energy becomes
R 1 2 R 1 2 R 3
2 ρv dṁ 2 ρv ρvdA v dA 1 2 ⟨v 3 ⟩ 1 2 α 2
= = 3 ρ⟨v⟩ = 3 ρ⟨v⟩ = ρ⟨v⟩ (6.4)
ṁ ρ⟨v⟩A ⟨v⟩ A 2 ⟨v⟩ 2 2

where ⟨v⟩ is the mean velocity on the cross section and α is called the kinetic energy correction factor :

⟨v 3 ⟩
α= 3 (6.5)
⟨v⟩

Based on these, we get


2 2
ρ⟨v1 ⟩ ρ⟨v2 ⟩
P1 + α1 + ρgh1 = P2 + α2 + ρgh2 (6.6)
2 2
Here 1 and 2 are cross-sections rather than points.
For turbulent flow the plug-flow approximation is accurate enough to set α = 1 and replace ⟨v⟩
by v. These simplifications are suggested by results for fully developed flow in circular tubes. For
turbulent flow, using the velocity profile (5.30) gives
 3 Z R 
60  r 3/7
α= 1− dA A = 1.06 (6.7)
49 0 R

An error of about 6% is ordinarily negligible, given the other approximations being made. For laminar
flow, using (5.22) and (5.24) gives
Z R   r 2 3 
α=8 1− dA A = 2 (6.8)
0 R

More generally, the value of α could be expressed as


p
α = 1 + 0.78ff (15 − 5.9 ff ) (6.9)

where ff is the Fanning friction factor of the fluid, which will discussed later.

6.1.2 Bernoulli Equation : Correction for Friction


Friction manifests itself by the disappearance of mechanical energy. In frictional flow, the sums of
pressure and the kinetic and potential energy is not constant along a streamline, but always decreases
in the direction of flow. In accordance with the principle of conservation of energy, an amount of heat
6.1. CORRECTION OF THE BERNOULLI EQUATION 113

equivalent to the loss in mechanical energy is generated. Thus the energy loss (or heat generation)
should be added in the Bernoulli equation (6.6)
2 2
ρ⟨v1 ⟩ ρ⟨v2 ⟩
P1 + α1 + ρgh1 = P2 + α2 + ρgh2 + hf (6.10)
2 2
Here hf represents the frictional heat(energy loss) generated per unit volume of fluid when the fluid
flows the pipe between cross-section 1 and cross-section 2 and is always positive. There exist the
conversion or loss of mechanical energy to heat by viscous action, sudden change in diameter, fitting,
and so on. Therefore
hf = hf s + hf c + hf e + hf f (6.11)
hf s , hf c , hf e , and hf f are energy loss by skin friction due to viscous action, by sudden contraction,
sudden expansion, and fittings and valves, respectively. This will be discussed as follows.

Ex. 27. Calculate PA (Example 1 of Chapter 2 in Welty’s book)

Figure 6.2: A U-tube manometer

Answer: PB =PC and use the Bernoulli equation between a point on A and at a point on B, and
between a point onC and a point on D.
PA + ρT gd1 = PB and PC = PD + ρM gd2 =⇒ PA = Patm + ρM gd2 − ρT gd1 (6.12)

Ex. 28. A siphon(흡입관) may be used to draw a liquid above its level in an open tank and discharge
it below that level. Once initiated, the flow is sustained by the height difference. Assume that viscous
losses are negligible, and that the kinetic energy correction factor is one.
• After an empty tube has been inserted into the tank, how much suction must be applied at
position C to start the flow?
• Find the velocity of the fluid exiting the siphon. Here the velocity is assumed to be the same
on the cross section.
• Show that during operation the pressure at position B is less than atmospheric pressure.
• Lowering the outlet will increase the flow rate. However, cavitation might eventually make the
siphon unreliable. That can be avoid if the pressure everywhere exceeds Pv , the vapor pressure
of the liquid. Find the maximum velocity vmax without the cavitation.
Answer: The pressure at a point on B is PB = P0 − ρgHB . Because the tube between B and C is
occupied by air before the flows begins, this is also value of PC needed to start the flow.
PC = PB + ρair g(HB + HC ) ≈ PB (6.13)
This suction pressure at C becomes P0 after removing the suction. Applying the Bernoulli equation
at a point on the surface of tank and at a point on C gives
1 1 2 p p
PS + ρvS2 + ρghS = PC + ρvC + ρghC =⇒ vC = 2g(hS − hC ) = 2gHC (6.14)
2 2
114 CHAPTER 6. APPLICATION OF MOMENTUM TRANSFER

Figure 6.3: Open tank with siphon

because PS = PC = P0 and vS ≈ 0. Applying the Bernoulli equation at a point on B and at a point


on C gives

1 2 1 2
PB + ρvB + ρghB = PC + ρvC + ρghC =⇒ PB = P0 − ρg(HB + HC ) (6.15)
2 2
because vB = vC . Though the pressure at B is lower than the pressure at C, the gravity makes the flow
downward. From (6.16), it is clear that lowering the outlet HC increases the velocity, but decreases
PB . Applying the Bernoulli equation at a point on the surface of tank and at a point on B gives

1 1 2 p
PS + ρvS2 + ρghS = PB + ρvmax + ρghB =⇒ vmax = 2[(P0 − Pv )/ρ − gHB ] (6.16)
2 2

6.2 Energy Loss by Skin Friction for Flow in Pipes


6.2.1 Skin Friction
When a fluid flows in a pipe or channel, there occurs a friction between the solid surface and the
fluid, so-called the skin friction. It arises from the friction of the fluid against the “skin” of the object
that is moving through it because the fluid layer does not slide on the surface due to the interaction
between the fluid and the skin of the body, and is directly related to the wetted surface, the area of
the surface of the body that is in contact with the fluid.
The viscous friction force by the skin friction is dependent of the velocity, density, and viscosity
of the fluid, and the characteristic length (e.g. diameter):

Ff = f (⟨v⟩, ρ, µ, D) (6.17)

where we neglect pipe roughness. By the dimensional analysis (see Sect. E.3), we are able to signifi-
cantly reduce number of experiments to obtain the relation (6.17) between the friction force and other
variables. The dimensional analysis has enabled us to relate the original five variables in terms of only
two dimensionless parameters in the form

Ff /D2
2 = f¯(Re) (6.18)
ρ⟨v⟩

and thus
1 2
Ff = Cf (Re) × ρ⟨v⟩ × Aw (6.19)
2
It is clear that the skin friction coefficient is a function of the Reynolds number only. Of course, if we
consider pipe roughness, the friction coefficient should be also a function of the roughness.
6.2. ENERGY LOSS BY SKIN FRICTION FOR FLOW IN PIPES 115

More commonly, the Fanning friction factor 1 ff (cf. fD = 4ff the Darcy friction factor ) has been
used by chemical engineers. Very similar to (6.19), the friction force can be expressed as
1 2
Ff = ff (Re) × ρ⟨v⟩ × Aw
| {z } 2 |{z} (6.20)
area on which friction acts
| {z }
Fanning friction factor
magnitude of the kinetic energy

Thus, the Fanning friction factor is also a function of the Reynolds number only and can be determined
by experiments as shown in Fig 6.4.
Ff /Aw τ
ff (Re) ≡ 1 2 = 1 2 (6.21)
2 ρ⟨v⟩ 2 ρ⟨v⟩

The symbols in Fig. 6.4 show representative data for ff obtained over a range of Re spanning four
orders of magnitude. In this log–log plot, there is a linear decline in the friction factor at relatively
low Reynolds numbers. At a critical value of the Reynolds number 2,100 a sharp rise occurs, followed
by a decline that is more gradual than the earlier one. What happens at the critical Reynolds number
is a transition from laminar to turbulent flow. In laminar flow all elements of the fluid move in
straight lines that are parallel to the tube wall, whereas in turbulence the pattern is chaotic, with
eddies of varying size continually forming and decaying. Such eddies greatly increase the shear stress.
The critical Reynolds number can be increased significantly if special precautions are taken to avoid
vibrations or other flow disturbances. Thus, where the laminar–turbulent transition will occur is not
entirely predictable, and in piping design it is prudent to avoid values of Re between 2,000 and 4,000.

Figure 6.4: Friction factor for smooth cylindrical tubes. The curves labeled Laminar, Prandtl–Kármán,
and Blasius are based on Eqs. (6.22), (6.23), and (6.24), respectively. The dashed lines are extrap-
olations. The symbols show data for water: circles, D = 1 cm from Koury (1995); filled and open
triangles, D = 5 cm and 10 cm, respectively, from Nikuradse (1932).

In the laminar region, the data could be well fitted by


16
ff = , (Re < 2, 100) (6.22)
Re
Surprisingly, this relation obtained from the experimental values exactly corresponds to the one derived
theoretically (see Example 31). There are some expressions on the friction factors for turbulent flow
in smooth tubes. The most widely accepted expression is the Prandtl–Kármán equation,
1 p
p = 4.0 log(Re ff ) − 0.4, (Re ≥ 3 × 103 ) (6.23)
ff
1 named after (J. T. Fanning, 1837-1911). Generally, the Fanning friction factor is used for chemical engineers whereas

civil and mechanical engineers prefer to use the Darcy fiction factor.
116 CHAPTER 6. APPLICATION OF MOMENTUM TRANSFER

This is accurate to within about 4% for Re up to 3 × 106 . In chemical engineering applications, Re in


pipes rarely exceeds. Even more convenient, but valid over a more limited range of Re, is the Blasius
equation
ff = 0.0791 Re−1/4 , (3 × 103 ≤ Re ≤ 1 × 105 ) (6.24)

Ex. 29. At Re = 1×104 , ff in turbulent flow is approximately five times what it would be if laminar
flow could be maintained. Explain why.

Answer: As shown in Fig. 5.8, the variation in velocity with radial position is parabolic in a laminar
flow, whereas in a turbulent flow the profile is more blunted. Thus, the higher shear rate at the wall
is what increases τ = Ff /Aw in turbulent flow (τrz in (4.76)) and ultimately leads to such an increase
in ff according to (6.20).
Turbulence can be costly in terms of energy usage. In a horizontal pipe, the required pumping power
equals Q∆P and ∆P is proportional to τ (or ff ). For certain other purposes, however, turbulence is
advantageous. For example, the increased mixing due to the turbulent eddies facilitates heat transfer
between the fluid and the pipe wall. That is helpful whenever a process stream must be heated or
cooled.

Wall Roughness
No matter how carefully fluid conduits are manufactured, there are always at least microscopic ir-
regularities on their surfaces. Irregularities can increase over time, as a result of corrosion, mineral
deposits, or the growth of microorganisms. Wall roughness tends to increase the friction factor at a
given Reynolds number, and ff is found to be governed by roughness alone when Re is sufficiently
large. The effects of roughness are noticeable only in turbulent flow. In that regime, where most of
the velocity variation occurs very near the wall, even small protrusions or indentations can affect the
key part of the velocity profile. In laminar flow the profile is much less sensitive to what happens in
the immediate vicinity of the wall.
What is needed to quantify the effects of roughness, at the very least, is the height of the irregular-
ities. This new length scale, denoted as k, gives rise to an additional dimensionless group k/D. Thus,
dimensional analysis indicates that ff = ff (Re, k/D). The effect of wall roughness on the friction
factor is shown in Fig. 6.5.

Figure 6.5: Friction factor for circular pipes. k is the roughness parameter.
6.2. ENERGY LOSS BY SKIN FRICTION FOR FLOW IN PIPES 117

6.2.2 Relation between Pressure Drop and Friction Factor


Because ∆P is easier to measure and pumps are essentially devices that boost the pressure at selected
places in piping networks, values of ∆P are needed in design. For the horizontal pipe, the conservation
of linear momentum gives
2 2
ρ⟨v⟩ L ρ⟨v⟩
P1 A − P2 A − Ff = 0 =⇒ ∆P A = Ff = ff Aw =⇒ ∆P = 4ff (6.25)
2 D 2
On the contrary, the conservation of linear momentum for a upwarding flow in the vertical pipe
becomes2
2 2
ρ⟨v⟩ L ρ⟨v⟩
P1 A−P2 A−Ff −mg = 0 =⇒ ∆P A = Ff +ρALg = ff Aw =⇒ ∆P = 4ff +ρgL (6.26)
2 D 2

P2

P1 P2 Ff g

Ff
P1

Figure 6.6: Total force balances.

In terms of the dynamic pressure, both of (6.25) and (6.26) become


2
L ρ⟨v⟩
∆P = 4ff (6.27)
D 2
In fact, Eq. (6.27) is true for a cylinder inclined at any angles. For a horizontal one, ∆P = ∆P ,
∆P = ∆P + ρg∆h for a vertical one (where ∆h = h1 − h2 ).

Ex. 30. [Pressure drop for water in process pipes] It is desired to find the values of ∆P for
water flow at 20 ◦ C through either of two pipes. One is entirely horizontal and the other has horizontal
and vertical segments. In each, the flow rate is Q = 8.0 × 10−3 m3 /s, the diameter is D = 0.1 m, and
the total length is L = 30 m. The second pipe has a bend, such that flow in the first 20 m is horizontal
and that in the last 10 m is upward. Here ρ = 1.00 × 103 kg/m3 and ν = 1.00 × 10−6 m2 /s. Compare
the pressure drop for each pipe.

Answer: The mean velocity and the Reynolds number are


Q D⟨v⟩
⟨v⟩ = = 1.02 m/s, Re = = 1.02 × 105 (6.28)
πD2 /4 ν
Using the Blasius equation (6.24), the friction factor is
ff = 0.0791(1.02 × 105 )−1/4 = 4.43 × 10−3 (6.29)
For the entirely horizontal one, the pressure drop
2
L ρ⟨v⟩ 30 (1 × 103 )(1.02)2
∆P = 4ff = 4(4.43 × 10−3 ) = 2.76 kPa (6.30)
D 2 0.1 2
On the other hand, for the vented one,
2
L ρ⟨v⟩
∆P = 4ff + ρgL = 2.76 kPa + (1 × 103 )(9.8)(10) Pa = 1.01 × 102 kPa (6.31)
D 2
2 마찰력과 중력을 이겨내고 유체를 흐르게 하는 것은 압력이다. 아래로 향한 flow 경우 압력과 중력이 마찰력을 이겨내고 흐르게

된다.
118 CHAPTER 6. APPLICATION OF MOMENTUM TRANSFER

Ex. 31. For the steady-state laminar flow of an incompressible Newtonian fluid in a circular pipe,
show that the friction factor can be obtained theoretically as

16
ff = (6.32)
Re

Answer: Using the Hagen-Poiseuille (5.25) and (6.27), we have

∆P D 32µ⟨v⟩ D 16µ
ff = 2 = 2 2 = (6.33)
L 2ρ⟨v⟩ D 2ρ⟨v⟩ ρ⟨v⟩D

6.2.3 Noncircular Cross-Sections


Turbulent Flow
Many conduits are not cylindrical. For example, heating and ventilating ducts often have rectangular
cross-sections, and the flow in certain heat exchangers is in the annular space between two tubes. A
remarkable finding is that, for turbulent flow, the results for circular cross-sections can be applied to
noncircular ones, provided that an appropriate effective diameter is used. What works is the hydraulic
diameter,
4A
DH = (6.34)
C
where C is the wetted perimeter, is the length of wall within a cross section that is in contact with the
fluid. It can be√easily proven that DH = D for a cylindrical tube, DH = 2ab/(a + b) for a rectangular,
and DH = h/ 3 for a triangular (see Table 2.1 in Deen’s book).
For noncircular cross-sections, D is replaced by DH both in the friction factor and in the Reynolds
number.
2
L ρ⟨v⟩ ρ⟨v⟩DH
hf s = ∆P = 4ff , ReH = (6.35)
DH 2 µ
This proves to be quite accurate for turbulent flow, as shown in Schlichting (1968, p. 576) for square,
rectangular, triangular, and annular channels.
One reason this approach works well is that, for any conduit of uniform cross-section, DH arises
naturally from an overall force balance. That is,
2 2 2 2
ρ⟨v⟩ ρ⟨v⟩ CL ρ⟨v⟩ L ρ⟨v⟩
(∆P )A = Ff = ff Aw = ff CL =⇒ ∆P = ff = 4ff (6.36)
2 2 A 2 DH 2

That is true whether the flow is laminar or turbulent. What is special about turbulent flow is that the
velocity profile very close to the wall has a nearly universal form. Because the profile near the wall is
insensitive to the channel shape, so is the relationship between ff and ReH .

Laminar Flow
Similar to the friction factor for a laminar flow in a circular pipe, the friction factor for a laminar flow
in noncylindrical conduits can be predicted from first principles as
c
ff = (6.37)
ReH

the c usually has a value other than 16. For a parallel-plate channel c = 24, and for an equilateral
triangular channel c = 40/3. For a rectangular conduit with α = a/b ≤ 1
 ∞ −1  
24 X tanh(λn /α) 1
c= 1 − 6α , λn = n+ π (6.38)
(1 + α)2 n=0
λ5n 2
6.3. ENERGY LOSS BY OTHER FRICTIONS 119

6.2.4 Skin Frictional Energy Loss in Terms of Friction Factor


Regardless of a straight horizontal or vertical pipe flow (Fig. 6.6), the frictional energy loss by the
skin friction is obtained from (6.10) and (6.27) :
2
L ρ⟨v⟩
hf s = ∆P + ρg∆h = ∆P = 4ff (6.39)
D 2
where P = P + ρgh is dynamic pressure.

6.3 Energy Loss by Other Frictions


6.3.1 Energy Loss by Form Friction from Changes in Velocity or Direction
Whenever the velocity of a fluid is changed, in either direction or magnitude, friction is generated
in addition to the skin friction resulting from flow through the straight pipe. Such friction includes
form friction resulting from vortices that develop when normal streamlines are disturbed and when
boundary layer separation occurs. If the cross section of the pipe is suddenly enlarged, the fluid stream
separates from the wall and issues a jet into the enlarged section. The jet then expands to fill the
entire cross section of the larger conduit. The space between the expanding jet and the conduit wall is
filled with fluid in vortex motion characteristic of boundary layer separation, and considerable friction
is generated within this space. This effect is shown in Fig. 6.7.

Figure 6.7: Flow at sudden enlargement of cross section.

The frictional energy loss hf e from a sudden expansion of cross section is proportional to the
kinetic energy of the fluid in the small conduit and can be written as
2
ρ⟨va ⟩
hf e = Ke (6.40)
2
where Ke is called the expansion loss coefficient. In this case the calculation of Ke can be made
theoretically and a satisfactory results obtained. Consider the control volume defined by sections AA
and BB in Fig. 6.7. Gravitational forces do not appear because the pipe is horizontal. The wall friction
is negligible because the wall is relatively short and there is almost no velocity gradient at the wall
between the sections. The conservation of linear momentum (6.41) gives
X
Ṗout − Ṗin = F =⇒ ṁ(βb ⟨vb ⟩ − βa ⟨va ⟩) = Pa Sa − Pb Sb (6.41)

where β is call the momentum correction factor 3 , which is analogous to the kinetic energy correction
3 This can be determined in a very similar way to (6.4) and defined as
⟨v 2 ⟩
β= (6.42)
⟨v⟩2
120 CHAPTER 6. APPLICATION OF MOMENTUM TRANSFER

factor. From the Bernoulli equation (6.10), we get


2 2
ρ⟨vb ⟩ ρ⟨va ⟩
Pa − Pb = αb − αa + hf e (6.43)
2 2
For usual flow conditions, αa = αb = 1 and βa = βb = 1 because the local velocity at most of the cross
section is close to the mean velocity in a usual turbulent flow. These correction should be switched if
the type of flow differs. Using these give
 2
Sa
Ke = 1 − (6.44)
Sb

When the cross section of the conduit is suddenly reduced, the fluid stream cannot follow around
the sharp corner and the stream breaks contact with the wall of the conduit. A jet is formed, which
flows into the stagnant fluid in the smaller section. The jet first contracts and then expands to fill the
smaller cross section, and downstream from the point of contraction the normal velocity distribution
eventually is reestablished. The cross section of minimum area at which the jet changes from a
contraction to an expansion is called the vena contracta. The flow pattern of a sudden contraction is
shown in Fig. 6.8. Section CC is drawn at the vena contracta. Vortices appear as shown in the figure.

Figure 6.8: Flow at sudden contraction of cross section.

The frictional energy loss hf c from a sudden contraction of cross section is proportional to the
kinetic energy of the fluid in the smaller conduit and can be written as
2
ρ⟨vb ⟩
hf c = Kc (6.45)
2
where Kc is called the contraction loss coefficient. Experimentally, for laminar flow, Kc < 0.1 so that
hf c is negligible. For turbulent flow, Kc is given by the empirical equation:
 
Sb
Kc = 0.4 1 − (6.46)
Sa

6.3.2 Energy Loss by Fitting and Valves


Fittings and valves disturb the normal flow lines and cause friction. In short lines with many fittings,
the frictional energy loss from the fittings may be greater than that from the straight pipe. The
frictional energy loss from fittings is found from an equation similar to (6.40) and (6.47):
2
ρ⟨va ⟩
hf f = Kf (6.47)
2
The frictional energy loss factor for fitting is empirically determined. The values for standard elbows
of 90◦ is 0.75, whereas half open gate valve is about 4.5.
6.4. BERNOULLI EQUATION WITH A PUMP AND FRICTION 121

6.3.3 Total Frictional Energy Loss in the Bernoulli Equation


Consider the flow of incompressible fluid through the two enlarged headers, the connecting tube,
and the open globe valve shown in Fig. 6.9. When skin friction in the entrance and exit headers is
neglected, the total frictional energy loss is
  2
L ρ⟨v⟩
hf = 4ff + Kc + Ke + Kf (6.48)
D 2
This should be used in the Bernoulli equation (6.10).

Figure 6.9: Flow of incompressible fluid through typical assembly.

6.4 Bernoulli Equation with a Pump and Friction


If a pump is installed to push the flowing fluid (to increase the mechanical energy of the flowing fluid),
then the Bernoulli equation becomes
2 2
ρ⟨v1 ⟩ ρ⟨v2 ⟩
P1 + α1 + ρgh1 + ηWp = P2 + α2 + ρgh2 + hf (6.49)
2 2
where η is a pump efficiency and ηWp is the mechanical energy per unit volume delivered to of
the flowing fluid by a pump. The performance of a pump is generally shown in its characteristics
performance curve where its capacity (flow rate) is plotted against its total developed head TDH [m],
efficiency [%], required input power [W], and net positive suction head NPSH [m].

Ex. 32. A pump draws a solution of specific gravity 1.84 from a storage tank shown in Fig. 6.10. Its
efficiency is 60%. The velocity in the suction line is 0.914 m/s. Friction losses in the entire system are
29.9 J/kg. (a) What is the power delivered to the fluid by the pump? and (b) What pressure must
the pump develop? Here the flows is assumed to be turbulent so that the correction factors are nearly
unity.

5.0 cm
15.2 m

7.5 cm

Figure 6.10: Flow diagram.


122 CHAPTER 6. APPLICATION OF MOMENTUM TRANSFER

Answer: (a) Take a and b


ρ
ηWp = (Pb − Pa ) + (vb 2 − va 2 ) + ρg(hb − ha ) + hf (6.50)
2
Since
 2
dsuc
va = 0, vb = vsuc = 2.057 m/s, hb − ha = 15.2 m, and Pa = Pb = Patm (6.51)
db

we have
1 ρvb 2
 
Wp = + ρg(hb − ha ) + hf (6.52)
η 2
The power delivered to the pump is the produce of Wp and the volume flow rate:

Ab vb ρvb 2
 
P = V̇ Wp = + ρg(hb − ha ) + hf = 2.24 kW (6.53)
η 2

and the power delivered to the fluid is then

ηP = 0.6 × 2.24 = 1.34 kW (6.54)

(b) In order to get the pressure developed by the pump, we choose two points just before and after
the pump and apply the Bernoulli’s equation.
ρ
Paf − Pbe = (vbe 2 − vaf 2 ) + ηWp = 330 kN/m2 (6.55)
2
because
hbe = haf , and hf = 0 (6.56)

Total Developed Head(TDH, 총괄개발두)


In the hydraulics literature the velocity and pressure terms in the Bernoulli equations are often
expressed as equivalent height, called heads [m]. Such heights are obtained by dividing each term
by ρg. Thus, the velocity and pressure head are v 2 /2g and P/ρg, respectively and either may be
compared with the height h. The total head denoted by H is the sums of velocity head, pressure head,
and heights.
P v2
H≡ + +h (6.57)
ρg 2g

Figure 6.11: Pump flow system


6.5. BUOYANCY AND DRAG FORCES 123

the total developed head (TDH) is a head(or pressure) increased by a pump only. The Bernoulli
equation can be written between stations a and b in Fig. 6.11 as

Pa va 2 ηWp Pb vb 2 hf (= 0)
+ Za + + = + Zb + + (6.58)
ρg 2g ρg ρg 2g ρg
where hf is introduced due to fluid friction (any conversion of mechanical energy to heat in a flowing
system). Since the only friction is that occurring in the pump itself and is accounted for by η, hf = 0.
Then (6.58) becomes

vb 2 va 2
   
Pb Pa ηWp
TDH = ∆H = Hb − Ha = + Zb + − + Za + = (6.59)
ρg 2g ρg 2g ρg

Power requirement
The power supplied to the pump drive from an external source is denoted by PB :
Qρg∆H
PB = QWp = (6.60)
η

Net positive suction head (NPSH, 유효흡입두)


If the suction pressure is only slightly greater than the vapor pressure, some liquid may flash to vapor
inside the pump, a process called cavitation (공동화), which greatly reduces the pump capacity and
causes severe erosion.
To avoid cavitation, the pressure at the pump inlet must exceed the vapor pressure, called the net
positive suction head. The required value of NPSH is about 2∼3 m for small centrifugal pumps and
15 m for large pumps.
The Bernoulli equation can be written between stations a′ and a in Fig. 6.11 as

Pa′ va′ 2 Pa va 2 hf
+ Z a′ + = + Za + + (6.61)
ρg 2g ρg 2g ρg
Since va′ = 0 and Za′ = 0, this becomes

Pa Pa ′ va 2 hf
= − Za − − (6.62)
ρg ρg 2g ρg
and letting Pa /ρg = Pv /ρg + NPSH gives

Pa′ − Pv va 2 hf
Za ≈ − − − NPSH (6.63)
ρg 2g ρg
where Pv is the vapor pressure. In general, the velocity term can be neglected because it contributes
very little to the NPSH compared to the others.

6.5 Buoyancy and Drag Forces


In Ch. 4 and Ch. 5, we have discussed the forces acting on fluid by surrounding. On the contrary, we
will discuss the forces acting on solid by a surrounding fluid in this section. When an object freely
drops in air, the gravitational force of the object competes with the buoyancy and drag forces that are
upwards. Now let’s discuss them. In summary, they are originated from the surface forces, pressure
and viscous force, acting on the object by the fluid, air in this case.
As shown in Fig. 6.12, the surface forces acting on surface by a fluid are composed of the pressure
force in the normal direction to the surface and the viscous force in both the normal and tangential
directions to the surface.
Z Z Z Z Z
Fs = t dS = tP dS + tτ dS = − nP dS + τ · n dS (6.64)
S S S S S
124 CHAPTER 6. APPLICATION OF MOMENTUM TRANSFER

tτt t

tp+tτn
v

Fluid

Figure 6.12: The surface force acting on a falling sphere by a surrounding fluid.

where tP and tτ are portions of the stress vector caused by pressure and viscous forces, respectively.
It should be noticed that these are counteractive to the force acting on the fluid by the sphere. In
addition, we used the pressure and viscous stress vectors expressed as

tP = −nP, and tτ = τ · n (6.65)

because of its opposite direction to pressure and using (4.52) and (4.60).
As seen from (5.152), the movement of surrounding fluid makes a change in the pressure acting
on an object placed in the moving fluid4 (Fig. 6.13):
"  2 #
3  µv∞  R
P (r, θ) = P∞ − ρgr cos θ + − cos θ (6.66)
| {z } 2 R r
Pstat | {z }
Pmov

Thus, contributing to the pressure force are the sum of static pressure variations, which ultimately

z
P∞ - ρgR
r

FPstat FPmov Fτ
R P∞ P∞
Form drag Wall drag

Moving
v∞ P∞ + ρgR
fluid
3μv∞
3μv 3μ
3μvv∞
Pstat = P∞ - ρgR cosθ Pmov = - cosθ τ= sinθ
2R 2R

Figure 6.13: Pressure and viscous forces acting on a sphere by a creeping flow past the sphere.

create the buoyancy force, and pressure variations caused by the fluid motion. Finally the total surface
force acting on the sphere by the fluid can be written as
Z Z Z
FS = −nPstat dS + −nPmov dS + τ · n dS
S S S
= FP stat + FP mov + Fτ (6.67)
| {z } | {z }
Fbuoy = surface force due to FD = surface force due to
the difference in static pressure the movement of a fluid
4 When air blows, the pressure on a sphere can be written as
3 0.018 × 10−3 Pa · s × v∞
P (R, θ) = P∞ − 1.293 kg/m3 × 9.8 m/s × R cos θ − cos θ
2 R
v∞
≈ 101.3 × 103 − 12.7R cos θ − 2.7 × 10−5 cos θ [Pa]
R
6.5. BUOYANCY AND DRAG FORCES 125

6.5.1 Buoyancy Force


As mentioned in (6.67), there exists the surface force on an object caused by the static pressure
variations when an object is immersed in a fluid. Suppose that a cylindrical can is fully immersed in
a static water as depicted in Fig. 6.14. The total pressure force acting on the cylinder by water is

a FP1 V
water (ρ)
A P1 Fbuoyancy = FP = ρgV
ρp
Pside
h FP3

Fbody

P2

FP2
https://www.britannica.com/science/Archimedes-principle

Figure 6.14: Buoyancy forces acting on a cylinder fully immersed in water and on a boat partially
immersed in water.

FP stat = FP 1 + FP 2 + FP 3 = −(P0 + ρga)Aez + [P0 + ρg(a + h)]Aez = ρgV ez = Fbuoy (6.68)


R
because FP 3 = −nPside dS = 0. An object of volume V surrounded by a fluid of constant density ρ
always experiences an upward force, ρgV due to pressure increase (or difference) with depth.
It is called the buoyancy force (부력), which is equal to the weight of the displaced fluid.
In the case of gases with tiny density, it is ordinarily negligible. If the buoyancy force is larger than
the gravitational force, then the cylinder will rise to the height where both are the same. Thus, the
Archimedes’ law is straightforward; the upward buoyant force that is exerted on a body immersed in
a fluid, whether fully or partially submerged, is equal to the weight of the fluid that the body displaces
(wikipedia). A small steel will sink sink in water. On the contrary, a ship made by steel will float
because of its shape. The ship displaces more water than the small steel.

6.5.2 Drag Force


As mentioned in (6.67), there exists the surface force on an object caused by the relative motion of
a surrounding fluid: tangential and normal viscous forces (Fτ ) and normal pressure force (FP mov ).
The drag on an object is the component of the fluid-dynamic force, FP mov + Fτ , that resists its
translational motion. Using the (6.67), the drag FD can mathematically be expressed as
FD = ez · (FP mov + Fτ ) = FDf orm + FDwall (6.69)
Here the positive z direction is an opposite direction of the moving solid, which is identical to the
direction of the moving fluid if the solid is stationary.
Locally, pressure acts perpendicular to the surface, as shown in Fig. 6.13. Because flow tends to
elevate the pressure on the upstream side, the net effect is a force in the z direction. There is also
a viscous stress acting tangent to the surface. The net effect of this is again in the z direction. The
pressure and viscous contributions are customarily referred to as form drag (or pressure drag) and
wall drag (or skin friction drag), respectively. When the velocity of the sphere or (the fluid) is large,
several vortices are created behind the sphere and form extra friction(energy loss), which is also called
form drag. See the following section.
126 CHAPTER 6. APPLICATION OF MOMENTUM TRANSFER

6.5.3 Drag in a Flow Past Solid Surfaces


The drag grows as the velocity difference between object and the surrounding fluid increases. This is
important in applications ranging from molecular diffusion to ship an aero dynamics. Consider a flow
past a solid sphere as shown in Fig. 6.15, which is identical to the moving sphere in a stationary fluid.
From the stagnation point (v = 0), the boundary layer on the surface of the sphere gradually grows.

n
n
Moving tio
tio rf ic
ric
fluid
in
f
in

Sk
sk

Wall drag
Stationary Moving
fluid Stationary Form drag
sphere Form drag
sphere

Stagnation
Drag = Form drag + Wall drag point

Figure 6.15: Flow past a solid sphere

Skin friction appears in the unseparated boundary layer. When the fluid reaches the backside of the
sphere, its momentum prevents it from making the sharp turn around the sphere, and it separates
from the sphere and proceeds outward into the bulk of the fluid if the velocity of the fluid is enough
fast. Behind the sphere is a backwater zone of strongly decelerated fluid, in which large eddies, called
vortices, are formed. This zone is known as the wake. The eddies in the wake are kept in motion by
the shear stresses between the wake and the separated current. They lead to a large pressure lose in
the fluid and thus form large form friction drag.
When a fluid flows past a parallel thin flat plate, there appears only skin friction drag on the two
sides of the plate. For a time after the fluid leaves the plate, the layers and velocity gradient presist.
Soon, however, the gradients fade out, the boundary layers intermingle and disappear, and the fluid
once more moves with a uniform velocity. There occurs no form drag by eddies. When the fluid flows
past the vertical plate, there occurs large vortex, leading to large form drag.

Skin friction drag

Form drag

Figure 6.16: Flow past a flat plate: (a) parallel with plate (skin friction) (b) perpendicular to plate
(form).

The form drag is dependent upon the shape of an object. More generally, the drag is dependent
6.5. BUOYANCY AND DRAG FORCES 127

of the velocity, density, and viscosity of the fluid, and the characteristic length (e.g. diameter):

FD = f (v, ρ, µ, Dp ) (6.70)

By the dimensional analysis (see Sect. E.3), we are able to significantly reduce number of experiments
to obtain the relation (6.70) between the drag and other variables. The dimensional analysis has
enabled us to relate the original five variables in terms of only two dimensionless parameters in the
form
FD /D2
= f¯(Rep ) (6.71)
ρv 2

where Rep = ρvDp /µ indicates the Reynolds number of the solid particle. It is clear that the drag
coefficient is a function of the Reynolds number only.

6.5.4 Drag Coefficient (항력계수)


In the study of the flow past a solid surface, especially turbulent flow, the drag coefficient CD is very
useful. From (6.71), the drag force exerted on the solid surface can be generally expressed as

1 2
FD = CD (Re) × ρv × AP
|2 {z } (6.72)
| {z } |{z}
drag coefficient projection area
magnitude of the kinetic energy

where AP is the area of the solid projected to the direction of flow. Ap (cylinder) = LDp , Ap (sphere) =
πDp 2 /4.
In addition, the drag coefficient is a function of the Reynolds number only:

FD /AP
CD ≡ 1 2
= CD (Rep ) (6.73)
2 ρv

For regular shapes such as spheres or cylinders at low fluid velocities, the flow patterns and drag
forces can be estimated by the linear momentum balances. For high velocities and irregular shapes
they are most easily determined by experiments. Figure 6.17 shows the drag coefficient measured
experimentally for regular shapes as a function of the Reynolds number.

Figure 6.17: Drag coefficient measured experimentally for spheres, disk, and cylinder as a function of
Reynolds number.
128 CHAPTER 6. APPLICATION OF MOMENTUM TRANSFER

Figure 6.18: Types of behavior for the flow around a sphere. Regions of turbulent flow are shaded in
gray

Sphere
Figure 6.18 shows types of behavior for the steday-state incompressible flow of a Newtonian fluid past
a sphere.

Regime I: Rep ≪ 1 (Creeping flow), the viscous forces predominate compared to the inertia forces and
affect at considerable distance from the cylinder. No eddies occur.

FD /AP 3πµvDp /(πDp2 /4) 24


CD = 1 2
= 1 2
= ∵ FD = 3πµvDp (6.74)
2 ρv 2 ρv Re p

where the drag has been calculated from the surface force by viscous and pressure forces obtained
in Sect. 5.9 (see more details in p. 125 of Bird’s book or Ch. 8 of Deen’s book.). More exactly,
it can be obtained from the surface integration of the viscous and pressure forces (5.153) and
(6.69).
Regime II: 1 <Rep < 103 , At Rep =10, a pair of vortices appears behind the cylinder. The type of flow
persists up to about Rep =40, when there appear two separation points (about 85◦ from the
front stagnation point) at which the streamlines separate from the solid surface. With further
increase in Rep , the vortices separate regularly from alternate sides of the cylinder (vortex street
in Figure 6.19).
Regime III: 103 <Rep < 2 × 105 (Constant CD region), This constancy of CD implies that viscous stresses
(skin friction drag) are negligible and that the drag (form drag) is proportional to v 2 . As it was
hypothesized by Newton that the drag on any object should be proportional to the fluid density,
the square of the velocity, and a characteristic area, this is called the Newton’s-law regime.
CD
FD /AP = cρv 2 , c= = constant (6.75)
2
6.5. BUOYANCY AND DRAG FORCES 129

Figure 6.19: Vortex street

When Rep > 2500, the wake is no longer characterized by large eddies. The flow on the surface
of the body from the front stagnation point to the separation point is laminar, and the shear
stress in this interval is appreciable only in a thin boundary layer. The drag coefficient remains
constant, approximately 0.445 for a sphere.
Regime IV: As Rep increases, the front boundary layer becomes turbulent and the separation point moves
toward the rear of the sphere (about 140◦ )and the wake shrinks. The remarkable drop in the drag
coefficient is the result of this decrease in the size of the wake and the corresponding decreases
in form drag. This is called Eiffel phenomena.5
This is why golf balls have lots of dimples on their surface to make turbulent flow (300∼500,
0.175 mm depth) .

Figure 6.20: Dimples of a golf ball (300 ∼ 500, 0.175 mm depth) decrease the form friction by decreasing
the areas of wakes.

Disk
Some of experimental equations are available for the drag coefficients of disks.
 
20.4
CD = 1 + Re0.792 , Re ≤ 133
Re (6.76)
CD = 1.17, Re > 133
The Eiffel phenomenon does not appear.
5 The phenomenon of the sudden change of sphere drag was first observed in a rather amusing way. Prandtl in

Göttingen and Eiffel in Paris measured the drag of the sphere: Prandtl obtained a value for the drag coefficient which
was more than twice that obtained by Eiffel. They exchanged information, and one of the young engineer in Prandtl’s
laboratory said, “Oh, M. Eiffel forgot a factor of two. He calculated the coefficient referred to ρv 2 , not 12 ρv 2 .’’ This
remark somehow reached Paris and the elderly M. Eiffel became very angry. He then measured the drag for a wider range
of Reynolds numbers and discovered that a sudden decrease in the drag coefficient occurred beyond a certain Reynolds
number. But he did not find the physical reason for the sudden change. It was Prandtl who gave the explanation
mentioned above. He put a fine wire ring around a sphere a short distance in front of the separation points of the
laminar layer. He found out that the total drag was reduced by the presence of the wire because laminar separation
was prevented
- Theodore Von Karman, “Aerodynamics: Selected Topics in the Light of Their Historical Development,” Dover
Publications, New York (2004).
130 CHAPTER 6. APPLICATION OF MOMENTUM TRANSFER

Cylinder
As shown in Fig. 6.17, a nearly linear decline in a log–log plot is followed by a leveling off. The Eiffel
phenomenon occurs, CD declining suddenly from 1.2 to 0.3 at Re = 2 × 105 . Thereafter, CD increases
gradually.
For Re below the Eiffel threshold,

4 × 10−4 Re
 
6.8 1.96
CD = 1.18 + + − , Re < 2 × 105 (6.77)
Re0.89 Re1/2 1 + 3.64 × 10−7 Re2

Flat plates
Consider a thin plate parallel to the flow (The same category is a long cylinder aligned end-on, for
which A|| = DL ≫ A⊥ = πD2 /4) as shown in Fig. 5.4. Since A⊥ is negligible, pressure cannot act
in the flow direction and form drag will be absent. Thus, a thin plate parallel to the flow experiences
only skin friction (or wall) drag.
As will discussed in Sect. 6.2, the skin friction drag can be expressed as
1
FD = Cf (Re) × ρv 2 × LW (6.78)
2
where L and W are the length and width of the flat plate. It should be clearly noticed that Cf is an
average value since the Reynolds number varies with the distance from the entrance, x (Rex = ρvx/µ).
As derived in Ch. 9 of Deen’s book, if the boundary layer is entirely laminar and ReL > 100, then

Cf = 1.328 Re−1/2 (6.79)

If there are both laminar and turbulent regions, then


0.455 B
Cf = 2.58
− (6.80)
(log Re) Re

The value of B depends on where the transition occurs, with B = 1050 for Ret = 3×105 and B = 8700
for Ret = 3 × 106 .

Ex. 33. [Drag on a cylinder in water] calculate the drag per unit length on a long cylinder due
to water flow perpendicular to its axis. The diameter is D = 0.10 m, the relative velocity is U = 1.0
m/s, ρ = 1.0 × 103 kg/m3 , and µ = 1.0 mPa·s.

Answer: The Reynolds number is

ρDU (1)(0.1)(1.0 × 103 )


Re = = = 1.0 × 105 (6.81)
µ 1.0 × 10−3

The projected area is AP = DL and, from Eq. (6.77), CD = 1.18. The drag per unit length is then

FD 1 1.0 × 103 12 0.1


= CD × ρU 2 × D = 1.18 × = 59 N/m (6.82)
L 2 2

Ex. 34. [Comparative drag on a cylinder and a flat plate] Compare the drag on a long cylinder
to that on a flat plate of equal surface area, for the conditions in the above example. Imagine that
the cylinder is flattened into a plate, giving a length in the flow direction that equals half the cylinder
circumference (Lp = πD/22) and a width that equals the cylinder length (W = Lc ). It is desired to
compare FD /W for the plate to FD /Lc for the cylinder.

Answer: For D = 0.1 m, the length of the equivalent plate is Lp = π(0.1)/2 = 0.157 m and the plate
Reynolds number is
ρLp U (1)(0.157)(1.0 × 103 )
Re = = = 1.57 × 105 (6.83)
µ 1.0 × 10−3
6.5. BUOYANCY AND DRAG FORCES 131

Because this is less than 3 × 105 , the boundary layer is entirely laminar. Accordingly,
Cf = 1.328 Re−1/2 = 1.328 (1.57 × 105 )−1/2 = 3.35 × 10−3 (6.84)
Allowing for boundary layers on both sides of the plate, the drag per unit width is
FD
= ρU 2 Lp Cf = (1 × 103 )(1)2 (0.157)(3.35 × 10−3 ) = 0.53 N/m (6.85)
W
This is only about 1% of 59 N/m, the value of FD /Lc from the above example. Assuming that the
flat-plate drag roughly equals the friction drag on the equivalent cylinder, this suggests that almost
the entire drag on a cylinder at large Re is form drag. Pressure measurements and more precise
calculations have confirmed that.

6.5.5 Terminal Velocity


Consider a particle is falling in the gravitational field as depicted in Fig. 6.21.

Fbuoy (buoyancy) = ρgV

ρp v FD(drag) = CD 1/2ρv2 Ap

Fg(gravity) = mg = ρp gV

Figure 6.21: Force balance of a falling particle (움직이는 고체에 작용하는 힘=중력+부력+항력= ma).
즉 움직이는 고체의 속도의 변화는 중력, 부력, 항력의 합에 의하여 결정된다. 힘이 합이 0이면 고체의
속도는 일정하게 된다.

The Newton’s second law of the particle (or the force balance) says that
ρv 2
 
dv ρ
m = Fg − Fbuoy − FD = mg 1 − − CD Ap
dt ρp 2
(6.86)
CD ρv 2 Ap
 
dv ρ
=⇒ =g 1− −
dt ρp 2m
For simplicity, the positive velocity of the particle corresponds to the direction of the gravitational force
(downward). For a centrifugal field the gravitational acceleration is changed to centrifugal acceleration
rω 2 . What if we drop a ball at a very high place? From (6.86) It can be seen that the acceleration
decreases with time and approaches zero because g is constant while the drag always increases with
velocity. Thus the particle quickly reaches a constant velocity (dv/dt = 0), which is called the terminal
velocity vt . In other words, the terminal velocity is the velocity of a particle whose gravitational force
is equal to sum of the buoyancy force and the drag force.
Let the velocity of a particle relative to the fluid be v. A reference frame is chosen in which the
sphere is stationary and the fluid velocity is v. At the situation of the terminal velocity (no change in
the velocity) by taking dv/dt = 0 in (6.86), we have
s
2g(ρp − ρ)m
vt = (6.87)
CD ρρp Ap

1. Settling of spherical particles


For spherical particles with diameter Dp
1 1
m= πDp3 ρp and Ap = πDp2 (6.88)
6 4
132 CHAPTER 6. APPLICATION OF MOMENTUM TRANSFER

and thus s
4g(ρp − ρ)Dp
vt = (6.89)
3CD ρ
Generally, the drag coefficient is determined by experiments. However for the following limiting
cases it can be obtained directly.
• Very low Rep : The drag coefficient at very low Reynolds numbers can be expressed by the
Stokes’ law as 24/Rep as seen in (6.74) and the Stokes terminal velocity becomes
s
4g(ρp − ρ)Dp ρDp vt gDp2 (ρp − ρ)
vt = =⇒ vt = (6.90)
72ρµ 18µ

In liquids, this expression applies for Dp as small as a few nm. For particles that small
in gases, the continuum approximation breaks down and a correction factor involving the
Knudsen number is needed (see p. 64 in. Deen’s book).
• Very high Rep : For 1,000<Rep <200,000, the drag coefficient is approximately 0.44 as
shown in Fig. 6.17. The terminal velocity becomes
s
g(ρp − ρ)Dp
vt = 1.75 (6.91)
ρ

2. Settling of nonspherical particles


Since CD and Ap of nonspherical particles is greater that those of spherical particles, the terminal
velocity of nonspherical particles is less than that of spherical particles.
3. Bubble or drops
Unlike solid particles, liquid drops or gas bubbles can change shape as they fall in the gravita-
tional field. Form drag by pressure tends to flatten the drops, but the surface tension opposes
this force.
(a) Dp < 0.5 µm: Since Dp is small, the drag expressed by FD = CD ρv 2 /2 × πDp 2 /4 is not
enough to change the shape. Therefore, the drops are falling without changing their shapes
and have about the same drag coefficients and terminal velocity as solid particles.
(b) 1 < Dp < a few mm: The drops are somewhat flattened in the direction of flow and thus
fall more slowly that a sphere of the same volume.

Ex. 35. Estimate the terminal velocity for 80- to 100-mesh particles of limestone (ρp = 2, 800 kg/m3
falling in water at 30 ◦ C. (ρ = 995.7 kg/m3 , µ =0.801 cP).

Answer: Since DP for 100-mesh = 0.147 and DP for 80-mesh = 0.175, the average of the particles is
0.161 mm.
First assume that Rep < 1, then the terminal velocity becomes from (6.90)
9.8 × (0.161 × 10−3 )2 (2800 − 995.7)
vt = = 0.032 m/s (6.92)
18 × 0.801 × 0.001
From this value, we can calculate Rep
995.7 × 0.161 × 0.001 × 0.032
Rep = = 6.4 (6.93)
0.801 × 0.001
Thus, we did a wrong guess. Let’s guess again that Rep = 2.5, then CD ≈ 20 from Fig. 6.17. The
terminal velocity from (6.89) becomes
r
4 × 9.8(2800 − 995.7) × 0.161 × 0.001
vt = = 0.138 m/s (6.94)
3 × 20 × 995.7
From this we can get Rep = 2.76. Therefore our guess is correct.
6.5. BUOYANCY AND DRAG FORCES 133

Ex. 36. A metal empty sphere (D=4.5 mm and m=0.05 g) is placed in a liquid with ρ=0.95 g/cm3 .
Determine a viscosity of the liquid and the moving direction of the sphere when the terminal velocity
is 4.0 mm/s.

Answer: Calculate the gravitational and buoyance forces.

950
Fg = 0.00005 × 9.8 = 0.00049 N, Fb = 0.00005 × 9.8 = 0.00044 N (6.95)
1048

where the density of the sphere is ρp = m/V = 0.05/(π/6 × 0.453 ) = 1.048 g/cm3 . Since Fb < Fg , the
sphere is moving down which corresponds to the gravitational direction.
Now assume that the Reynold number of the particle is very low, the terminal velocity becomes

9.8 × 0.00452 × (1048 − 950) kg


0.004 = =⇒ µ = 0.27 = 0.27 Pa · s (6.96)
18 × µ m·s

Now we have to check the Reynold number.

950 × 0.0045 × 0.004


Rep = = 0.063 < 1 (6.97)
0.27
Therefore our assumption is correct and thus

µ = 0.27 Pa · s = 2.7 P (6.98)

Approach to Terminal Velocity

Practically, the distance traveled during the acceleration can be roughly estimated by multiplying vt
and t0 , which is the time scale for the acceleration (see Eq. (6.101)). As an example, for a 1 mm sand
grain in air, t0 = 0.75 s, and vt = 7.4 m/s so that vt t0 = 5.6 m. The transient could be ignored in
calculating a settling time if the sand were falling from a height much greater than that.
A complication in applying Newton’s second law (6.86) is that, as the neighboring fluid is displaced
by an accelerating sphere, it too accelerates. The acceleration of the fluid tends to slow the acceleration
of the sphere, just as if the sphere mass were larger than its actual value. This can be accounted for
by replacing the sphere mass with an effective or virtual mass m∗ . The increment, or added mass, is
calculated most easily for potential flow, and the objective is to evaluate it for a solid sphere of radius
R. Assume that the sphere moves at a velocity U (t) in the z direction and that the inviscid fluid of
density ρ is otherwise at rest. The fluid-dynamic force on the sphere is (see Problem 9.5 of Deen’s
book )
2 dU
Fz (t) = − πR3 ρ (6.99)
3 dt
The negative sign confirms that displacing the surrounding fluid slows the sphere, and the coefficient
of dU/dt shows that the added mass is half the mass of the displaced fluid. For a sphere of volume V
the added mass is ρV /2, or half the mass of the displaced fluid. Accordingly, m∗ = (ρp + ρ/2)V .

ρv 2
 
dv ρ
m∗ = Fg − Fbuoy − FD = mg 1 − − CD Ap
dt ρp 2 (6.100)
=⇒ v(τ ) = vt tanh τ

where
t (ρp /ρ + 1/2)vt
τ= , and t0 = (6.101)
t0 |ρp /ρ − 1| g

Noting that tanh−1 (0.95) = 1.83, this indicates that the acceleration is 95% complete at τ = 1.83 or
t = 1.83 t0 .
134 CHAPTER 6. APPLICATION OF MOMENTUM TRANSFER

6.6 Flow in a Packed Bed


In many chemical processes, liquids or gases flow through beds of solid particles as shown in Fig.
6.22. For this system, it is very important to know how much pressure drops in the packed bed. The
most common method to determine the pressure drop in the packed bed is based on estimates of total
friction loss. To do so, we follow the procedures shown in Figure 6.22 with concepts of sphericity and
equivalent channel diameter.

Flow
v
S0 Vp Sp

L Deq
Φs (Vp const.)
(total S
const.)
Flow Dp
v
S0
L

Figure 6.22: Procedure to estimate pressure drop of the flow through a bed of solids

• Sphericity (Φs , 구형도)


Φs is defined as the ratio of surface area of a sphere whose volume is the same as the volume of
the solid to the actual surface area of the solid.
Ssphere
Φs ≡ <1
Sp
(6.102)
Ssphere /Vsphere 6/Dp
= =
Sp /Vp Sp /Vp
where Sp and Vp are actual surface area and volume of the solid particle.
• Equivalent channel diameter (Deq )
The number of channels n and their diameter Deq are determined by assuming that the total
surface area in the bed is the same as the surface area for n parallel channels of length L, where
A is the cross-sectional area of the bed
Stot = nπDeq L (6.103)
and
Stot = Sp × number of particles
6/Dp 6/Dp (6.104)
= Vp × number of particles = S0 L(1 − ε)
Φs | {z } Φs
volume occupied by all particles

where the void volume in the bed is the same as the total volume of the n channels.
πDeq 2
S0 Lε = n L (6.105)
4
Combining eqs. (6.103) through (6.105) gives
2Φs Dp ε
Deq = (6.106)
3(1 − ε)
6.6. FLOW IN A PACKED BED 135

• Average velocity in the channel v̄


v
v̄ = ∵ S0 v = S0 εv̄ (6.107)
ε
Because the pressure drop in a channel is the same as the pressure drop in the equivalent bed
composed of n channels, it can be expressed as (6.25). However, the correction factor λ should be
added to account for the fact that the channels are actually tortuous and not straight and parallel.
∆P 2ρv̄ 2
= λff (6.108)
L Deq
Case 1: Laminar flow. For flow at very low Reynolds number
∆P 2ρv̄ 2 32µv̄ 72λ1 µv (1 − ε)2
= λ1 ff = λ1 2 = (6.109)
L Deq Deq Φs 2 Dp 2 ε3
The Kozeny-Carman empirical equation: λ1 = 2.1 (approximate tortuosity of the channels),
∆P 150µv (1 − ε)2
= (6.110)
L Φs 2 Dp 2 ε3

Case 2: Turbulent flow. For flow at high Reynolds number


∆P 2ρv̄ 2 3ρv 2 (1 − ε)
= λ2 ff = λ2 ff (6.111)
L Deq Φs Dp ε3

The Burke-Plummer empirical equation (3λ2 ff ≈ 1.75):


∆P 1.75ρv 2 (1 − ε)
= (6.112)
L Φs Dp ε3
We may say λ2 = 1.75/0.03 = 58 if ff = 0.01, which is too large to explain the tortuosity of
the channels. In fact, the main contribution to the pressure drop measured by an experiment is
the kinetic energy losses caused by changes in channel cross section and flow direction. As the
fluid passes between particles, the channel becomes smaller and then larger, and the maximum
velocity is much greater than the average velocity. Since the channel area changes rapidly, most
of the kinetic energy of the fluid is lost as an expansion loss. This will be discussed later
The Ergun equation: an equation covering the entire range of flow rate can be obtained by summing
the viscous losses with the kinetic losses.
∆P 150µv (1 − ε)2 1.75ρv 2 (1 − ε)
= 2 2 3
+ (6.113)
L Φs Dp ε Φs Dp ε3
The first and second terms indicate the pressure drops by the viscous losses and the kinetic energy
losses, respectively.

Ex. 37. McCabe 7.1: A partial oxidation is carried out by passing air with 1.2 mole percent hydro-
carbon (Mw = 30) through 40-mm tubes packed with 2m of 3-mm catalyst pellets. The air enters at
350◦ C and 2.0 atm with a superficial velocity of 1 m/s. What is the pressure drop through the packed
tubes? Here the ΦS = 1 and µ = 3 × 10−5 Pa·s. How much would the pressure drop be reduced ny
using 4-mm pellets? Assume ε = 0.4.

Answer: The only variable we don’t have in (6.113) is the density of the air with 1.2 mole percent
hydrocarbon. Since
PM 2 × 30
P = cRT =⇒ ρ = = = 1.174 g/l (6.114)
RT 0.082 × 623
Thus, the pressure is
150 × 3 × 10−5 × 1 (1 − 0.4)2 1.75 × 1.174 × 12 (1 − 0.4)
 
2
∆P = 2 + = 18, 466 N/m (6.115)
12 × 0.0032 0.43 1 × 0.003 × 0.43
When Dp = 0.004, ∆P = 12, 798 N/m2 , 31% reduction.
136 CHAPTER 6. APPLICATION OF MOMENTUM TRANSFER

Ex. 38. The pressure drop through a particle bed can be used to determine the external surface
area and the average particle. Data for a bed of crushed ore particles show ∆P/L = 84 (lbf /in2 )/ft
for airflow at a superficial velocity of 0.015 ft/s. The measured void fraction is 0.47 and the estimated
sphericity Φ is 0.7. Calculate the average particle size and the surface area per unit mass if the densities
of air and solid are 0.00123 and 4.1 g/cm3 . The viscosity of air is 0.018 cP.

Answer: Assume Rep < 1, so that (6.110) applies:


s r
150µv (1 − ε)2 L 150 × 0.018 · 10−3 × 0.015 · 0.3048 (1 − 0.47)2 0.3048
Dp = 2 =
Φs ε3 ∆P 0.72 0.473 84 · 6895 (6.116)
= 5.99 µm
Check Reynolds number:
1.23 × 5.99 · 10−6 × 0.015 · 0.3048
Rep = = 1.9 × 10−3 (6.117)
0.018 · 10−3
Thus, our assumption is correct. To calculate Sp we use (6.102)

6/Dp πDp 2 π × (5.99 · 10−6 )2


Sp = = = = 1.6 × 10−10 m2 (6.118)
Φs /Vp Φs 0.7
The surface area per unit mass becomes
6/Dp 6/5.99 · 10−6
Sp /mp = = = 349 m2 /kg (6.119)
ρ p Φs 4.1 · 1000 × 0.7

6.7 Flow in a Fluidized Bed


A fluidized bed is used as a modern process that has the ability to promote high levels of contact
between gases and solids. Let’s see from Fig. 6.23 how the pressure drop in the fluidized bed is related
to a superficial velocity at the bottom of the bed.

Figure 6.23: Pressure drop and bed height versus superficial velocity for a bed of solids

1. Up to Point A: As the superficial velocity v0 increases, the pressure drop increases by the Ergun
equation (6.113), but the particles do not move. At the velocity of point A, the pressure drop
counterbalances the gravity on the particles.
2. Point A to Point B: The bed expands slightly with the grain still in contact due to closed
packing and keep ∆P constant.
6.7. FLOW IN A FLUIDIZED BED 137

3. Point B to Point C: True fluidization begins. The pressure drop keep constant but the height
of the bed increases as increasing the superficial velocity.

If the flow rate is gradually reduced, the pressure drop remains constant, and the bed height decreases,
following the line BC. However,, the final bed height may be greater than the initial value due to less
tight packing.

Minimum Fluidization Velocity


At the point B, the pressure drop across the bed is equal to the weight of the bed per unit area of
cross section, allowing for the buoyant force.6

P2 Fb
S

P1 S + Fb = P2 S + Fe

Fe Fb(buoyance) = mg ρ/ρp
P1
Fe(gravitation) = mg

vOM

Figure 6.24: Force balance at the minimum fluidization velocity.

Fg Fb
∆P = − = ρp L(1 − ε)g − ρL(1 − ε)g = g(ρp − ρ)L(1 − ε) ∵ m = ρp SL(1 − ε) (6.120)
S S

Using (6.113) and (6.120), we have

150µv0M (1 − εM ) 1.75ρv0M 2
+ = g(ρp − ρ) (6.121)
Φs 2 Dp 2 εM 3 Φs D p ε M 3

Case 1: Laminar flow


g(ρp − ρ) εM 3
v0M ≈ Φs 2 Dp 2 (6.122)
150µ 1 − εM

Case 2: Turbulent flow s


Φs Dp g(ρp − ρ)εM 3
v0M ≈ (6.123)
1.75ρ

Ex. 39. For a fluid bed shown in Fig. 6.23, find the ratio of the terminal velocity to the minimum
fluidization velocity.

Answer: Separate the following two cases.


6 More exactly, the pressure drop in the Ergun equation should be the dynamic pressure drop ∆P. Otherwise, Eq.

(6.26) should be used for deriving the Ergun equation. The ρg term in (6.26) indicates the buoyant force in the packing
bed.
138 CHAPTER 6. APPLICATION OF MOMENTUM TRANSFER

Case 1: Very small particles (< 300 µm): Rep < 1, Comparing (6.90) with (6.122), we have

vt 8.33(1 − εM )
= (6.124)
v0M Φs 2 εM 3
For spheres with εM ≈ 0.45,
vt ≈ 50 v0M (6.125)
We can operate the fluidized bed with v0 = 50 v0M without particles carried out with the exit
gas.
Case 2: Very large particles (Rep > 1000): Using (6.123) and (6.91), we have

vt 2.32
= (6.126)
v0M εM 3/2
For spheres with εM ≈ 0.45,
vt ≈ 7.7 v0M (6.127)
which is much lower ratio than for fine particles. This is a slight disadvantage in the use of
coarse particles, but the optimum particle size depends on other factors such as chemical reactor
efficiency, heat- and mass-transfer rates, grinding costs, and the gas velocity.

Summary
6.7. FLOW IN A FLUIDIZED BED 139

베르누이 방정식(마찰 고려)


- Kinetic energy correction factor : 단면에서
v2
위치에 따라 운동에너지가 다르기 때문
P2

%%
v1 #= 층류 ≈ 2, 난류 ≈1
P1 h2 %%
h1 - Frictional energy loss: 마찰에 의한 열 발
P+1/2ρv2+ρgh = consant at points on a streamlne
생으로 역학적에너지가 감소함.
$ %! " $ %# "
!! + #! + $'ℎ! = !# + ## + $'ℎ# + ℎ$
2 2

- Skin Friction
- Friction from sudden
expansion/contraction
- Friction from fittings and valves

- $% "
ℎ$ = ℎ$& + ℎ$' + ℎ$( + ℎ$$ = 4,$ + /( + /' + /$
. 2
화학공학부 울산대학교
SCHOOL OF CHEMICAL ENGINEERING UNIVERSITY OF ULSAN

Skin Friction
- Skin friction: 유체는 고체표면에서 미끄러지지 않으므로 유체가 고체표면을 흐를
때 마찰이 발생함.

Skin Friction = Fanning 마찰계수 ´ 운동에너지 ´ 접촉면적


Ff = ff (Re, k/D) ´ rv2/2 ´ Aw

Ø 층류(low Re) – H.-S. eq.


∆" 32'(
= 16
# )! ,$ =
23
Ø 난류(high Re)
ü Prandtl-Karman (Re > 3✕103)
1
= 4.0 log 23 ,$ − 0.4
,$

ü Blasius (3✕103 <Re<1✕105)


,$ = 0.791 23 )*/,
화학공학부 울산대학교
SCHOOL OF CHEMICAL ENGINEERING UNIVERSITY OF ULSAN
140 CHAPTER 6. APPLICATION OF MOMENTUM TRANSFER

Energy Loss by Friction


• Skin Frictional Energy Loss
# % & !
ℎ!" = 4$!
$ '

• Energy Loss by Friction from sudden expansion


# #
$% .$
ℎ!" = #" , where #" = 1 −
2 .%

• Energy Loss by Friction from sudden contraction

$% # .%
ℎ!& = #& , where #& = 0.4 1 −
2 .$

• Energy Loss by Friction from fittings and valves


$% # #! = 0.75 for 90 elbow
ℎ!! = #! ,
2 = 4.5 for a half open gate valve
4 $% #
ℎ! = ℎ!' + ℎ!" + ℎ!& + ℎ!! = 43! + #& + #" + #!
5 2
화학공학부 울산대학교
SCHOOL OF CHEMICAL ENGINEERING UNIVERSITY OF ULSAN

베르누이 방정식(마찰+펌프)
• 펌프의 효율 및 파워 : 펌프가 한 일중에 유체로 전달된 일
# #
$ %$ $ %%
6$ + 7$ + $8ℎ$ + 9:( = 6% + 7% + $8ℎ% + ℎ!
2 2
- 9: 펌프 효율
- :( : 유체의 단위부피당 펌프가 한일 [J/m3]
- ̇ ( [W]
펌프파워 = =:

• 총괄개발두(Total Developed Head): 펌프에 의하여 증가된 총괄두 [m]


6 %# 6 %# 9:(
∆? = ?")*+ − ?",+"- = + +ℎ − + +ℎ =
$8 28 ")*+
$8 28 ",+"-
$8

• 유효흡입두(Net Positive Suction Head): 펌프 내 cavitation을 피하기 위


하여 필요한 압력 두. 큰 펌프는 15m 이상, 적은 펌프 2-3m 정도임.

6",+"- 6. 6$/ %$ ! # 6. %$ #
NPSH = − ⟹ 펌프위치 = ℎ$ − ℎ$! = + − + NPSH +
$8 $8 $8 28 $8 28
화학공학부 울산대학교
SCHOOL OF CHEMICAL ENGINEERING UNIVERSITY OF ULSAN
6.7. FLOW IN A FLUIDIZED BED 141

부력 및 항력
표면힘 = 정압력힘 + 동압력힘 + 점성힘

부력 (Buoyancy force) 항력 (Drag force)

움직이는 유체(또는 고체)로 인한


깊이에 따른 압력차로 인한 표면힘
표면힘

정압력힘 동압력힘 + 점성힘


(표면의 정압력차에 의한 힘) (표면의 동압력차 및 점성력)

유체의 흐름방향
대체된 유체의 무게
(또는 고체 움직임의 반대 방향)

z 부력 항력
rfluidgVdisplaced r 유체 P∞ - ρgR

FPstat FPmov Fτ
R P∞ P∞
Form drag Wall drag

Moving
v∞ P∞ + ρgR
fluid
3μv∞
3μv 3μ
3μvv∞
Pstat = P∞ - ρgR cosθ Pmov = - cosθ τ= sinθ
2R 2R

화학공학부 울산대학교
SCHOOL OF CHEMICAL ENGINEERING UNIVERSITY OF ULSAN

부력 (Buoyancy Force)
• 물체가 유체에 잠겨 있을 때 높이에 따른 정압력 차이로 발생함.
• 중력에 항상 반대 방향으로 작용.
• 부력의 세기 = 잠긴 부분에 해당하는 유체 무게 = !!"#$% "#%$&'"()*%

a FP1 V
water (ρ)
A P1 Fbuoyancy = FP = ρgV
ρp
Pside
h FP3

Fbody

P2

FP2
https://www.britannica.com/science/Archimedes-principle

화학공학부 울산대학교
SCHOOL OF CHEMICAL ENGINEERING UNIVERSITY OF ULSAN
142 CHAPTER 6. APPLICATION OF MOMENTUM TRANSFER

항력 (Drag force)
• 물체가 유체에 잠겨 있을 때 유체의 흐름(또는 물체의 움
Skin friction drag

직임)에 의하여 유체가 물체에 미치는 힘.


Form drag

• 유체의 흐름방향 또는 물체 움직임의 반대 방향으로 작용

• 압력에 의한 형태항력(form drag)와 skin friction에 의한


벽항력 (wall drag)로 나뉨
on
n

Moving
cti
tio

fri Boundary layer


ric

fluid
n
f

ki
in

S
sk

Wall drag
Stationary Moving
fluid Stationary Form drag
sphere Form drag
sphere

Stagnation
Drag = Form drag + Wall drag point

!! ⁄)%
• 항력 = 항력계수 ✕ 운동에너지 ✕ 투영면적 #! (Re) =
+, $ ⁄2
"# !
!! = #! (Re) × × )%
$ !! cylinder = +,! !! (sphere) = 2 ,! " ⁄4
화학공학부 울산대학교
SCHOOL OF CHEMICAL ENGINEERING UNIVERSITY OF ULSAN

항력: 구를 지나는 유체의 흐름

• When Re% ≪ 1 (관성력 무시)


24 '(
#! = cf. #& =
Re% )*

∵ "! = 3%&'(" from Stokes’ equation

딤플은 상대적으로 낮은 속도에서 난류(작은


소용돌이)를 형성시켜 분리점을 뒤로 밀어내
며, 이로 인하여 항력을 줄어듦.
화학공학부 울산대학교
SCHOOL OF CHEMICAL ENGINEERING UNIVERSITY OF ULSAN
6.7. FLOW IN A FLUIDIZED BED 143

종말속도
"#
부력 Ø 뉴튼의 운동법칙 ' = 중력 − 부력 − 항력
"$
!
= )! *+ − )*+ − ," #$ -% .!
rp 항력
Ø 종말속도 (terminal velocity)
입자의 자유낙하 시 입자의 일정 낙하속도( "#⁄"$ = 0 )
중력
중력 = 부력+항력 2 )! − ) *'
#& =
))! ," .!

ü Low Re (!! = 24⁄Re" )


)! − ) *0! %
#&,(!) =
183

ü High Re (!! = 0.44)


)! − ) 0!
#&,(!) = 1.75
)
화학공학부 울산대학교
SCHOOL OF CHEMICAL ENGINEERING UNIVERSITY OF ULSAN

충전층 압력강하
Ø 충전층 부분에서 압력 강하 ü 상당 채널 지름 (equivalent diameter)
e 충전층 전체 표면적 (Sp´np)
= n개 채널의 전체 표면적 (pDeqL´n)
v D
구형화 2Φ( 0! ?
0+, =
3(1 − ?)
S
L
채널화 ü 구형도 (sphericity)
(S, V 일정)
Vp=Vsphere
Dp

#
D C-./010 6⁄0!
P1 P2 Φ( = = ≤1
C! C! ⁄+!
S
L
ü 평균 속도 (채널)
< )#̅ % #
∆8 = 94;* #̅ =
0+, 2 ?
화학공학부 울산대학교
SCHOOL OF CHEMICAL ENGINEERING UNIVERSITY OF ULSAN
144 CHAPTER 6. APPLICATION OF MOMENTUM TRANSFER

충전층 압력강하 - Ergun Eq.


e

2Φ' (& 5 +
v D (#$ = +̅ =
3(1 − 5) 5

S
L
q 층류 흐름 (low Re) : Kozeny-Carman empirical equation (l1=2.1)

∆" 4 )+̅ % 64 )+̅ % 1503+ 1 − 5 %


= %! &" = %! =
# (#$ 2 Re& (#$ 2 Φ' % (& % 5 (

q 난류 흐름 (high Re) : Burke-Plummer empirical equation (3l2f=1.75)

∆" 4 )+̅ % 1.75)+ % 1 − 5


= %% &" =
# (#$ 2 Φ' (& 5 (

Ergun equation – 충전층에서 층류 및 난류 유동

∆" 1503+ 1 − 5 % 1.75)+ % 1 − 5


= +
# Φ' % (& % 5 ( Φ' (& 5 (
화학공학부 울산대학교
SCHOOL OF CHEMICAL ENGINEERING UNIVERSITY OF ULSAN

유동층 압력강하 및 최소 유동화 속도


q 최소 유동화 속도 !!"

∆#$! = 중력 − 부력 = '# () − '() * = +" , 1 − $!


P2
S0 부력
1503+)* 1 − 5* % 1.75)+)* % 1 − 5*
∆" = +
Φ' % (& % 5* ( Φ' (& 5* (

중력 ü Small particles(<300mm): Rep<1


v0M
P1
Air )& − ) >5* ( % %
+)* ≈ Φ (
1503 1 − 5* ' &
++ ≈ 50+)*
If $! = 0.45

ü Large particles(>1mm): Rep>1000

)& − ) >5* (
+)* ≈ Φ' (& ++ ≈ 7.7+)*
1.75)
If $! = 0.45
화학공학부 울산대학교
SCHOOL OF CHEMICAL ENGINEERING UNIVERSITY OF ULSAN
Appendix A

Summary of Differential Vector


Operations

A.1 Rectangular Cartesian Coordinates


Gradient

∂P ∂P ∂P
∇P = ex + ey + ez (A.1)
∂x ∂y ∂z

∂vx ∂vx ∂vx ∂vy ∂vy ∂vy


∇v = ex ex + ex ey + ex ez + ey ex + ey ey + ey ez
∂x ∂y ∂z ∂x ∂y ∂z
(A.2)
∂vz ∂vz ∂vz
+ ez ex + ez ey + ez ez
∂x ∂y ∂z

Divergence

∂vx ∂vy ∂vz


∇·v = + + (A.3)
∂x ∂y ∂z

     
∂τxx ∂τxy ∂τxz ∂τyx ∂τyy ∂τyz ∂τzx ∂τzy ∂τzz
∇·τ = + + ex + + + ey + + + ez
∂x ∂y ∂z ∂x ∂y ∂z ∂x ∂y ∂z
(A.4)

Curl
     
∂vz ∂vy ∂vx ∂vz ∂vy ∂vx
∇× v= − ex + − ey + − ez (A.5)
∂y ∂z ∂z ∂x ∂x ∂y

Laplacian

∂2T ∂2T ∂2T


∇2 T = 2 + 2 + (A.6)
∂x ∂y ∂z 2

∂ 2 vx ∂ 2 vx ∂ 2 vx ∂ 2 vy ∂ 2 vy ∂ 2 vy ∂ 2 vz ∂ 2 vz ∂ 2 vz
     
2
∇ v= + + ex + + + ey + + + ez (A.7)
∂x2 ∂y 2 ∂z 2 ∂x2 ∂y 2 ∂z 2 ∂x2 ∂y 2 ∂z 2

145
146 APPENDIX A. SUMMARY OF DIFFERENTIAL VECTOR OPERATIONS

A.2 Cylindrical Coordinates


Gradient
∂P 1 ∂P ∂P
∇P = er + eθ + ez (A.8)
∂r r ∂θ ∂z
   
∂vr 1 ∂vr vθ ∂vr ∂vθ 1 ∂vθ vr ∂vθ
∇v = er er + − er eθ + er ez + eθ er + + eθ eθ + eθ ez
∂r r ∂θ r ∂z ∂r r ∂θ r ∂z
∂vz 1 ∂vz ∂vz
+ ez er + ez eθ + ez ez
∂r r ∂θ ∂z
(A.9)

Divergence
1 ∂ 1 ∂vθ ∂vz
∇·v = (rvr ) + + (A.10)
r ∂r r ∂θ ∂z

   
1 ∂ 1 ∂τrθ ∂τrz τθθ 1 ∂ 2
 1 ∂τθθ ∂τθz
∇·τ = (rτrr ) + + − er + 2 r τθr + + eθ
r ∂r r ∂θ ∂z r r ∂r r ∂θ ∂z
  (A.11)
1 ∂ 1 ∂τzθ ∂τzz
+ (rτzr ) + + ez
r ∂r r ∂θ ∂z

Curl
     
1 ∂vz ∂vθ ∂vr ∂vz 1 ∂ ∂vr
∇× v= − er + − eθ + (rvθ ) − ez (A.12)
r ∂θ ∂z ∂z ∂r r ∂r ∂θ

Laplacian

1 ∂2T ∂2T
 
1 ∂ ∂T
∇2 T = r + 2 2 + (A.13)
r ∂r ∂r r ∂θ ∂z 2

1 ∂ 2 vr ∂ 2 vr
   
∂ 1 ∂ 2 ∂vθ
∇2 v = (rvr ) + 2 − + er
∂r r ∂r r ∂θ2 r2 ∂θ ∂z 2
1 ∂ 2 vθ ∂ 2 vθ
   
∂ 1 ∂ 2 ∂vr
+ (rvθ ) + 2 + 2 + eθ (A.14)
∂r r ∂r r ∂θ2 r ∂θ ∂z 2
1 ∂ 2 vz ∂ 2 vz
   
1 ∂ ∂vz
+ r + 2 2 + ez
r ∂r ∂r r ∂θ ∂z 2

A.3 Spherical Coordinates


Gradient
∂P 1 ∂P 1 ∂P
∇P = er + eθ + eϕ (A.15)
∂r r ∂θ r sin θ ∂ϕ
     
∂vr vθ 1 ∂vr 1 ∂vr vϕ ∂vθ vr 1 ∂vθ
∇v = er er + − + er eθ + − er eϕ + eθ er + + eθ eθ
∂r r r ∂θ r sin θ ∂ϕ r ∂r r r ∂θ
   
1 ∂vθ vϕ ∂vϕ 1 ∂vϕ 1 ∂vϕ vr + vθ cot θ
+ − cot θ eθ eϕ + eϕ er + eϕ eθ + + eϕ eϕ
r sin θ ∂ϕ r ∂r r ∂θ r sin θ ∂ϕ r
(A.16)
A.4. VECTOR CALCULATION 147

Divergence
1 ∂ 1 ∂ 1 ∂vϕ
r 2 vr +

∇·v = 2
(vθ sin θ) + (A.17)
r ∂r r sin θ ∂θ r sin θ ∂ϕ

 
1 ∂ 2
 1 ∂ 1 ∂τrϕ τθθ + τϕϕ
∇·τ = r τrr + (τ rθ sin θ) + − er
r2 ∂r r sin θ ∂θ r sin θ ∂ϕ r
 
1 ∂ 1 ∂ 1 ∂τθϕ τϕϕ cot θ
r3 τθr +

+ 3 (τθθ sin θ) + − eθ (A.18)
r ∂r r sin θ ∂θ r sin θ ∂ϕ r
 
1 ∂ 1 ∂ 1 ∂τϕϕ τθϕ cot θ
r3 τϕr +

+ 3 (τϕθ sin θ) + + eϕ
r ∂r r sin θ ∂θ r sin θ ∂ϕ r

Curl
     
1 ∂ 1 ∂vθ 1 ∂vr 1 ∂ 1 ∂ 1 ∂vr
∇× v= (vϕ sin θ) − er + − (rvϕ ) eθ + (rvθ ) − er
r sin θ ∂θ r sin θ ∂ϕ r sin θ ∂ϕ r ∂r r ∂r r ∂θ
(A.19)

Laplacian
∂2T
   
1 ∂ ∂T 1 ∂ ∂T 1
∇2 T = r2 + sin θ + (A.20)
r2 ∂r ∂r r2 sin θ ∂θ ∂θ 2
r2 sin θ ∂ϕ2

1 ∂2 ∂ 2 vr
   
2 2
 1 ∂ ∂vr 1
∇ v = 2 2 r vr + 2 sin θ + 2 2 er
r ∂r r sin θ ∂θ ∂θ r sin θ ∂ϕ2
∂ 2 vθ
     
1 ∂ 2 ∂vθ 1 ∂ 1 ∂ 1 2 ∂vr 2 cot θ ∂vϕ
+ 2 r + 2 (vθ sin θ) + 2 2 + 2 − 2 eθ
r ∂r ∂r r ∂θ sin θ ∂θ r sin θ ∂ϕ2 r ∂θ r sin θ ∂ϕ
∂ 2 vϕ
     
1 ∂ ∂vϕ 1 ∂ 1 ∂ 1 2 ∂vr 2 cot θ ∂vθ
+ 2 r2 + 2 (vϕ sin θ) + 2 2 + + eϕ
r ∂r ∂r r ∂θ sin θ ∂θ r sin θ ∂ϕ2 r2 sin θ ∂ϕ r2 sin θ ∂ϕ
(A.21)

A.4 Vector Calculation


ex ey ez


∂/∂x ∂/∂y ∂/∂z

∇ × (∇ × v) =
∂vz ∂vy ∂vx ∂vz ∂vy ∂vx
− − −
∂y ∂z ∂z ∂x ∂x ∂y

 2
∂ 2 vy ∂ 2 vz ∂ 2 vx ∂ 2 vx ∂ 2 vx

∂ vx
= + + − − − ex
∂x2 ∂x∂y ∂x∂z ∂x2 ∂y 2 ∂z 2
 2
∂ 2 vz ∂ 2 vx ∂ 2 vy ∂ 2 vy ∂ 2 vy

∂ vy
+ + + − − − ey
∂y 2 ∂y∂z ∂x∂y ∂x2 ∂y 2 ∂z 2
 2
∂ 2 vx ∂ 2 vy ∂ 2 vz ∂ 2 vz ∂ 2 vz

∂ vz
+ + + − − − ez
∂z 2 ∂x∂z ∂y∂z ∂x2 ∂y 2 ∂z 2
= ∇(∇ · v) − ∇2 v
148 APPENDIX A. SUMMARY OF DIFFERENTIAL VECTOR OPERATIONS
Appendix B

Incompressible and Irrotational


Flow

If a fluid is incompressible (ρ = constant), the volume of the fluid keeps constant during a deformation
(i. e., isochoric deformation). However the converse is not true. A compressible body can also undergo
an isochoric motion (or incompressible flow). Now we calculate the ratio of volumetric flux through
an element to the original volume of the element shown in Fig. B.1

(x+∆x, y+∆y, z+∆z)

(x, y, z)

Figure B.1: Volumetric flux through a differential control volume

Volumetric Flux (net volume per unit area and unit time)
Original volume
[(vx |x+∆x − vx |x )∆y∆z + (vy |y+∆y − vy |y )∆x∆z + (vz |z+∆z − vz |z )∆x∆y]
= (B.1)
∆x∆y∆z
In the limit ∆x, ∆y, ∆z → 0, it becomes
[(vx |x+∆x − vx |x )∆y∆z + (vy |y+∆y − vy |y )∆x∆z + (vz |z+∆z − vz |z )∆x∆y]
lim
∆x,∆y,∆z→0 ∆x∆y∆z
∂vx ∂vy ∂vz (B.2)
= + +
∂x ∂y ∂z
=∇·v
For an incompressible flow, the divergence of velocity should be zero:
Volumetric increase by flux
∇·v =
Original volume (B.3)
=0

149
150 APPENDIX B. INCOMPRESSIBLE AND IRROTATIONAL FLOW

As shown in Fig. B.2, let’s consider an element deformed by translational and rotational motions
in the xy plane during time ∆t The average angular velocity (the average rotation at a point) about

Figure B.2: Rotation of a fluid element.

the z axis can be expressed as  


d α+β
ωz = (B.4)
dt 2
where the counterclockwise sense is positive. From the figure, it can be seen that

(vy |x+∆x − vy |x )∆t −(vx |y+∆y − vx |y )∆t


tan α = and tan β = (B.5)
∆x ∆y

Thus equation (B.4) becomes


     
1 (vy |x+∆x − vy |x )∆t −(vx |y+∆y − vx |y )∆t
ωz = lim arctan + arctan ∆t
∆x,∆y,∆z,∆t→0 2 ∆x ∆y
(B.6)
In the limit, it reduces to
 
1 ∂vy ∂vx arctan(a∆t)
ωz = − , ∵ lim =a (B.7)
2 ∂x ∂y ∆t→0 ∆t

In the xz and yz planes the rotations of a point about the y and x axises are given by
   
1 ∂vx ∂vz 1 ∂vz ∂vy
ωy = − and ωx = − (B.8)
2 ∂z ∂x 2 ∂y ∂z

Thus it can be written as


     
∂vz ∂vy ∂vx ∂vz ∂vy ∂vx
∇× v= − ex + − ey + − ez
∂y ∂z ∂z ∂x ∂x ∂y (B.9)
= 2ω

where ω is called the vorticity vector. As the derivation of the word ‘vorticity’ from ‘vortex’ (a swirling
body of fluid) suggests, ω is a measure of rotational motion. If a flow is irrotational, then

∇ × v=0 (B.10)
Appendix C

Equation of Continuity and Motion


for an Incompressible Flow

∂ρ
+ ∇ · (ρv) = 0 =⇒ ∇ · v = 0
∂t
 
∂v
ρ + (∇v) · v = −∇P + ∇ · τ + ρb ∵ T = −P I + τ
∂t

If a fluid is Newtonian
 
∂v
ρ + (∇v) · v = −∇P + µ∇2 v + ρb ∵ τ = µ(∇v + ∇vT )
∂t

Rectangular Cartesian coordinates (x, y, z)

• Mass balance
∂vx ∂vy ∂vz
+ + =0
∂x ∂y ∂z

• Momentum balance

– x-component
   
∂vx ∂vx ∂vx ∂vx ∂P ∂τxx ∂τyx ∂τzx
ρ + vx + vy + vz =− + + + + ρbx
∂t ∂x ∂y ∂z ∂x ∂x ∂y ∂z

– y-component
   
∂vy ∂vy ∂vy ∂vy ∂P ∂τxy ∂τyy ∂τzy
ρ + vx + vy + vz =− + + + + ρby
∂t ∂x ∂y ∂z ∂y ∂x ∂y ∂z

– z-component
   
∂vz ∂vz ∂vz ∂vz ∂P ∂τxz ∂τyz ∂τzz
ρ + vx + vy + vz =− + + + + ρbz
∂t ∂x ∂y ∂z ∂z ∂x ∂y ∂z

• Components of stress
   
Txx Txy Txz −P + τxx τxy τxz
Tyx Tyy Tyz  =  τyx −P + τyy τyz 
Tzx Tzy Tzz τzx τzy −P + τzz

151
152APPENDIX C. EQUATION OF CONTINUITY AND MOTION FOR AN INCOMPRESSIBLE FLOW

If a fluid is Newtonian
    
∂vx ∂vx ∂vy ∂vx ∂vz
2µ µ + µ +
   ∂x ∂y ∂x  ∂z ∂x 
τxx τxy τxz    
τyx
 ∂vx ∂vy ∂vy ∂vy ∂vz 
τyy τyz = µ
  + 2µ µ + 
τzx τzy τzz   ∂y ∂x   ∂y  ∂z ∂y 
 ∂vx ∂vz ∂vy ∂vz ∂vz 
µ + µ + 2µ
∂z ∂x ∂z ∂y ∂z

• Momentum balance
– x-component
 2
∂ 2 vx ∂ 2 vx
  
∂vx ∂vx ∂vx ∂vx ∂P ∂ vx
ρ + vx + vy + vz =− +µ + + + ρbx
∂t ∂x ∂y ∂z ∂x ∂x2 ∂y 2 ∂z 2

– y-component
 2
∂ 2 vy ∂ 2 vy
  
∂vy ∂vy ∂vy ∂vy ∂P ∂ vy
ρ + vx + vy + vz =− +µ + + + ρby
∂t ∂x ∂y ∂z ∂y ∂x2 ∂y 2 ∂z 2

– z-component
 2
∂ 2 vz ∂ 2 vz
  
∂vz ∂vz ∂vz ∂vz ∂P ∂ vz
ρ + vx + vy + vz =− +µ + + + ρbz
∂t ∂x ∂y ∂z ∂z ∂x2 ∂y 2 ∂z 2

Cylindrical coordinates (r, θ, z)

• Mass balance
1 ∂ 1 ∂vθ ∂vz
(rvr ) + + =0
r ∂r r ∂θ ∂z
• Momentum balance
– r-component

vθ 2
   
∂vr ∂vr vθ ∂vr ∂vr ∂P 1 ∂ 1 ∂τθr ∂τzr τθθ
ρ + vr + − + vz =− + (rτrr ) + + − +ρbr
∂t ∂r r ∂θ r ∂z ∂r r ∂r r ∂θ ∂z r

– θ-component
   
∂vθ ∂vθ vθ ∂vθ vr vθ ∂vθ 1 ∂P 1 ∂  1 ∂τθθ ∂τzθ
ρ + vr + + + vz =− + 2 r2 τrθ + + +ρbθ
∂t ∂r r ∂θ r ∂z r ∂θ r ∂r r ∂θ ∂z

– z-component
   
∂vz ∂vz vθ ∂vz ∂vz ∂P 1 ∂ 1 ∂τθz ∂τzz
ρ + vr + + vz =− + (rτrz ) + + + ρbz
∂t ∂r r ∂θ ∂z ∂z r ∂r r ∂θ ∂z

If a fluid is Newtonian
    
∂vr vθ 1 ∂vr ∂vθ ∂vr ∂vz
2µ µ − + + µ +
   ∂r r r ∂θ ∂r  ∂z ∂r 
τrr τrθ τrz     
 v θ 1 ∂vr ∂vθ v r 1 ∂v θ ∂vθ ∂vz 
µ − r + r ∂θ + ∂r
τθr τθθ τθz  =  2µ + µ + 
τzr τzθ τzz     r r ∂θ  ∂z ∂θ 
 ∂vr ∂vz ∂vθ ∂vz ∂vz 
µ + µ + 2µ
∂z ∂r ∂z ∂θ ∂z

• Momentum balance
153

– r-component
vθ 2
 
∂vr ∂vr vθ ∂vr ∂vr
ρ + vr + − + vz
∂t ∂r r ∂θ r ∂z
1 ∂ 2 vr ∂ 2 vr
   
∂P ∂ 1 ∂ 2 ∂vθ
=− +µ (rvr ) + 2 − + + ρbr
∂r ∂r r ∂r r ∂θ2 r2 ∂θ ∂z 2
– θ-component
 
∂vθ ∂vθ vθ ∂vθ vr vθ ∂vθ
ρ + vr + + + vz
∂t ∂r r ∂θ r ∂z
1 ∂ 2 vθ ∂ 2 vθ
   
1 ∂P ∂ 1 ∂ 2 ∂vr
=− +µ (rvθ ) + 2 + 2 + + ρbθ
r ∂θ ∂r r ∂r r ∂θ2 r ∂θ ∂z 2
– z-component
 
∂vz ∂vz vθ ∂vz ∂vz
ρ + vr + + vz
∂t ∂r r ∂θ ∂z
1 ∂ 2 vz ∂ 2 vz
   
∂P 1 ∂ ∂vz
=− +µ r + 2 + + ρbz
∂z r ∂r ∂r r ∂θ2 ∂z 2

Spherical coordinates (r, θ, ϕ)


• Mass balance
1 ∂ 2 1 ∂ 1 ∂vϕ
2
(r vr ) + (vθ sin θ) + =0
r ∂r r sin θ ∂θ r sin θ ∂ϕ
• Momentum balance
– r-component
vθ 2 + vϕ 2
 
∂vr ∂vr vθ ∂vr vϕ ∂vr
ρ + vr + − +
∂t ∂r r ∂θ r r sin θ ∂ϕ
 
∂P 1 ∂ 1 ∂ 1 ∂τϕr τθθ + τϕϕ
r2 τrr +

=− + 2 (τθr sin θ) + − + ρbr
∂r r ∂r r sin θ ∂θ r sin θ ∂ϕ r
– θ-component
vϕ 2 cot θ
 
∂vθ ∂vθ vθ ∂vθ vr v θ vϕ ∂vθ
ρ + vr + + + −
∂t ∂r r ∂θ r r sin θ ∂ϕ r
 
1 ∂P 1 ∂ 1 ∂ 1 ∂τϕθ τϕϕ cot θ
r3 τrθ +

=− + ρbθ + 3 (τθθ sin θ) + −
r ∂θ r ∂r r sin θ ∂θ r sin θ ∂ϕ r
– ϕ-component
 
∂vϕ ∂vϕ vθ ∂vϕ vr vϕ vθ vϕ cot θ vϕ ∂vϕ
ρ + vr + + + +
∂t ∂r r ∂θ r r r sin θ ∂ϕ
 
1 ∂P 1 ∂ 3
 1 ∂ 1 ∂τϕϕ τϕθ cot θ
=− + ρbϕ + 3 r τrϕ + (τθϕ sin θ) + +
r sin θ ∂ϕ r ∂r r sin θ ∂θ r sin θ ∂ϕ r
If a fluid is Newtonian
 
τrr τrθ τrϕ
 τθr τθθ τθϕ 
τϕr τϕθ τϕϕ
     
∂vr vθ 1 ∂vr ∂vθ 1 ∂vr ∂vϕ vϕ
2µ µ − + + µ + −

  ∂r r r ∂θ ∂r  r sin θ ∂ϕ ∂r r 
v 1 ∂v ∂v

v 1 ∂v

1 ∂v sin θ ∂ h v i
 θ r θ r θ θ ϕ 
=  µ − r + r ∂θ + ∂r 2µ + µ + 
    r r ∂θ  r sin θ ∂ϕ
 r ∂θ sin θ  
 1 ∂vr ∂vϕ vϕ 1 ∂vθ sin θ ∂ h vϕ i 1 ∂vϕ vr + vθ cot θ 
µ + − µ + 2µ +
r sin θ ∂ϕ ∂r r r sin θ ∂ϕ r ∂θ sin θ r sin θ ∂ϕ r
154APPENDIX C. EQUATION OF CONTINUITY AND MOTION FOR AN INCOMPRESSIBLE FLOW

• Momentum balance
– r-component

vθ 2 + vϕ 2
 
∂vr ∂vr vθ ∂vr vϕ ∂vr
ρ + vr + − +
∂t ∂r r ∂θ r r sin θ ∂ϕ
2
∂ 2 vr
   
∂P 1 ∂ 1 ∂ ∂vr 1
+ µ 2 2 r 2 vr + 2

=− sin θ + 2 2 + ρbr
∂r r ∂r r sin θ ∂θ ∂θ r sin θ ∂ϕ2

– θ-component

vϕ 2 cot θ
 
∂vθ ∂vθ vθ ∂vθ vr vθ vϕ ∂vθ 1 ∂P
ρ + vr + + + − =− + ρbθ
∂t ∂r r ∂θ r r sin θ ∂ϕ r r ∂θ
∂ 2 vθ
     
1 ∂ 2 ∂vθ 1 ∂ 1 ∂ 1 2 ∂vr 2 cot θ ∂vϕ
+µ 2 r + 2 (vθ sin θ) + 2 2 + 2 − 2
r ∂r ∂r r ∂θ sin θ ∂θ r sin θ ∂ϕ2 r ∂θ r sin θ ∂ϕ

– ϕ-component
 
∂vϕ ∂vϕ vθ ∂vϕ vr vϕ vθ vϕ cot θ vϕ ∂vϕ 1 ∂P
ρ + vr + + + + =− + ρbϕ
∂t ∂r r ∂θ r r r sin θ ∂ϕ r sin θ ∂ϕ
∂ 2 vϕ
     
1 ∂ ∂vϕ 1 ∂ 1 ∂ 1 2 ∂vr 2 cot θ ∂vθ
+µ 2 r2 + 2 (vϕ sin θ) + 2 2 + +
r ∂r ∂r r ∂θ sin θ ∂θ r sin θ ∂ϕ2 r2 sin θ ∂ϕ r2 sin θ ∂ϕ
Appendix D

Stream Function and Streamline for


Incompressible Fluids

The stream function can be used to visualize a velocity field that has been found either analytically
or numerically. It is applicable to incompressible flows that are bidirectional. There is little need for
Ψ in a unidirectional flow, and Ψ is undefined if there are three velocity components or if ρ varies.
Many flows of technological or scientific interest are unidirectional or bidirectional, provided that the
coordinate system is chosen properly. Bidirectional flows fall into two broad categories, according to
the type of symmetry that causes the third velocity component to be absent. In planar flow, vz = 0
and the other velocity components are independent of z. Planar stagnation flow (Example 5.2-1) is
an example. Depending on what is being modeled, a planar problem might be analyzed using either
Cartesian (x, y) or cylindrical (r, θ) coordinates. In axisymmetric flows there is rotational symmetry
about the z axis, such that all quantities are independent of a cylindrical or spherical angle.
For a two-dimensional incompressible flow shown in Fig. D.1, the continuity equation is

∂vx ∂vy
+ =0 (D.1)
∂x ∂y

Let
∂Ψ
vx ≡ (D.2)
∂y
Then (D.1) becomes
 
∂ ∂Ψ
+ vy =0 (D.3)
∂y ∂x
for this to be true in general
∂Ψ
vy = − (D.4)
∂x
Instead of having 2 unknowns vx , vy , we have only one unknown Ψ(x, y), called the stream function.

Figure D.1: Streamlines and the stream function

155
156APPENDIX D. STREAM FUNCTION AND STREAMLINE FOR INCOMPRESSIBLE FLUIDS

The total derivative of Ψ is


∂Ψ ∂Ψ
dΨ = dx + dy
∂x ∂y (D.5)
= −vy dx + vx dy

What makes the stream function valuable for flow visualization is that constant values of Ψ correspond
to streamlines. A streamline is a curve that is tangent everywhere to the velocity vector. Consider a
path in the xy plane such that Ψ=constant. Along this path, dΨ=0, and the equation becomes

dy vy
= (D.6)
dx Ψ=constant
vx

This is the equation of the streamlines.


There are three field lines in a fluid flow.
1. Streamline: A curve tangent to the particle’s velocity vector at every point in space at a particle
time. The particle’s velocity in a flow should be only one so that streamlines can’t across each
other.
2. Path line: The curve in space along which the material particle travels is referred to as the
path line for the material particle. The pathlines can across each other.

3. Streak Line: The streak line through the point x(0) at time t represents the positions at time
t of the material particles that at any time τ ≤ t have occupied the place x(0) ., i.e., the locus of
points of all the fluid particles that have passed continuously through a particular spatial point
in the past. Dye steadily injected into the fluid at a fixed point extends along a streakline.
For one particular type of flow v ̸= f (t), the streamline, the path line, and the streak line coincide.
Appendix E

Dimensional Analysis

To group the variables into dimensionless parameters that are less numerous than the original vari-
ables. By combining the variables into a smaller number of dimensionless parameters, the work of
experimental data reduction is considerably reduced.

z =x+y (E.1)

If we let
z x
z⋆ ≡ x⋆ ≡ (E.2)
y y
then (E.1) becomes
z ⋆ = x⋆ + 1 (E.3)

E.1 Dimensions
Fundamental dimensions: Mass, Length, Time

E.2 Dimensional Analysis


Arbitrary reference values: L0 , t0 , v0 , g0 , P0
x y z v0 t v g P
x⋆ ≡ y⋆ ≡ z⋆ ≡ t⋆ ≡ v⋆ ≡ g⋆ ≡ P⋆ ≡ (E.4)
L0 L0 L0 L0 v0 g0 P0

157
158 APPENDIX E. DIMENSIONAL ANALYSIS

From this we can see


∇⋆ = L0 ∇ ∆⋆ = L0 2 ∆ (E.5)
Now the continuity and the Navier-Stokes equations may easily be rearranged to the form

∇⋆ · v ⋆ = 0 (E.6)

and
∂v⋆ 1 1 ⋆
+ (∇⋆ v⋆ ) · v⋆ = −NEu ∇⋆ P ⋆ + ∇⋆ 2 v ⋆ + g (E.7)
∂t⋆ NRe NF r
Here NEu , NRe , and NF r are the Euler number, the Reynolds number, and the Froude number defined
as
P0 ρv0 L0 v0 2
NEu ≡ , NRe ≡ , N F r ≡ (E.8)
ρv0 2 µ g0 L0

E.3 The Buckingham Method


There are many situations of interest in which the governing equation such as the differential momen-
tum balance is not known. In this case we need an alternative method for dimensional analysis.

Buckingham pi theorem
The number of dimensionless groups used to describe a situation involving in variables is equal to
n − r, where r is the rank of the dimensional matrix of the variables and n is the number of variables.

Ex. 40. Determine the dimensionless groups formed from the variables involved in the flow of fluid
external to a solid body. The force exerted on the body is a function of v, ρ, µ, and D(a significant
dimension of the body).
F = F (v, ρ, µ, D)

Answer: Do the following steps


E.3. THE BUCKINGHAM METHOD 159

1. Construct the dimensional matrix of the variables and their dimensions.

F [kg m/s2 ] v[m/s] ρ[kg/m3 ] µ[kg/m s] D[m]


M 1 0 1 1 0
(E.9)
L 1 1 −3 −1 1
t −2 −1 0 −1 0

2. Find the rank of the matrix (see Appendix E.3.1)


     
1 0 1 1 0 1 0 1 1 0 1 0 1 1 0
1 1 −3 −1 1 → 0 −1 4 2 −1 → 0 1 −4 −2 1
−2 −1 0 −1 0 0 −1 2 1 0 0 0 −2 −1 1
  (E.10)
1 0 0 1/2 1/2
→ 0 1 0 0 −1  ∴ Rank = 3
0 0 1 1/2 −1/2

The number of dimensionless parameters = 5 -3 =2 (π1 , π2 )


3. Find the three (rank) core parameters that will appears simultaneously in the two dimension-
less parameter π1 and π2 . F shouldn’t be chosen, v, ρ, and D which include M , L, and t will be
chosen.

π1 = v a ρb D c F
(E.11)
π2 = v d ρe D f µ

Calculate a, b, c, and d, e, f to satisfy


 a  b
L M c ML
M 0 L0 t0 = 3
(L)
t L t2
 d  e
L M f M (E.12)
M 0 L0 t0 = (L)
t L3 Lt
∴ a = −2, b = −1, c = −2, and d = −1, e = −1 f = −1

Thus
F F/D2
π1 = =
ρv 2 D2 ρv 2 (E.13)
µ
π2 = = 1/Re
ρvD

The dimensional analysis has enabled us to relate the original five variables in terms of only two
dimensionless parameters in the form
F/D2
= f¯(Re) (E.14)
ρv 2

E.3.1 Rank of a Matrix


The rank of a matrix is the maximum number of independent rows or columns.
The easiest way to compute the rank of a matrix A is given by the Gauss elimination method: by
the following transformations

1. Row-switching transformation

2. Row-multiplying transformation

3. Row-addition transformation
160 APPENDIX E. DIMENSIONAL ANALYSIS

a matrix can be transformed to the form shown below


   
r11 r12 r13 r14 r15 1 0 0 0 a
r21 r22 r23 r24 r25  row 0 1 0 0 b
  −−−−−−−−−−→   (E.15)
r31 r32 r33 r34 r35  transformations 0 0 1 0 c
r41 r42 r43 r44 r45 0 0 0 1 d

The rank of this matrix is 4.


Now, determine the rank of the matrix given in Section E.3.
     
1 0 1 1 0 1 0 1 1 0 1 0 1 1 0
r −r2 →r2 −r2 →r2
1 1 −3 −1 1 −−1−−− −−→ 0 −1 4 2 −1 −−−− −−−→ 0 1 −4 −2 1
2r1 +r3 →r3 r3 −r2 →r3
−2 −1 0 −1 0 0 −1 2 1 0 0 0 −2 −1 1

 
1 0 0 1/2 1/2
r1 +r3 /2→r1
−−−−−−−−−−−−−−→ 0 1 0 0 −1 
r2 −2r3 →r2 ,−r3 /2→r3
0 0 1 1/2 −1/2
(E.16)

Therefore the rank of the matrix is 3 due to three independent rows.

You might also like